Pharmacist MCQ Review 2019

Download as pdf or txt
Download as pdf or txt
You are on page 1of 316
At a glance
Powered by AI
The document provides an overview of a book aimed at preparing readers for Canadian pharmacy licensing exams. It discusses the author's qualifications and experience in pharmacy education and licensing exam preparation.

The book is titled 'Canadian Pharmacy Exams - Pharmacist MCQ Review' and aims to prepare readers for the Qualifying Exam Part 1 (MCQ) which is one of the exams required for Canadian pharmacy licensure.

The author, Dr. Fatima S. Marankan, holds a postgraduate pharmacy degree and has extensive experience in pharmacy instruction and education. She has received awards for her academic and teaching achievements.

Canadian Pharmacy Exams

Pharmacist MCQ Review

Dr. Fatima S. Marankan


Phi Publishing

Canadian Pharmacy Exams – Pharmacist MCQ Review

Copyright © 2019 by Phi Publishing

All rights reserved. No part of this publication may be reproduced, stored in a


retrieval system, or transmitted in any form or by any means, electronic,
mechanical, photocopying, recording, or otherwise, without the prior written
permission of the copyrights owner.

Pharmacy is an ever-changing science. As new research and clinical


experience broaden our knowledge, changes in treatment and drug therapy
are needed. The author and contributors of Canadian Pharmacy Exams –
Pharmacist MCQ Review have checked with resources believed to be reliable
in their efforts to provide information that is complete and generally in accord
with the standards accepted at the time of publication. However, in view of
the possibility of human error or changes in medical sciences, neither the
author nor any other party who has been involved in the preparation or
publication of this work warrants that the information contained herein is in
every respect accurate or complete and they disclaim all responsibility for
any errors or omissions or for the results obtained from the use of the
information contained in this work.
Library and Archives Canada Cataloguing in Publication

Phi Publishing
Canadian Pharmacy Exams Pharmacist MCQ Review / Author, Dr. Fatima S.
Marankan – 5th Canadian Edition.

About the Author

Dr. Marankan holds a postgraduate degree in pharmacy from the College of


Pharmacy at UIC, USA coupled with extensive experience in pharmacy
instruction at the University of British Columbia, Canada. Dr. Marankan was
recently a visiting medical professor. Her academic, research and teaching
achievements have been recognized by the Paul Sang Award at the
University of Illinois at Chicago and the TLEF Award at the University of
British Columbia, Canada. Furthermore, Dr. Marankan was the lead
consultant in the development and implementation of Pharmacist OSCE
training in Vancouver, Canada.

Throughout her education and career as pharmacy instructor at the University


of British Columbia, Fatima has gained extensive understanding of the
requirements of pharmacy licensing exams in Canada. This knowledge has
guided the development of the Canadian Pharmacy Exams Series:

- Pharmacist Evaluating Exam Practice - Volume 1


- Pharmacist Evaluating Exam Practice - Volume 2
- Pharmacist MCQ Review
- Pharmacist OSCE Workbook
- Pharmacy Technician MCQ Review
- Pharmacy Technician OSPE Workbook

Thank you to all contributing reviewers!

Preface

The Qualifying Exam Part I (MCQ) is a primary exam towards Canadian


pharmacy licensure. The MCQ has been designed to evaluate the knowledge
of Canadian Pharmacy Graduates and International Pharmacy Graduates
seeking licensure. A candidate for the MCQ must be prepared to demonstrate
knowledge in the following nine competencies as per the Pharmacy
Examining Board of Canada (PEBC®):
- Patient Care
- Health Promotion
- Intra and Inter-Professional Collaboration
- Knowledge and Research Application
- Ethical, Legal and Professional Responsibilities
- Quality and Safety
- Communication and Education
- Product Distribution
- Practice Setting

Canadian Pharmacy Exams™ - Pharmacist MCQ Review is designed as a self-


study tool to help the student seeking pharmacy licensure in Canada test
his/her exam readiness, identify areas of strength and weakness. The book
provides and extensive review of the nine competencies listed above. It
contains a mix of 470 exam-type questions and answers and case scenarios
with solutions. In addition, 26 appendices have been included to summarize
important drug related adverse effects and interactions. Finally, 5 bonus
OSCE cases are included. Canadian Pharmacy Exams™ - Pharmacist MCQ
Review 5th edition has been fully updated according to PEBC curriculum.
New Questions!
New Appendices!
New Topics: Canadian Cannabis and Physical Assessment Skills

All answers are supplemented by numerous comments and explanations to


ensure further understanding and learning of new concepts. These comments
are truly the keystone of Canadian Pharmacy Exams™. We trust that each
Canadian Pharmacy Exams™ book is a valuable learning and self-assessment
tool towards Canadian Pharmacy licensure. The following book is now
available at Amazon: Canadian Pharmacy Exams™ - Pharmacist OSCE
Workbook

Trusted Convenient Comprehensive Canadian Pharmacy Exams™ Online


Review for Pharmacists and Pharmacy Technicians at
www.cpepreponline.com

FREE Computer-Based Exam Readiness Tests


FREE Pharmacy Resources
References

- Compendium of Therapeutic Choices, 2019


- Compendium of Pharmaceuticals and Specialties, 2019
- Compendium of Products for Minor Ailments, 2019
- Rx Files, 2017
- Drug Facts and Comparisons, 2017
Content

PATIENT CARE AND HEALTH PROMOTION


Questions 7
Answers 20

INTRA AND INTER-PROFESSIONAL COLLABORATION


KNOWLEDGE AND RESEARCH
APPLICATION
Questions 25
Answers 34

ETHICAL, LEGAL AND PROFESSIONAL RESPONSIBILITIES


Questions 37
Answers 41

QUALITY AND SAFETY


Questions 43
Answers 50

COMMUNICATION AND EDUCATION


Questions 53
Answers 63

DRUG DISTRIBUTION
Questions 67
Answers 87

PRACTICE SETTING
Questions 93
Answers 102

CASE SCENARIOS
Questions 106
Answers 155

ADDITIONAL QUESTIONS
Bonus Questions 171
Answers 189

New Questions
197
Answers
204

Competency #1: Patient Care and Health Promotion

Questions

1. Which of the following blood pressure values is targeted in the treatment


of a diabetic patient with hypertension?

A. Lower than 130/80 mm Hg


B. Lower than 135/85 mm Hg
C. Lower than 140/90 mm Hg
D. Lower than 145/95 mm Hg
2. Which of the following clinical tests is NOT included in the initial
investigation of hypertension?

A. Serum sodium
B. Serum creatinine
C. Serum magnesium
D. Urinalysis

3. All are drugs linked to secondary hyperlipidemia, EXCEPT:

A. Corticosteroids
B. Thiazide diuretics
C. Alpha blockers
D. None of the above

4. Ms. John’s clinical results show the following:

A. Abdominal circumference 83 cm
B. Fasting glucose 5.4 mmol/L
C. HDL 0.9 mmol/L
D. Blood pressure 125/80 mm Hg

Which of the values above would be considered a risk factor for metabolic
syndrome?

5. Which of the following drugs has the highest effect in raising HDL?

A. Ezetimibe
B. Atorvastatin
C. Gemfibrozil
D. Niacin

6. Which of the following patient characteristics is unlikely to increase statin-


related myotoxicity?

A. Untreated hypothyroidism
B. Alcohol abuse
C. Polypharmacy
D. History migraines

7. Which of the following conditions is NOT a contraindication to statin


therapy?

A. Liver disease
B. Alcohol abuse
C. Pregnancy
D. Kidney disease

8. Wrong statement concerning rosuvastatin:

A. Decreased bioavailability with concomitant administration of


magnesium or aluminum hydroxide containing antacids
B. Initial dose is 20 mg/day at anytime, however Asian patients use half
the dose
C. Liver function monitoring is recommended
D. Monitor INR with concomitant use of warfarin

9. Which of the following is NOT an adverse effect of niacin?


A. Hot flushes
B. Reactivation of peptic ulcer
C. Decreased blood glucose
D. Increased uric acid

10. The following drugs can cause dysglycemia, EXCEPT:

A. Abacavir
B. Risperidone
C. Metoprolol
D. Interferon Alpha

11. All are correct statements concerning insulin therapy in type I diabetes,
EXCEPT:

A. Intensive and conventional treatments control blood glucose equally


B. Intensive treatments reduce the risk of diabetes complications better
than conventional treatments
C. Regular insulin should be administered 20 to 30 minutes before meals
D. Annual influenza immunization and one-time pneumococcal
vaccination are recommended during the course of therapy

12. HM is a 35 years old woman has been just diagnosed with type II
diabetes. Her laboratory test shows 10.3% HbA1c (glycosylated
hemoglobin).
Which of the following would be recommended?

A. Initiation of non-pharmacologic therapy such as diet, exercise and self-


monitoring of blood glucose
B. Start monotherapy with an antihyperglycemic drug
C. Start taking a combination of two antihyperglycemic drugs from two
different classes
D. Start taking one antihyperglycemic drug and insulin

13. Control of blood glucose reduces the risk of long-term complications of


diabetes. Which of the following is the recommended target range for 2 hr
postprandial plasma glucose (PPG)?

A. 3 – 5 mmol/L
B. 4 -7 mmol/L
C. 5 – 10 mmol/L
D. 9 – 12 mmol/L

14. The interaction between sulfonylureas and salicylates, sulfonamides or


monoamine oxidase inhibitors leads to:

A. Hypoglycemia
B. Hyponatremia
C. Hypokalemia
D. Ketoacidosis

15. Antiepileptic drugs such as gabapentin are used to treat primarily which
of the following anxiety disorders?

A. Panic disorder
B. Obsessive-compulsive disorder
C. Social anxiety disorder
D. Post-traumatic stress disorder

16. All are adverse effects of gabapentin, EXCEPT:

A. Vision change
B. Dizziness
C. Ataxia
D. Hypotension

17. Which of the following bacterial pathogens is unlikely to be involved in


acute otitis media?

A. Streptococcus pneumonia
B. Staphylococcus aureus
C. Haemophilus influenzae
D. Moraxella catarrhalis

18. Which of the following is unlikely to be a risk factor for antibiotic


resistance that prevents the use of standard dose of amoxicillin in the
treatment of acute otitis media in children?

A. Daycare attendance
B. Early recurrence of otitis media
C. Fever
D. Recent antibiotic therapy

19. Recommended ceftriaxone regimen for a 5 years old with risk factors for
antibiotic resistance.

A. 30 mg/kg/day PO for 3 days


B. 30 mg/kg/day IM for 3 days
C. 30 mg/kg/day PO for 5 days
D. 30 mg/kg/day IM for 5 days

20. All are correct statements concerning the use of nitrofurantoin, a urinary
antiseptic, for the treatment of UTIs, EXCEPT:
A. Less effective than the combo sulfamethoxazole/trimethoprim
B. Adverse effects include pulmonary and hepatic toxicity with long-term
use
C. Contraindicated if clinical test shows creatinine clearance level < 60
ml/min
D. Standard formulation of nitrofurantoin is better tolerated than
macrocrystal formulation

21. Correct duration of treatment of pyelonephritis using nitrofurantoin as


first line therapy.

A. PO for 5 days
B. PO for 7 -10 days
C. PO for 10- 15 days
D. None of the above

22. Risperidone is known to induce sexual dysfunction. Which of the


following second-generation antipsychotics is the least likely to improve this
adverse effect?

A. Quetiapine
B. Paliperidone
C. Olanzapine
D. Ziprasidone

23. Verapamil is a non-dihydropyridine CCB and a substrate for CYP 450


3A4 which explains its various drug-drug interactions. Which of the
following Verapamil interactions should be a primary concern?

A. Cyclosporine
B. Lovastatin
C. Digoxin
D. Quinidine

24. Which of the following therapeutics interact with beta blockers resulting
in excessive cardiodepression?

A. Anesthetics
B. Calcium channel blockers
C. Antiarrhythmics
D. All of the above

25.Which of the following dihydropyridine calcium channel blockers (CCBs)


may worsen heart failure (HF)?

A. Nifedipine
B. Amlodipine
C. Felodipine
D. None of the above

26. Which of the following drugs reduce the risk of death in patients with
systolic heart failure (HF)?

A. Beta blockers
B. Antiarrhythmics
C. Calcium channel blockers
D. Diuretics

27. Which of the following diuretics is linked to ototoxicity at high dose?

A. Hydrochlorothiazide
B. Furosemide
C. Spironolactone
D. Triamterene

28. Which of the following benzodiazepines is preferred for the management


of initial insomnia due to fast onset and short duration of action?

A. Flurazepam
B. Nitrazepam
C. Temazepam
D. Triazolam

29. Folic acid supplements are required for patients taking:

A. Anticonvulsants
B. Glucocorticoids
C. Metformin
D. Methotrexate

30. In general biotechnology drugs such as monoclonal antibodies have a


short shelf life compared to small molecule drugs due to their sensitivity to:

A. Exposure to UV light
B. Exposure to extreme temperatures
C. Handling
D. All of the above

31. During clinical trial, an investigational drug, drug A, is found to be


metabolized to a single metabolite via CYP3A4. This metabolite is found to
be equipotent to Drug A. After administration of a single dose of drug A, the
apparent half-life of both A and its metabolite were found to be 6 hr.
However, when clinical studies were conducted with the metabolite alone, the
pharmacologic effect of the metabolite was found to be of shorter duration
but faster onset of action than that of drug A. A possible explanation is:

A. The elimination rate of the metabolite is actually faster than drug A


B. The absorption rate of the metabolite is faster than that of drug A
C. The metabolite is more potent than drug A
D. The metabolite and drug A have same volume of distribution (Vd)

32. A patient with chronic obstructive pulmonary disease (COPD) is being


treated with theophylline by I.V. infusion. What fraction of the steady-state
level will be achieved after a time equal to two half-lives from the start of the
infusion?

A. 25%
B. 50%
C. 75%
D. 90%

33. When a loading dose is administered, the initial drug concentration in


plasma is primarily dependent on which of the following?

A. Elimination rate constant


B. Half-life
C. Volume of distribution
E. Rate of absorbance

34. 500 mg of a drug is administered to a patient and the drug is later


measured in plasma to be 1 μg/ml. What is the apparent volume of
distribution (Vd)?

A. 0.5 L
B. 5L
C. 50 L
D. 500 L

35. Drug Z is given by constant intravenous infusion to a patient with


ventricular arrhythmias. The elimination half-life for drug Z is 3.5 hours.
When do you expect to achieve 90% of the predicted steady-state level in
plasma?

A. 7.5 hours
B. 11.5 hours
C. 14.5 hours
D. 17.5 hours

36. Drug X is given as a rapid, single i.v. infusion to a 50 kg individual. The


volume of distribution (Vd) for drug X is 2 L/kg. What is the predicted initial
concentration (Co) in plasma if a 600 mg dose is administered?

A. 2 ug/ml
B. 4 ug/ml
C. 6 ug/ml
D. 13 ug/ml

37. A 37-year old female has been scheduled for laparoscopic


cholecystectomy. Her pre-operative blood pressure is normal. After the
administration of midazolam, rocuronium and propofol, she was intubated
and volatile anesthesia started. She suddenly became flushed and tachycardic.
Her airway pressure increased and blood pressure decreased. Anaphylaxis
was suspected. The hospital pharmacist on duty dispensed epinephrine for
immediate administration. She was later referred for allergy testing. Which of
the following statements best describes the symptoms experienced by this
patient?

A. Hypersensitivity Type I
B. Hypersensitivity Type II
C. Hypersensitivity Type III
D. Hypersensitivity Type IV

38. JR is a 55-year old diabetic patient. His blood glucose is under control.
However, as his pharmacist, you are concerned about his elevated blood
pressure. Despite a weight reduction of 11kg and the implementation of
healthy dietary habits, his blood pressure is not within the target range. You
call his physician to express your concerns and recommend prescribing a beta
blocker to help reduce his blood pressure. Which of the following beta
blockers would be preferred for the treatment of hypertension in this patient?

A. Nadolol
B. Atenolol
C. Propranolol
D. Carvedilol

39. A 33-year old male visits your office for the evaluation of progressive
loss of peripheral vision. His intraocular pressure was measured by tonometry
and was found to be elevated. He was diagnosed with open- angle glaucoma.
Which of the following beta blockers would you recommend to help reduce
intraocular pressure associated with open-angle glaucoma?
A. Nadolol
B. Timolol
C. Metoprolol
D. Acebutolol

40. A patient suffering from hypertension and migraine headaches presents to


your pharmacy with a new prescription. His physician has prescribed an
adrenergic drug with dual effect in hypertension and migraine headaches.
Which of the following drugs has been prescribed?

A. Esmolol
B. Metoprolol
C. Propranolol
D. Acebutolol

41. The Pap smear test detects cervical cancer in women at an early stage of
the disease when there are no symptoms and the disease is not evident on
visual examination. Which of the following statements applies to a woman
with no history of cervical cancer who undergoes a Pap smear?

A. She is practicing primordial prevention


B. She is practicing primary prevention
C. She is practicing secondary prevention
D. She is practicing tertiary prevention

42. Which of the following statements concerning the underlying


philosophies of health promotion and health protection is accurate?

A. Health promotion seeks to foster the capacity for self-responsibility


in a community, enabling it to improve its health via collective action
B. Health protection is a set of actions, often supported by legislation,
applied by an external authority to avoid adverse health consequences
in the community
C. The involvement of the community may be limited in health
protection
D. All of the above
43. All of the following are desired elements for a health promotion
strategic plan within a community, EXCEPT:

A. Creating supportive environments within communities


B. Avoiding direct community involvement
C. Creating public policies that promote healthy lifestyle
D. Developing the skills of healthcare professionals

44. Which of the following is a major influence that determines the


successful implementation of a healthcare intervention?

A. Patient’s education, health knowledge, and income


B. Physician’s training and technical expertise
C. Healthcare delivery system factors including costs, risks,
effectiveness and acceptability of the intervention
D. All of the above

45. All of the following are basic values of a health-promoting approach,


EXCEPT:

A.Evidence forms the basis of agreements between programme and policy


decision-makers

B.Individual should be fully responsible for owns health without burdening


the society

C.Interactions between people and their physical and social surroundings


affect health and health behaviours
D.Social justice, equity, mutual respect and caring are necessary for health

46. All of the following are primary preventive interventions recommended


in healthcare practice, EXCEPT:

A. Counselling against the use of tobacco


B. Counselling for increased physical activity
C. Prescribing folic acid to women wishing to become pregnant
D. Prescribing atorvastatin for cholesterol reduction

47. All of the following are elements of patient-centered counselling,


EXCEPT:

A. Helps patients recognize and acknowledge their specific risk behaviors


B. Helps patients decide on a personalized plan of action
C. Promotes the implementation of a personalized plan of action
D. Promotes one-way information transfer from healthcare professional to
patient

48. To ensure effective patient counselling skills, the first step for
pharmacists is to learn how to explore the context of the behavioral risk
factors in a patient’s life and to begin to understand the process of
empowering patients who desire a change. The following are the 5 A’s model
for patient-centered behavioral change, EXCEPT:

A. Address the behavior at risk first then the patient’s agenda for the visit
B. Assess the patient’s knowledge of the risks and his/her readiness to
change
C. Assist the patient’s readiness
D. Arrange follow-up to reinforce the effort and address relapse
49. Which of the following statements best describes the likely impact of
the aging population on the health care system?

A. Cost of acute care


B. Cost of chronic care
C. Cost of health technologies
D. Cost of pharmaceuticals

50. Which of the following statements describes a secondary intervention


targeting a single patient in the prevention of hepatitis C?

A. Counselling on safe drug use to prevent hepatitis C infection


B. Counselling on safer sex habits to prevent hepatitis C infection
C. Screening for hepatitis C infection if the patient has a history of drug
use by injection
D. Recommending hepatitis C infection therapy to an infected patient
Answers

1. A
In general, the target value is 140/90 mm Hg. A applies also to a patient with
chronic kidney disease and hypertension.

2. C
Potassium level, total cholesterol and standard 12-lead ECG analysis are also
performed.

3. D

4. C
HDL < 1.3 mmol/L is a risk factor for women and HDL< 1.0 mmol/L
presents the same risk for men.
5. D
Statins have the highest effect on lowering LDL and decrease cardiovascular
disease morbidity as well.

6. D
Gender is also a factor, women are more affected than men

7. D
However, use with caution in the case of moderate to severe kidney
malfunction

8. B
Initial dose is 10 mg/day anytime and Asian patients get 5 mg/day anytime.

9. C
Increased blood glucose instead. Monitor uric acid, glucose and
transaminases (ALT, AST) (hepatotoxicity). Other adverse effects of niacin
include dry skin and pruritus.

10. A
Abacavir is NRTI, however HIV drugs that are protease inhibitors such as
indinavir, ritonavir, atazanavir and nelfinavir do cause dysglycemia.

11. A
Intensive treatments control blood glucose better than conventional
treatments.

12. C
With HbA1c higher or equal to 9% at presentation drug treatment is initiated
without delay.

13. C
B refers to target value for fasting plasma glucose (FPG).
14. A

15. C

16. D

17. B
Streptococcus pneumoniae is the top pathogen; Streptococcus pyogenes is
also associated with otitis media.

18. C

19. B
Ceftriaxone is given by IV or IM

20. D

21. D
Pyelonephritis is an ascending urinary tract infection that has reached the
pyelum (pelvis) of the kidney. Nitrofurantion is not recommended for the
treatment of pyelonephritis; Fluoroquinolones are used for mild to moderate
cases and aminoglycosides for severe cases.

22. B
Paliperidone is the metabolite of risperidone therefore the worst alternative to
risperidone.

23.C
Verapamil increases digoxin level by 50 to 75% within a week
Other Verapamil interactions include azole antifungals, macrolides,
carbamazepine, protease inhibitors, simvastatin, rifampin, amiodarone, beta
blockers

24. D
Bradycardia with digoxin, amiodarone, diltiazem and verapamil (non-
dihydropyridine CCBs); Hypertension with alpha agonists.
25. A
Among CCBs felodipine and amlodipine are exceptions. Among
antiarrhythmics amiodarone and dofetilide (both are class III) are exceptions.

26. A
Also, ACEIs, ARBs, aldosterone antagonists (spironolactone and eplerenone
which does not induce gynecomastia), combo nitrate/hydralazine. Digoxin
and diuretics do not reduce the risk of mortality.

27. B
Loop diuretics in general: ethacrynic acid, torsemide and bumetanide

28. D
Triazolam has no hangover therefore there is no effect on daytime alertness.

29.B
Anticonvulsants induce folic acid deficiency. Folic acid increases the
effectiveness of antidepressants. Calcium and vitamin D are recommended
for patients taking glucocorticoids to reduce the incidence of osteoporosis.
Refer to Appendix V
30.D

31. A
The two compounds are equipotent.

32. C
50% of steady state is achieved in one half life, and 50% further in another
half life or 75% in 2 half lives, 50+25+12.5= 87.5% in 3 half lives and 90%
in 3.3 half lives.

33. C
The two primary factors affecting the initial concentration are the amount of
drug administered (the loading dose or dose) and the volume of distribution
(Vd). The relevant equation is: plasma concentration = (Loading Dose)/Vd.
34. D
Vd = drug concentration in plasma / dose

35.B
90% of steady state will be reached in 3.3 half-lives or 3.3 x 3.5 hours = 11.5
hours.

36. C
Considering that Co=Dose/Vd, Co=600mg/(50kg x 2L/kg) = 600 mg/100 L
or 6mg/L or 6 ug/ml

37. A
It is a case of immediate reaction.

38. B
Atenolol is a cardioselective (beta 1 antagonist) drug.

39. B
Timolol is a non-selective beta-adrenergic antagonist proven effect in the
treatment of glaucoma.

40. C
Propranolol is a non-selective beta-adrenergic antagonist proven effect in the
prevention of migraines and management of essential tremors and stage
fright.

41. C
Secondary Prevention
A. Primordial prevention acts early in the causal chain, to alter general
social or economic circumstances that give rise to risk factors. The Pap test
does not address general circumstances, but a specific disease process.
B. Primary prevention is defined as the avoidance of disease – having a Pap
smear would not prevent the disease from occurring.
C. Secondary prevention is defined as the interruption of any disease
process before the emergence of recognized symptoms or diagnostic
findings of the disorder. The Pap smear test forms an essential step in this
process: it identifies the disease process before the emergence of symptoms.
D. Tertiary prevention is defined as the avoidance of negative sequelae of a
disease process, once the disease has been diagnosed and treated. The Pap
smear concerns early detection; it has nothing to do with subsequent effects
of the disease.
E. Yes, it is a screening test, but that forms an integral component of an
approach to prevention. The intent in administering the test is to prevent
further progression of the cancer.

42. D

43. B
Strengthening community action is highly desirable in health promotion.

44. D

45. B
Society as whole must take care of all its members. In addition, health care,
health protection and disease prevention complement health promotion

46. D
Primary prevention refers to interventions that are offered before a disease
occurs. Counselling for healthy diet is also a primary intervention.

47. D
An effective patient-centered counselling is a two-way transfer of
information

48. A
Instead, address agenda. Deal first with the patient’s agenda for the visit
(new Rx, refill….), and then introduce the topic of the at-risk behavior; e.g.
I would like to talk to you about...
49. B
An aging population does not increase the cost of individual therapeutics;
however due to polypharmacy (more than 4 prescribed drugs) the overall cost
of drug therapy increases in elderly patients.

50. C
A and B are primary interventions; D is a tertiary intervention. Counselling
an infected patient on hepatitis C transmission prevention strategies is also a
tertiary intervention.

Competency #2: Intra and Inter-Professional Collaboration


Knowledge and Research Application

Questions

1. Which of the following statements described best the process of


pharmaceutical care?
A. The preparation and processing of prescription and non-
prescription medications intended to enable the patient to
achieve defined goals of therapy.
B. The responsible provision of pharmacotherapy for the purpose
of achieving definite outcomes that improve or maintain a
patient’s quality of life.
C. Identifying an undesirable event experienced by a patient that
involves drug therapy and interferes with achieving goals of
therapy.
D. The conscientious, explicit and judicious use of current best
evidence in making decisions about the care of individual
patients.

2. Which of the following statements is correct concerning collaborative


patient-centered health care?

A. Improved access to care.


B. Improved patient safety.
C. Increased efficiency of human resources.
D. All of the above

3. As a community pharmacist which of the following statements describes


your roles in effective health care?

A. Reduction of the frequency of adverse events post-discharge


B. Enhance preventive health care
C. Optimize the use of technology in communication
D. All of the above

4. In pharmacy practice which of the following strategies is NOT


recommended for effective health care professionals’ collaboration with the
goal of optimizing drug therapy?

A. The maintenance of professional autonomy in achieving profession


related outcomes should be a priority
B. It is important to be available to other care providers when they need to
discuss a patient care issue
C. All care providers need to respect a patient’s right to make informed
decisions regarding his or her care
D. Adoption of technology to improve communication between healthcare
professionals

5. When discussing a drug therapy error with the patient’s physician, the
pharmacist is NOT required to include:

A. The implementation of a plan to alleviate the patient’s harm if any


B. The individual responsible for causing the drug therapy problem
C. The potential harm to the patient
D. The follow-up that needs to occur to ensure patient’s safety

6. Which of the following statements are TRUE regarding pharmacists-


physicians relationship?

A. Apologizing is one way to show respect for a physician’s time when


needing to have him or her come to the phone.
B. Be knowledgeable in your field of practice
C. Trust in a professional relationship can take time to develop and is
built on experience
D. Recognize when an issue can be handled by the physician’s staff and
when you really must speak to the physician.
7. Documentation of a drug-related problem by a pharmacist contains all of
the following components EXCEPT:

A. Monitoring plan
B. Patient assessment notes
C. Goals of therapy
D. Recommended interventions

8. All of the following strategies can enhance collaboration between


pharmacists and physicians, EXCEPT:

A. Ensure you speak directly with the physician on all patient-care


matters
B. Determine which method of communication is preferred by
physician’s office
C. Implement a “physician only” phone line
D. Introduce yourself to the physician to establish a face-to-face
relationship

9. A 88-year-old female patient has been experiencing continuous nausea and


her symptoms have not improved in the past 2 months that she has been on
donepezil 5 mg daily. You would:

A. Consult with the physician to add vitamin E to her therapy


B. Consult with the physician to discontinue donepezil therapy
C. Advise her to remain on the same dose for another 2 months then
reassess
D. Advice her to remain on the same dose for another 2 months, then
reassess and recommend dimenhydrinate for nausea

The clinical trial results of an experimental antihypertensive drug are


shown below.
Number of patients Number of enrolled
who experienced patients
orthostatic
hypotension
Experimental 55 3500
antihypertensive
Standard drug 402 3500

10. Based on the results above, what is the control event rate of this study?

A. 3.7%
B. 5.0%
C. 8.8%
D. 11.5%

11. Based on these results, what is the number needed to be treated for this
study?

A. 5
B. 8
C. 10
D. 15

12. Which of the following factors affect the stability of Total Parenteral
Nutrition (TPN)?

A. Concentration of amino acid


B. Concentration of electrolytes
C. Concentration of dextrose
D. All of the above
13. A clinical study is conducted to determine the effectiveness of an
experimental antidepressant. Which of following values will NOT help
evaluate the significance of the observed differences between the study group
and control group?

A. Relative Risk Reduction (RRR)


B. Control Event Rate (CER)
C. Absolute Risk Reduction (ARR)
D. Number Needed to Treat (NNT)

14. In a clinical study, bias is a term that covers any systematic errors that
result from the way the study was designed, executed and interpreted. Which
of the following is a common flaw in clinical studies?

A. Lack of blinding
B. Randomization
C. Patient follow up
D. Intent to treat

15. A nurse requires your assistance in switching a patient from IV


haloperidol to PO. Which of the following is a single dose of haloperidol that
must be provided to the patient twice daily as equivalent of 30 mg IV daily
assuming that the bioavailability of haloperidol is 60%?

A. 10 mg
B. 15 mg
C. 20 mg
D. 25 mg
16. A nurse requires your assistance in switching a patient from PO morphine
to IV. Which of the following is the daily IV dose of morphine that must be
provided to the patient as equivalent of 15 mg bid PO assuming that
morphine has a conversion ratio IV: PO of 1:3?

A. 5 mg
B. 10 mg
C. 15 mg
D. 30 mg

17. A 35-year old patient with history of Atrial Fibrillation visits your
pharmacy to fill a prescription. She is on warfarin and metoprolol for the
management of heart condition. During your interaction, she expresses her
intention to become pregnant. How would you counsel this patient regarding
her anticoagulant therapy?

A. The dose of warfarin should be increased during pregnancy due to


reduction of bioavailability
B. Warfarin should be discontinued immediately, and vitamin k
supplement should be provided to reverse its effect
C. Warfarin should be replaced with heparin
D. Warfarin should be replaced with aspirin at analgesic dose throughout
the pregnancy

18. A 21-year-old female was brought by the paramedics to the emergency


department at 8:27am. She was found in her room by her parents in a state of
altered consciousness and unable to recognize them. Her symptoms include
pinpoint pupils, respiratory depression and slow heart rate. Opioid overdose
is suspected. The physician on duty asks you to recommend an effective
antidote to manage the symptoms of the patient. Which of the following
agents is an effective antidote for the management of opioid overdose?
A. Naltrexone
B. Naloxone
C. Methadone
D. Meperidine

19. A 8-year-old boy who fell off his bike has a laceration on his right knee
that requires sutures. As hospital pharmacist, you recommend a mix of
lidocaine hydrochloride as a 2% solution with epinephrine 1:100,000 for
local infiltration anesthesia. What is the purpose of the inclusion of
epinephrine?

A. To produce vasoconstriction resulting in enhanced effect lidocaine


B. To decrease the sensitivity of C fibers to lidocaine
C. To decrease the sensitivity of sympathetic nerve endings to lidocaine
D. To increase the sensitivity of A fibers to lidocaine

20. A 55-year-old female patient is complaining of pain in her right leg. She
just returned from a trip to Russia. Her flight was more than 10 hours long.
Duplex ultrasonography shows the presence of deep vein thrombosis in the
affected leg. The physician prescribes enoxaparin. Which of the statements
best describes the rational use of enoxaparin in this patient?

A. Enoxaparin can be effectively monitored by measuring the patient’s


INR
B. Enoxaparin can be effectively monitored by measuring the patient’s
aPTT
C. Enoxaparin has an effective antidote called protamine sulfate
D. Enoxaparin therapy does not require monitoring

21. In clinical trials the number need to treat (NNT) refers to the number of
patients that must be treated to benefit one patient. NNT equals:
A. 1/CER
B. 1/EER
C. 1/RRR
D. 1/ARR

22. An investigational new drug is tested in 2500 patients. The control group
has the same number of patients. At the end of the trial, 625 patients have
improved symptoms compared to only 500 in the control group. What is the
NNT of this study?

A. 20
B. 40
C. 60
D. 80

23. Which of the following statements describes sensitivity?

A. Measures the number of people who have the disease and tested negative
B. Measures the number of people who do not have the disease and tested
negative
C. Measures the number of people who have the disease and tested positive
D. Measures the number of people who do not have the disease and tested
positive

24. Correct statements regarding the statistical error type II:

A. Data shows no difference between two experimental treatments but


there is actually a difference
B. Data shows a difference between two experimental treatments but
there is actually no difference
C. p (probability) value < 0.05 is the level of significance
D. type II error is also called false positive or alpha error

25. An investigational drug clinical trial results showed that the symptoms of
patients taking the drug have improved compared to patients taking a placebo
with a p value of -0.9. What best describes these results?

A. The drug is more effective than the placebo


B. The drug is less effective than the placebo
C. There is high incidence of type I error
D. There is high incidence of alpha error

26. Which of the following clinical trials phases is post-marketing


surveillance?

A. Phase 1
B. Phase 2
C. Phase 3
D. Phase 4

27. Which of the following are reasons to perform audits during a clinical
trial?

A. To assure all patient protection measures are followed


B. To ensure protocol adherence
C. To ensure study results are valid
D. All of the above

28. Which of the following statements best describes the primary benefit of
electronic data collection systems over paper-based systems during a clinical
trial?

A. Maintain confidentiality of medical records


B. Improve data review and analysis
C. Ability to insure data integrity
D. Insure that all required items are collected

29. Which of the following statements best describes the purpose of the
informed consent review by the Research Ethics Board (REB)?

A. To protect the interests of the hospital conducting the clinical trial


B. To provide the hospital conducting the clinical trial with information
about proposed protocol
C. To protect the subjects enrolled in the clinical trial
D. To protect the interests of the pharmaceutical company sponsoring the
clinical trial

30. Which of the following is a required element of an informed consent?

A. A statement that participation is required after consent is signed


B. Full disclosure of all potential adverse effects
C. Guarantee of clinical improvement in enrolled subjects
D. A statement that enrolled subjects cannot withdraw until full
completion of the trial
31. Which of the following characterizes phase III drug testing?

A. It usually involves a comparison between different treatments


B. It usually requires small groups of patients
C. It can be done without approval from the Research Ethics Board (REB)
D. It always demonstrates superiority of the new treatment

32. Meta analysis is:

A. Combination of several studies that address related research


hypotheses
B. Assessment of the similarity between means of more than two groups
C. Assessment of the similarity between two means
D. Evaluation of differences between two variances

Answers
1. B
A, C and D are components of effective pharmaceutical care.

2. D

3. D

4. A

5. B

6. A

7. B

8. A
Some patient care matters can be effectively handled by the physician staff.

9. D
She is taking the minimum initial dose and it takes at least 4 months to make
a clear assessment of effectiveness.

10.D
Control Event Rate (CER) = 402/3500 = 11.5%
Experimental Event Rate (EER) = 55/3500 = 1.6%
Absolute Risk Reduction (ARR) = CER – EER
Relative Risk Reduction (RRR) = ARR / CER

11. C
Number Needed to be Treated (NNT) = 1/ARR
ARR = CER – EER = 11.5% - 1.6% = 9.9% = 0.099
NNT = 1/0.099 = 10

12. D
The pH and the order for mixing the ingredients also affect the stability of
TPNs.

13. B
CER and EER used individually help evaluate the results within either the
control group (CER) or the experimental group (EER).

14. A
Blinding, patients’ follow-up and randomization reduce the incidence of trial
errors. Intent to treat analysis enhances the accuracy of the results.

15. D
Considering that the daily dose IV is 30 mg, the daily dose PO will be: 30 mg
x 100% / 60% = 50 mg. Therefore, the single dose is: 50 mg / 2 = 25 mg

16. B
The daily dose PO is 15 mg x 2 = 30 mg. Assuming a conversion ratio IV:
PO of 1:3, 30 mg must be divided by 3 = 10 mg.

17. C
Warfarin is teratogenic.

18. B
Naloxone is a potent opioid receptors antagonist.

19. A

20. D

21. D

22. A
NNT = 1/ARR and ARR = EER – CER
ARR = 625/2500 – 500/2500 = 0.05 therefore NNT = 1/0.05 = 20
23. C
B refers to the definition of selectivity

24. A
B refers to statistical error type I also called false positive or alpha error.
Type II error is also called false negative or beta error; p value equal or lower
than 0.05 means that type I error is unlikely. p value > 0.05 is the level of
significance.

25. A

26. D
Phases 1, 2 and 3 are performed pre-marketing to test the safety of the drug
and to identify the required therapeutic dosage. Phase 4 or post-marketing
surveillance refers to long term assessment of drug safety following approval.

27. D

28. C

29. C

30. B

31. A

32. A
Competency #3: Ethical, Legal and Professional
Responsibilities

Questions

1. Which of the following products fall under the class of Natural Health
Products (NHPs) according to Health Canada’s regulations?
A. Arginine
B. Probiotics
C. Gingko
D. All of the above

2. Which of the following statements concerning methylphenidate regulations


in NOT accurate?

A. Sales records are required and kept for at least two years
B. Controlled drug part 1
C. Written prescription only
D. All of the above

3. The transmission of a prescription by fax follows all of the following


guidelines, EXCEPT:

A. The prescription must be sent only to the pharmacy of the patient’s


choice without the intervention of another person having access to it
B. The pharmacist is responsible for verifying the origin and authenticity
of the prescription
C. A faxed prescription must be accepted only within the same province
D. A signed certificate by the prescriber must be include to a prescription
transmitted by fax
4. A pharmacy is planning a temporary closure for 7 days due to strike. All of
the following guidelines should be fulfilled, EXCEPT:

A. The pharmacy manager must obtain the approval of the pharmacy


regulatory authority 30 days prior to closure
B. All prepared prescription recipients must be notified to give them the
opportunity to obtain their prepared prescriptions prior to closure
C. A notice of closure must be posted at the entrance and a telephone
answering machine message of the closure including the duration, the
location of the closest pharmacy and any other useful information
D. The public must be notified using in store postings and media
announcement at least 20 days prior to closure

5. A pharmacist may adjust the quantity of drug to be dispensed if:

A. The patient requests a smaller amount


B. The amount prescribed exceeds the amount covered by the patient’s
drug plan
C. The manufacturer’s unit-of-use standard of package does not match the
prescribed amount
D. All of the above

6. All drugs must be dispensed in a container that is certified as child-


resistant unless:

A. The patient decides otherwise


B. The pharmacist determines that it is not convenient for the patient
C. The child-resistant container does not suit the physical form of the
drug
D. All of the above
7. A prescription older than one year must not be dispensed with the
exception of:

A. Interferons
B. Oral contraceptives
C. Antibiotics
D. Narcotics

8. In a hospital setting unused dispensed drugs must be returned to the


hospital pharmacy. The drug can be re-dispensed if the following conditions
have been met, EXCEPT:

A. Reconstituted solution
B. It is returned in a sealed dosage unit
C. The label is intact and includes the drug lot number and expiry date
D. The integrity of the drug can be verified

9. BX has collapsed on the sidewalk outside your pharmacy. He is


unconscious and his breathing is shallow. A paramedic comes in with BX’s
wallet containing his identification and health card and asks you to check
BX’s patient profile because the hospital emergency department is unable to
respond immediately to his request for information. Which of the following
options is your BEST course of action?

A. Check his profile, provide the information and document the request
B. Obtain authorization to release the information from provincial
authority
C. Refuse the request because you need to protect BX’s right to
confidentiality; inform the paramedic that you could provide the
information to him if he were to obtain a court order
D. Refuse the request because you need to protect BX’s right of
confidentiality; inform the paramedic that you can only provide this
information to a doctor or emergency department staff

10. Sales reports are not required for:

A. Tylenol #4
B. Nabilone
C. Propoxyphene
D. Hydromorphone

11. The patient information material supplied by the manufacturer must be


dispensed with:

A. Methotrexate
B. Ticlopidine
C. Isotretinoin
D. All of the above

12. You are the pharmacy manager in a community pharmacy. Which of the
following will be an efficient strategy to integrate evidence-based medicine
in your practice?

A. Subscribe to all core medical journals.


B. Be familiar with clinical guidelines for common health
conditions seen in his or her practice.
C. Attend as many live education events as possible.
D. Maintain a current library of textbooks in the pharmacy.

13. Which of the following substances is NOT regulated as precursor for


illicit drug under the Precursor Control Regulations of the Controlled Drugs
and Substances Act?
A. Ergotamine
B. Ephedra
C. Epinephrine
D. Ephedrine

14. The following drug product characteristics are uniquely identified by the
Drug Identification Number (DIN), EXCEPT:

A. Manufacturer
B. Rate of clearance
C. Route of administration
D. Formulation

15. The majority of Canadian hospitals are operated by:

A. Community boards of trustees


B. Voluntary organizations
C. Provincial or territorial health authorities
D. All of the above

Answers

1. D
Under the natural health products regulations, NHPs include enzymes, amino
acids, essential fatty acids, vitamins, minerals, herbal remedies, probiotics
and traditional medicines such as traditional Chinese medicines.
Regulations are designed to ensure that NHPs are safe, effective and of high
quality before being approved. Once approved, each product is assigned an
eight-digit product license number, preceded by the letters “NPN.” Labels on
homeopathic medicines bear the designation DIN-HM.

2. C

3. C
A faxed prescription can be accepted from a practitioner registered to practice
in any Canadian province. In addition, to ensure confidentiality, the
equipment for the receipt of a faxed prescription must be in a secured area

4. D
The public must be notified using in store postings and media announcement
at least 30 days prior to closure. Also, arrangements must be made for
emergency access to patient’s records

5. D

6. D
Another situation is when the container is specially designed to enhance
compliance.

7. B
Oral contraceptives may be dispensed for up to 2 years.
8. A

9. A

10. C
Sales reports are not required for propoxyphene and dextroproxyphene.

11. D
Other drugs that require mandatory patient inserts are biologics, NSAIDs,
inhalers and transdermal patches.

12. B
13. C
Other controlled precursors under the act are: Norephedrine, pseudoephedrine
and ergometrine.

14. B
Other characteristics identified are product name, active ingredient(s) and
strength(s) of active ingredient(s).

15. D
Competency #4: Quality and Safety

Questions

1. Which of the following statements regarding non-adherence with


antipsychotics is NOT accurate?

A. Patients who are non-adherent are at an increased risk of


hospitalization
B. Risk factors for non-adherence include adverse reactions and
stigma attached to mental illness
C. Substance abuse is a risk factor for non-adherence
D. The risk of relapse is approximately 7.8 times as high in non-
adherent patients

2. In order to reduce the rate of non-adherence long-acting risperidone may


be a good option in some patients. How would you initiate long-acting
risperidone injection therapy in a patient using oral risperidone?

A. Stop oral risperidone and start long-acting risperidone on the


same day.
B. Taper oral risperidone until he is off, then start risperidone long-
acting injection.
C. Continue on oral risperidone for 3 weeks, start long-acting
injection (given q 2 weeks), and quickly increase the dosage
until the level is therapeutic.
D. Continue on oral risperidone for 3 weeks, start long-acting
injection (given q 2 weeks), and increase the dosage no sooner
than every 4 weeks.

3. The following statements explain the relevance of Evidence-Based


Medicine (EBM) principles in pharmacy practice, EXCEPT:

A. They emphasis on the outcomes of examining evidence from clinical


research
B. They shift decision-making away from intuition
C. They reduce the need to explore patient values and health goals.
D. They help integrate health research findings into healthcare practice

4. Ms. PR is a 17-year-old who is complaining about skin rash. She wonders


if she is having another flare-up of her eczema. She has a history of penicillin
allergy, eczema and seizures. Ms. PR is currently using betamethasone Cr
0.05% bid prn, ethinyl estradiol/levonorgestrel for 12 months and phenytoin
200mg daily for 3 weeks. What is the likely explanation for her rash?

A. An acute flare up of her eczema


B. A reaction to phenytoin
C. An interaction between ethinyl estradiol/levonorgestrel and phenytoin
D. Excessive sun exposure

5. All of the following medications may require dose adjustment if being


taken by a patient just starting on levothyroxine for hypothyroidism,
EXCEPT:

A. Cholestyramine
B. Digoxin
C. Insulin
D. None of the above

6. Initial daily (nighttime) dosage of triazolam in the treatment of insomnia.

A. 0.125 mg
B. 0.5 mg
C. 0.85 mg
D. 1.5 mg

7. Correct statement concerning pneumococcal vaccine:

A. Persons vaccinated prior to age 65 should be vaccinated at age 65 if


five or more years have passed since the first dose
B. The pneumococcal vaccine is given as one dose for most people at risk
any age
C. Side effects include soreness and/or redness at the site of the injection,
fever, rash, and allergic reactions
D. All of the above

8. Which one of the following is the drug of choice for the treatment of
Chlamydia urethritis during pregnancy?

A. Amoxicillin
B. Penicillin V
C. Erythromycin
D. Doxycycline

9. The following symptoms are associated with of UTI caused by Chlamydia


trachomatis, EXCEPT:

A. White, clumpy and curdy discharge


B. Pruritus and odor
C. Inflammation
D. Burning on urination

10. Standard dosage of amoxicillin for the treatment of UTIs.

A. 250 mg BID po
B. 250 mg TID po
C. 500 mg BID po
D. 500 mg TID po

11. A pregnant woman is about to travel to an area where chloroquine-


resistant P. falciparum is malaria exists. Which of the following drugs would
you recommend for her prophylactic treatment?

A. Pramaquine
B. Doxycycline
C. Malarone (atovaquone/proguanil)
D. Mefloquine

12. All are correct statements concerning the use of methylphenidate for the
treatment of ADHD in children, EXCEPT:

A. Use long-acting formulations to decrease abuse potential


B. Use high-dose formulations to reduce the risk of rebound hyperactivity
C. Monitor growth by measuring weight and height
D. Doses greater than 60 mg/day usually do not result in increased
efficacy
13. Which of the following medications may be used for the treatment of
allergic conjunctivitis to reduce inflammation?

A. Ketotifen drops
B. Polysorbate drops
C. Flubiproben drops
D. Naphazoline drops

14. The concentration of carbamazepine in a patient’s blood is being


monitored to determine the dose required to reach optimal therapeutic range.
The initial concentration of carbamazepine has been just doubled to improve
effectiveness. All of the following statements describe the effect of doubling
the dose of carbamazepine on the laboratory results, EXCEPT:

A. The UV-Vis absorbance will double


B. The HPLC peak area will double
C. The GC peak retention time will double
D. The TLC spot will double in size

15. In the process of the evaluation of clinical studies which of following is


an example of primary source of information?

A. Investigator’s research diary


B. Review articles
C. Meta-analysis
D. Textbook
16. The Professional Service Area is an important area in all licensed
pharmacies. Which of the following statements is NOT an accurate
description of the Professional Service Area?

A. The area where counseling is provided privately


B. The area under the audio control in the pharmacy
C. The area under the visual control in the pharmacy
D. The area which includes the dispensary and all shelves, displays or
fixtures bearing drugs

17. In Canada healthcare services are provided under which of the following
insurance plans:

A. Private
B. Federal
C. Provincial and Territorial
D. All of the above

18. Which of the following opioids combinations has the highest incidence of
addiction?

A. Codeine + Methadone
B. Codeine + Meperidine
C. Codeine + Propoxyphene
D. Codeine + Oxymorphone

19. A patient treated for renal insufficiency requires an opioid for the
treatment of chronic pain. Which of the following opioids will be the fastest
acting and the safest in this patient?

A. Meperidine
B. Morphine
C. Fentanyl
D. Oxycodone

20. Hepatic encephalopathy is the occurrence of confusion, impaired


consciousness and coma as a result of liver failure. It is due to the
accumulation in the body of toxic substances that are normally removed by
the liver. Hepatic encephalopathy is monitored by relying on:

A. Levels of ammonia in blood


B. Creatinine clearance
C. Body weight
D. All of the above

21. Which of the following antihypertensive classes has the highest incidence
of sexual dysfunction?

A. ACE inhibitors
B. Calcium channel blockers
C. Alpha blockers
D. Thiazide diuretics

22. Which of the following drugs interact with high fiber food products?

A. Atorvastatin
B. Digoxin
C. Metformin
D. All of the above
23. Nitrofurantoin is an antibiotic used to treat urinary tract infections. It
works by damaging bacterial DNA. The reduction of nitrofurantoin by
bacterial nitrofuran reductase leads to multiple reactive intermediates that
damage ribosomal proteins, DNA and other biomolecules. Nitrofurantoin is
classified:

A. Pregnancy category A
B. Pregnancy category B
C. Pregnancy category C
D. Pregnancy category D

24. A 26-year-old female with acquired immunodeficiency syndrome (AIDS)


develops cryptoccocal meningitis. She refuses all intravenous medications.
Which of the following antifungal agents can be given orally to treat this
patient?
A. Ketoconazole
B. Amphotericin B
C. Fluconazole
D. Nystatin

25. Empiric therapy can be started before final identification of an organism


if further delay can have significant adverse consequences. Which of the
following factors is the least likely to be considered when deciding on an
antibiotic for empiric therapy?

A. Patient’s history
B. Gram staining
C. Patient’s immune status
D. Cell culture
Answers

1. D
The risk of relapse is 3.7 times higher. Non-adherence with antipsychotics is
approximately 50%

2. A
Abrupt switch is more appropriate for long-acting formulations.

3. C

4. B
Phenytoin is also linked to increased liver enzymes, blood dyscrasias (blood
disorders) and coarse facial appearance with long-term use.

5. A
Levothyroxine dose adjustment is recommended every 4 to 6 weeks during
the course of therapy. Digoxin is adjusted to decrease the risk of
hyperthyroidism-like symptoms. Levothyroxine decreases glycemic control
therefore insulin dose adjustment is required. Cholestyramine decreases the
absorption of levothyroxine; dose adjustment is not required however
separate administration by at least 6 hours is recommended.

6. A
0.25 mg is the maximum dosage

7. D

8. A

9. A

10. D

11. D
12. B
Long-acting formulations reduce the risk of rebound hyperactivity.

13. A
Sodium cromoglycate, nedocromil may be used as well; both are mast cells
stabilizers. Polysorbate is used for dry eyes. Flurbiprofen is an NSAID used
primarily for ocular abrasion to reduce pain, swelling and redness.
Naphazoline is a vasoconstrictor use to reduce eye redness.

14. C
In chromatography, the retention time is determined by the physicochemical
properties of the drug therefore it is commonly used for drug identification
not quantification.

15. A
A is a primary source of information; primary sources are original materials
on which other research is based (interviews, patents, survey, speeches,
original articles, e-mails…). Secondary sources are interpretations and
evaluations of primary sources. Secondary sources are not evidence, but
rather commentary on and discussion of evidence (monographs,
commentaries….). A textbook is a tertiary source of information.

16. D
The professional product area sale is the dispensary area. The Professional
Service Area means the area in each licensed pharmacy which includes the
dispensary and all shelves, displays or fixtures bearing drugs and other items
as permitted and which shelves, displays or fixtures are in an area in the
vicinity of the dispensary so that they are under the audio and visual control
of the pharmacist.

17. D
Typical private insurance plans cover prescription drugs, dental and
ophthalmic services.

18. D
Here is the ranking of opioids from low incidence of addiction to the highest:
Low: Codeine, methadone, propoxyphene
Moderate: Meperidine, oxycodone, morphine, levorphanol, hydrocodone,
hydromorphone
High: Oxymorphone

19. C
Fentanyl and methadone are the safest. Use with caution hydromorphone,
oxycodone and oxymorphone. Meperidine, codeine, morphine and
propoxyphene are not recommended.

20. D
Benzodiazepines, narcotics, antipsychotics and alcohol intoxication increase
the risk of hepatic encephalopathy.

21. D
Thiazide diuretics and beta blockers have the highest incidence of sexual
dysfunction.

22. D Refer to appendix L.

23. B

24. C

25. D
Competency #5: Communication and Education

Questions

1. You are counseling a schizophrenic patient who has been non-adherent to


his new therapeutic regimen. Which of the following strategies would be the
least appropriate?

A. Explain to the patient that even though his symptoms may seem
better, his condition, like other medical conditions, requires long
term therapy
B. Explain that non-adherence has been linked to increased rate of
hospital stay
C. Check for the reason for non-adherence, and if it is an adverse
effect, explain that it is possible it may go away, or we can work
together to find another treatment option
D. Tell the patient that he has no choice and he simply have to take
the medication, or he won’t get any better

2. A 90-year-old patient wishes to purchase the following five products. His


profile shows no known allergies and he is currently using warfarin 2.5 mg
daily for 3 years and donepezil 5 mg daily for 1 month.
Which would you NOT sell based on the patient drug regimen information?

A. Ginkgo biloba
B. Evening primrose oil
C. Vitamin B complex
D. Aloe Vera

3. A patient asks you what she could do to decrease her risk of osteoporosis.
You would suggest all of the following options, EXCEPT:

A. Elemental calcium 1000 - 1500 mg and at least Vitamin D 400 IU


daily
B. Regular swimming
C. Regular weight lifting
D. Drinking milk
4. A 28 years old female with no known allergies is complaining about acne.
Medications: Minocycline 100 mg daily for 2 weeks
Benzoyl peroxide/erythromycin Gel Apply qhs for 2 weeks
She has not seen any improvement in her acne despite the treatment.
What would you recommend?

A. Continue minocycline and benzoyl peroxide/erythromycin for 2


months
B. Start tazarotene, discontinue other medications
C. Start isotretinoin, discontinue other medications
D. Start norgestimate/ethinyl estradiol, discontinue other medications

5. You are counseling a patient who is complaining about recent incidence of


nosebleeds. Which of the following strategies would you include to your
counseling session?

A. Humidify your home


B. Maintain the temperature between 15oC and 18oC in sleeping areas
C. Limit the use of aspirin
D. All of the above

6. A shock is a life-threatening condition characterized by all of the


following, EXCEPT:

A. Cool, pale and clammy skin


B. High blood pressure
C. Fast and shallow breathing
D. Thirst, nausea and vomiting
7. You are counseling a patient who is complaining about recurrent urinary
tract infections (UTIs). Which of the following preventive strategies would
NOT be included to your counseling session?

A. Drink at least two liters of water daily


B. Drink cranberry or blueberry juice
C. Use vaginal deodorants to mask odors
D. Wipe yourself from front to back

8. A cataract is a painless and cloudy area in the lens resulting in the


reduction of light reaching the retina. All of the following symptoms are
associated with cataracts, EXCEPT:

A. Cloudy, fuzzy and filmy vision


B. Increased ocular pressure
C. Decreased night vision
D. Spots or halos around light

9. Swimmer’s ear or otitis externa is an inflammation of the ear canal. Which


of the following are causes of Swimmer’s ear?

A. Psoriasis
B. Prolonged use of earplugs
C. Water in ear
D. All of the above

10. Migraine headaches are unilateral headaches characterized by nausea,


vomiting and sensitivity to light or sound. Which of the following is NOT a
recommended non-pharmacologic strategy for the prevention of migraine
headaches?
A. Practice of relaxation techniques
B. Taking a nap
C. Application of a warm compress on painful area
D. Avoidance of known triggers

11. Canker sores are painful open sores in the mouth. They are caused by
infections, certain foods, stress and female hormones. Canker sores can be
treated by:

A. Rinsing the mouth with an antacid


B. Rinsing the mouth with diluted hydrogen peroxide
C. Applying a paste made of baking soda and water
D. All of the above

12. Lice are tiny white insects that can live on skin, hair or clothing. Lice
spread by close physical contact or contact with clothing and bedding of an
infected person. All of the following strategies help get rid of lice from
clothing and bedding, EXCEPT:

A. Freezing for 6 hours


B. Dry cleaning
C. Washing with hot water
D. Seal in plastic bags for 2 weeks

13. Diabetic peripheral neuropathy (DPN) is a collection of nerve disorders


affecting diabetes patients. All of the following statements regarding DPN are
correct, EXCEPT:

A. Up to 50% of patients affected by diabetes develop DPN within ten


years of onset of the condition
B. DPN is characterized by progressive and painless loss of sensation in
hands and feet
C. DPN is the primary contributing factor for all foot ulcers
D. DPN affects specifically patients affected by type II diabetes

14. Pain associated with diabetic peripheral neuropathy (DPN) is treated


with:

A. Tricyclic antidepressants
B. Anticonvulsants
C. Opioids
D. All of the above

15. Peripheral vascular disease (PVD) is another complication of diabetes


which contributes to the development of foot ulcers. PVD is characterized by
the blockage of arteries resulting in poor blood flow and nutrients intake, and
ultimately impaired wound healing ability. Poor blood flow leads to all,
EXCEPT:

A. Pulses in the feet


B. Bonion
C. Skin discoloration
D. Swelling of feet

16. JP is a 7 years old female patient without a history of allergies. Her


weight is 27 kg. She is currently receiving gentamicin ear drops 3 drops in
the left ear TID - initiated 3 days ago. After 3 days of gentamicin ear drops,
JP’s mother presents a new prescription for her child for amoxicillin
suspension 250mg TID x 5 days. She states that JP’s ear infection has
worsened, she is even suspected to have a hole in the eardrum. She asks if
she should continue to give gentamicin ear drops to JP. Which of the
following would be your recommendation?

A. Continue gentamicin ear drops along with amoxicillin suspension


B. Discontinue gentamicin ear drops and start amoxicillin suspension
C. Call JP’s physician to decrease the dose of gentamicin ear drops to 2
drops TID
D. Call JP’s physician to change amoxicillin ear drops to
ciprofloxacin/hydrocortisone otic 2 drops TID

17. KG is a 55 years old female patient with no known allergies who has
been diagnosed with migraine headaches. Her medications include:
ASA/butalbital/caffeine/codeine (Fiorinal C ½) 1 q3-4h prn - filled 1 week
ago
Ergotamine/caffeine 2 stat then 1 q30min up to 6 per day
A family member presents the following prescription for KG who is at home
with an acute migraine: Sumatriptan 100mg Sig: 1 stat for migraine, to
maximum of 1 per attack or 2 per day. Which of the following would be your
recommendation based on the patient’s profile?

A. Fill the prescription and caution the patient not to start taking
Sumatriptan within 24 hours of taking ergotamine/caffeine
B. Fill the prescription and caution the patient not to start taking
Sumatriptan within 24 hours of taking Fiorinal C 1/2
C. Fill the prescription and counsel the patient about appropriate dosing of
Sumatriptan
D. Call the physician about a possible drug interaction between
Sumatriptan and Fiorinal C1/2

18. JC is a 30 years old male patient with known penicillin and aspirin
allergies.
JC is visiting the pharmacy to seek your counseling on how to prevent or
treat diarrhea while in Mexico. Your counseling will include all of the
following, EXCEPT:

A. Drink only bottled water/soft drinks without ice


B. Eat food only in reliable restaurants. Eat only fruits cut and peeled by
yourself
C. Take bismuth subsalicylate, 2 tablets every 30 minutes (max 8 doses
per day)
D. Take loperamide 2mg, 2 tablets at onset of diarrhea, then 1 tablet after
each loose bowel movement (maximum 8 tablets per day)
E. See your physician to get a prescription for ciprofloxacin;
ciprofloxacin will be useful in case of severe diarrhea

19. PS is a 55 years old male patient with known sulfonamides allergy,


however he is not allergic to glyburide. PS has type 2 diabetes and is
currently taking glyburide 10mg daily – initiated 3 years ago. PS comes to
your pharmacy with a new prescription for atenolol 50 mg daily to manage
his recently diagnosed high blood pressure. Which of the following would be
your concern regarding atenolol?

A. May mask signs of hypoglycemia


B. Increases blood sugar
C. Increases glyburide level
D. Decreases glyburide effectiveness

20. Which of the following counseling points are appropriate when


counseling a patient on emergency contraceptive pills (ECP)?

A. Take dimenhydrinate 50mg 1 hour before ECP to decrease nausea and


vomiting
B. Taking ECP can result in transient side effects such as abdominal
cramps, headaches, and breast tenderness
C. If you do not experience bleeding within 3 weeks after taking ECP,
you should see your family doctor
D. All of the above

21. EM is a 59 years old male patient with a history of sulfonamides, molds


and pollen allergies. His medical history includes: arthritis x 2 years and
asthma x 1 year. EM is currently on: Salbutamol 100 mcg HFA inhaler 2
puffs QID prn -initiated 1 year ago, Fluticasone 125mcg HFA inhaler 2 puffs
BID -initiated 1 year ago and Naproxen 500mg BID - initiated 2 years ago.
EM presents a prescription for celecoxib 100mg BID x 60 capsules. His
doctor wants to stop naproxen because he has been experiencing stomach
upset.
Which of the following would be your primary concern regarding
celecoxib?

A. Celecoxib not been proven to be as effective as naproxen in arthritis


B. Celecoxib is costly. Adding misoprostol to naproxen is cheaper than
switching to celecoxib.
C. Celecoxib is contraindicated in patients with asthma
D. Celecoxib is contraindicated in patients with sulfonamide allergy

22. DC is a 46 years old male patient with known penicillin allergy. His
medical history shows: hypertension diagnosed 16 months ago. He is
currently on hydrochlorothiazide 12.5 mg qam x 16 months and 1
multivitamin one daily x 10 months. While receiving his medications DC
asks if you can recommend another type of eye drops for his itchy and watery
eyes. He has used tetrahydrozoline for 5 days and it is not working.
Your recommendation would be:

A. Naphazoline 2 drops each eye QID


B. Naphazoline/antazoline 2 drops each eye QID
C. Sodium cromoglycate 2 drops each eye QID
D. Tetrahydrozoline/zinc sulphate 1-2 drops each eye TID-QID any of the
above

23. JS is a 67 years old female patient who is allergic to diclofenac. Her


medical history includes insomnia and neuropathic pain. She is currently
taking tryptophan 500mg 2 qhs for sleep x 6 months and amitriptyline 150
mg with mg 100 mg at bedtime x 3 days. JS’s daughter is visiting the
pharmacy to ask you to recommend something for her mother’s flu. She
explains that her mother is extremely agitated and confused; she is also
sweating and has a high fever. She’s in bed with muscle spasms and tremors.
What would you recommend?

A. Discontinue amitriptyline and see her doctor within 48 hours


B. Methocarbamol/ASA 2 QID with food and see her doctor if symptoms
don’t improve in 48 hours
C. Methocarbamol/acetaminophen 2 tablets QID and see her doctor if
symptoms don’t improve in 48 hours
D. Take JS to the closest emergency department as soon as possible

24. SE is a 55 years old male patient with known penicillin and aspiring
allergies. His medical history shows: arrhythmia, hypertension and
hypercholesterolemia. His current medications are: digoxin 0.125mg daily x
1 year, nitroglycerin 0.4mg patch-apply at 8 am, remove at 8 pm daily x 1
year, ramipril 5mg daily x 1 year, furosemide 40mg daily x 1 year,
simvastatin 20mg daily x 1 year. You receive the following prescription for
SE for community acquired pneumonia: amoxicillin 500mg TID x 10 days
When calling the doctor to change the prescription, which alternate antibiotic
would you recommend?

A. Amoxicillin/clavulanate 500mg TID


B. Cefixime 400mg daily
C. Clarithromycin 500mg BID
D. Doxycycline 100mg BID

25. MN is an 8 years old patient. His weight is 50 kg and allergic to captopril,


codeine, metoclopramide and penicillin. His medical history shows:
multicystic kidneys; paraplegic. His creatinine clearance CrCl is 110mL/min
(Normal CrCl = 125mL/min). He is currently on
sulfamethoxazole/trimethoprim susp 5mL daily for UTI Prophylaxis x 3
months, salbutamol 1mg/mL 2.5mL QID prn x 3 years, budesonide 250 mcg
BID x 3 yrs
MN has been coughing up yellowish sputum. The doctor calls you with a
prescription for amoxicillin/clavulanate 250mg TID x 10 days.
What would you do?

A. Fill the prescription as written


B. Suggest reducing the dose to amoxicillin/clavulanate 125mg TID x 10
days
C. Suggest changing to doxycycline 100mg daily x 10 days
D. Suggest changing to erythromycin 250mg QID x 10 days

26. DA is a 60 years old male patient with no known allergies. His medical
history includes type 2 diabetes, hypercholesterolemia. He is physically
active and drinks grapefruit juice each morning. He is currently on
atorvastatin 20mg qhs - initiated 3 years ago, metformin 500mg TID -
initiated 3 years ago, sildenafil 100mg hs prn - initiated 5 months ago,
nitroglycerin 0.4mg Spray 1-2 sprays sl prn - initiated 5 weeks ago.
DA requests a refill of his sildenafil prescription. You would be concerned
about all of the following EXCEPT:

A. Atorvastatin with grapefruit


B. Metformin with atorvastatin
C. Sildenafil with nitroglycerin
D. Sildenafil with grapefruit
27. JC is a 54 years old male patient with no known allergies. His medical
history shows: bone cancer, severe pain. His currently on: hydromorphone
2mg TID x 4 months,Tylenol No. 3, 2 QID prn pain x 5 months

In the past JC had trouble paying for his medications. His condition is
deteriorating, and he is in intense pain. Today JC presents a new prescription
for hydromorphone 6mg BID x 100 capsules. JC also shows the pharmacy
intern a vial of medication, labeled for a deceased patient, which he received
from the doctor. He wants to know if it is the same as what was prescribed
for him. The pharmacy intern confirms that the vial contains hydromorphone
3mg capsules. The pharmacy intern tells JC it is the same medication the
doctor prescribed but to take 2 of the capsules twice daily to get the right
dose.

Which of the following important factors could be less relevant in this case?

A. The other patient’s right to confidentiality has been breached


B. Dosing information on the vial may be inaccurate for JC
C. The integrity of the medication in terms of storage, expiry or tampering
is unknown
D. JC’s financial situation could compromise his treatment

28. A 31-year-old woman presented to the hospital with fever, productive


cough, and progressive shortness of breath of 3 weeks duration. She was a
previous intravenous drug user. She was recently treated for community-
acquired pneumonia (CAP) with cefotaxime and doxycycline. Unfortunately,
progressive dyspnea with respiratory distress developed. Following further
investigations, she was diagnosed with AIDS due to the presence of serum
HIV-1 antibody, a plasma HIV-1 viral load of 12 700 RNA copies/ml, and a
low CD4 count of 13 cells/μl. A highly active antiretroviral therapy
(HAART) is initiated with zidovudine-lamivudine-indinavir (ZDV-3TC-
IDV). During a follow up visit to the hospital pharmacy, she mentions to you
that she is experiencing some symptoms of depression. She adds that her
friend has recommended an herbal product called St. John’s wort and she is
currently using it. Based on your knowledge of the interaction between St.
John’s wort and indinavir, which of the following is the best intervention?

A. Decrease the dose of indinavir


B. Reduce the frequency of St. John’s wort intake
C. No problem. Her therapeutic regimen includes two additional drugs
D. Stop St. John’s wort and switch to another antidepressant

29. A 38-year-old female social worker with severe nicotine dependence


visits your pharmacy. She used to smoke 28 cigarettes per day. Some mild
depression assessed by the Self-rating Depression Scale (SDS) was also
documented in her profile. She was treated for nicotine dependence four
years ago and again two years ago. On each occasion, she was prescribed
nicotine patches. She was offered smoking cessation counseling throughout
the program. Her last relapse occurred two years ago. She wants to know if
there is any “pill” she could try. Which of the following drugs could be
recommended to her?

A. Mirtazapine
B. Citalopram
C. Fluoxetine
D. Bupropion

30. A 27 years old patient visits your pharmacy to seek counseling on


combined oral contraceptive (COC). She has decided to postpone starting a
family due to a hectic professional career. During your interaction she
mentioned that she has been on sumatriptan nasal spray prn for almost a year
for the treatment of migraines. Which of the following contraceptive would
you recommend to this patient?

A. Combined estrogen/progestin monophasic


B. Combined estrogen/progestin biphasic
C. Combined estrogen/progestin triphasic
D. Progestin only
Answers

1.D
You should also empathize with the patient regarding the difficulty of
schizophrenia management, show genuine concern, and work with him to
develop a solution to improve adherence.

2. A
Garlic, ginger and ginkgo biloba increase the risk of bleeding. In contrast
ginseng, green tea, soy and St. John’s Wort decrease the effectiveness of
warfarin.

3. B
Swimming and cycling are not high impact physical activities, therefore there
are not preferred in the prevention of osteoporosis.

4. A
5. D

6. B
Confusion and anxiety are also common symptoms.

7. C
Vaginal deodorants should be avoided.

8. B
Increased ocular pressure is a sign of glaucoma.

9. D

10. C.
Apply a cold compress instead.

11. D

12. A
Freezing for 24 hours is required.

13. D
DPN affects both type I and type II diabetics. DPN puts patients at risk of
developing foot ulcers, infections, gangrene and ultimately amputation. DPN
has several contributing factors including metabolic, autoimmune,
neurovascular and lifestyle.
14. D

15. B
Bonion is a deformation of the big toe. Poor blood flow leads also to
temperature differences throughout feet.

16. B
Side effects of gentamycin ear drops include hearing loss, ringing in the ear,
dizziness and loss of coordination. Gentamycin is also used in the
prophylaxis of bacterial endocarditis, IV/IM 1.5 mg/kg 30 min before
procedure may repeat in 8 hours.

To use eardrops, follow these steps:

• Hold the bottle in your hand for 1 or 2 minutes to warm the solution.

• Shake the bottle well.

• Lie down with the affected ear upward.

• Place the prescribed number of drops into your ear.

• Be careful not to touch the tip to your ear, fingers, or any other surface.

• Remain lying down with the affected ear upward for 30-60 seconds.

• Repeat steps 1-6 for the opposite ear if necessary.

17. A
Sumatriptan – ergotamine interaction results in increased blood pressure.

18. C
Bismuth subsalicylate is hydrolyzed to salicylate therefore it is
contraindicated in a patient with aspirin sensitivity.

19. A
Many of the initial symptoms of hypoglycemia are caused by the release of
adrenaline by the body to attempt to slow the rapid fall of sugar in the blood.
A sudden release of adrenaline results in the following symptoms:

• Trembling and shakiness


• Heart palpitations
• Sudden sweaty palms
• Inner trembling
• Nausea
• Cold hands and feet
20. D
Antiemetics are not routinely recommended with levonorgestrel-only because
they have low incidence of vomiting unlike combined ECPs. Common side
effects of levonorgestrel only ECP are abdominal pain, fatigue, headache,
dizziness, and breast tenderness. Side effects usually do not occur for more
than a few days after treatment, and they generally resolve within 24 hours.
Temporary disruption of the menstrual cycle is also commonly experienced.
If taken before ovulation, the high doses of progestin in levonorgestrel
treatments may induce progestin withdrawal bleeding a few days (usually
within 7 days) after the pills are taken.

21. D
Celecoxib contains a sulfonamide moiety and may cause allergic reactions in
those allergic to other sulfonamide-containing drugs. This is in addition to the
contraindication in patients with severe allergies to other NSAIDs. Celecoxib
is predominantly metabolized by cytochrome P450 2C9. Caution must be
exercised with concomitant use of 2C9 inhibitors, such as fluconazole, which
can greatly elevate celecoxib serum levels. In addition, celecoxib may
increase the risk of renal failure with angiotensin converting enzyme-
inhibitors, such as lisinopril, and diuretics, such as hydrochlorothiazide.

22. C
Tetrahydrozoline, a derivative of imidazoline, is found in over-the-counter
eye drops and nasal sprays. Other derivatives include naphazoline,
oxymetazoline, and xylometazoline. Tetrahydrozoline is an alpha agonist and
its main mechanism of action is the constriction of conjunctival blood vessels
resulting in the relief of redness of the eye caused by minor ocular irritants.
Antazoline is a 1st generation antihistamine with anticholinergic properties
used to relieve nasal congestion and in eye drops, usually in combination
with naphazoline, to relieve the symptoms of allergic conjunctivitis.

23. D
Amitriptyline is used for a number of medical conditions including
depressive disorders, anxiety disorders, attention deficit hyperactivity
disorder, migraine prophylaxis, eating disorders, bipolar disorder, post-
herpetic neuralgia, and insomnia. Amitriptyline is also used in the treatment
of nocturnal enuresis in children. Amitriptyline may be prescribed for other
conditions such as cyclic vomiting syndrome, post-traumatic stress disorder
(PTSD), chronic pain, tinnitus, chronic cough, carpal tunnel syndrome (CTS),
fibromyalgia, interstitial cystitis, male chronic pelvic pain syndrome, irritable
bowel syndrome (IBS), diabetic peripheral neuropathy, neurological pain,
laryngeal sensory neuropathy, chronic fatigue syndrome and painful
paresthesias (loss of sensation) related to multiple sclerosis. Typically, lower
dosages are required for pain modification of 10 to 50 mg daily. Maximum
doses of amitriptyline in adults and adolescents/elderly are respectively
150mg -300mg and 150mg-200mg.

Symptoms of overdose may include:

• Irregular heartbeat

• Seizures, loss of consciousness

• Confusion

• Problems concentrating

• Hallucinating

• Agitation

• Drowsiness

• Rigid muscles

• Vomiting

• Fever

• Cold body temperature

24. D

25. D
The dose of doxycycline is too high and erythromycin 125 mg QID is too low
compared to normal dose of 500 mg QID.
Clearance is often measured as milliliters/minute (ml/min). Normal values
are:

Male: 97 to 137 ml/min.


Female: 88 to 128 ml/min .

26. B

27. A

28. D. St. John’s wort reduces significantly the bioavailability of indinavir


resulting in failure of treatment.

29. D

30. D

Competency #6: Drug Distribution

Questions

1. The substance which is principally an emollient laxative is:

A. Bran
B. Methylcellulose
C. Magnesium hydroxide
D. Mineral oil

2. A vial of Sodium Chloride Injection contains 3 mEq of sodium chloride


per mL. What is the percentage strength of this solution? (Molecular weight
of sodium chloride is 58.44 g per mol.)
A. 5.6%
B. 8.0%
C. 13.7%
D. 17.5%

3. A patient is allergic to mineral oil; his physician has asked you to


substitute an equal weight of olive oil for the 10 ml of mineral oil in the
lotion. How many milliliters of olive oil would be needed? Mineral oil has a
specific gravity of 0.85. Olive oil has a specific gravity of 0.91.

A. 4.25 ml
B. 8.53 ml
C. 9.34 ml
D. 10.71 ml

4. How many milliliters of normal saline must be added as diluent to one


fluid ounce of 1% phenylephrine HCl solution to reduce the strength to
1/4%?

A. 60 ml
B. 75 ml
C. 80 ml
D. 90 ml

5. Compounded preparations containing one or more soluble ingredient(s)


dissolved usually in water where the ingredients molecules are uniformly
dispersed in the solvent by taking advantage of intermolecular interactions
resulting in the formation a unique phase are:

A. Solutions
B. Emulsions
C. Dispersions
D. Suspensions
6. All are correct statements concerning elixir, EXCEPT:

A. A sweetened alcoholic drug formulation


B. Convenient drug formulation for kids due to sweet taste
C. Contains about 40% to 60% of alcohol
D. Relatively stable and easy to prepare

7. The following are all advantages of using automated drug dispensing


systems, EXCEPT:
A. Dispensing errors are reduced
B. Control over medications increases
C. Cost effectiveness
D. Reduction of the likelihood of drug inventory contamination

8. The concentration each ingredient in a compounded preparation is heavily


dependent on its weight. Therefore, each pharmacy technician must be
knowledgeable in proper handling of balance weights. To keep balance
weights accurate, they should:
A. Be picked up with gloved fingers, in a clean state, never dented or
dropped, and calibrated once a year
B. Be picked up with gloved fingers, in a clean state, never dented or
dropped, and calibrated once a month
C. Be picked up by tweezers, in a clean state, never dented or dropped,
and calibrated once a month
D. Be picked up by tweezers, in a clean state, never dented or dropped,
calibrated once a year

9. Which of the following is an example of active ingredient in a


compounded preparation?
A. Flavoring agent
B. Coloring agent
C. Receptor agonist
D. Emulsifier agent

10. The following are all characteristics of parenteral preparations, EXCEPT:

A. Free of pyrogen
B. Free of physical particles
C. Free of oily solvent
D. Free of microorganisms

11. You are in charge of training a pharmacy technician in a compounding


laboratory. You will instruct the technician to perform all manipulations
under the LAH hood at least:

A. 2 inches from the edge to avoid contamination


B. 5 inches from the edge to avoid contamination
C. 7 inches from the edge to avoid contamination
D. 9 inches from the edge to avoid contamination

12. You are in charge of training a pharmacy technician in a compounding


laboratory. You will instruct the technician to follow the all of following
guidelines while using a LAH hood, EXCEPT:

A. Wipe the hood interior with 70% isopropyl alcohol


B. Clean the hood front cover with 70% glyceraldehyde
C. Operate the hood for 30 to 45 min before use
D. Wear appropriate protective clothing

13. Sterile products should be prepared in class 100 environment which is


accomplished using a HEPA filter. Class 100 environment means:
A. No more than 100 particles 0.22 micron or larger present per cubic foot
B. No more than 100 particles 0.5 micron or larger present per cubic foot
C. No more than 100 particles 0.7 micron or larger present per cubic foot
D. No more than 100 particles 0.7 micron or larger present per cubic foot

14. Contact with cytotoxic drug may cause all of the following acute
reactions, EXCEPT:

A. Rash
B. Dizziness
C. Nausea
D. Impaired fertility

15. Pharmacy technicians must possess which of the following basic


requirements to participate in sterile compounding?

A. A working knowledge of the policy and procedures manual for


compounding, dispensing, and delivering sterile products.
B. Adequate training and adherence to hygienic and aseptic combined
with proficiency in pharmaceutical calculations
C. Knowledge and awareness of the proper methods to store label and
dispose of drugs and supplies.
D. Adequate of the process of performing the final check to compounded
products

16. All are correct statements concerning compounding ingredients,


EXCEPT:

A. Bulk, or unformulated, drug substances and added substances, or


excipients, must be stored according to official monographs or
manufacturer
B. The date of receipt in the compounding facility is not important as long
as the label indicates the ingredient’s expiration date
C. After receipt by the compounding facility, packages of ingredients that
lack a supplier’s expiration date cannot be used after one year
D. However, if appropriate inspection or testing indicates that the
ingredient has retained its purity and quality for use in compounded
sterile preparations it can be used beyond one year

17. Effective record keeping is one of the keys concerns of pharmacy


manager. The record of adverse reactions related to a compounded
preparation must be maintained on file for a minimum of two years
following:

A. The date the complaint was submitted


B. The date the prescription was received
C. The date the prescription was dispensed
D. The date the prescription was compounded

18. You have just received by e-mail a notification for the release of a new
antihypertensive drug. Which of the following sources will be best suited for
learning about the newly approved drug?

A. Compendium of Pharmaceuticals and Specialties


B. Therapeutic Choices
C. Medline
D. E. Patient Self-Care

19. The quality of compounded preparations is highly dependent on the


quality of compounding ingredients. Therefore, the pharmacist should ensure
the quality of the ingredients by using products with a standard designation
such as:

A. National Formulary (NF)


B. United States Pharmacopeia (USP)
C. British Pharmacopeia (BP)
D. All of the above
20. A 70kg patient who is receiving procainamide infusion, 1.8mg/minute in
the hospital is being switched to oral procainamide. You are the hospital
pharmacist on duty. Which of the following procainamide SR tablets would
you recommend to this patient assuming a dosing regimen 1 tab QID?
Procainamide has oral bioavailability of 85%.

A. SR 100 mg
B. SR 250 mg
C. SR 500 mg
D. SR 750 mg

21. A 34-year old female patient has been prescribed fluticasone inhaler for
the treatment of asthma. The patient generally gets her prescription filled
every 30 days. If the inhaler contains 260 inhalations, what is the average
daily regimen followed by this patient?

A. 1 Inhalation TID
B. 1 Inhalation QID
C. 2 Inhalations TID
D. 2 Inhalations QID

22. The cancer chemotherapy drug daunorubicin is available as an injection


(2 mg/mL), which is diluted in 250 mL of 0.9% sodium chloride injection for
intravenous infusion. A physician order calls for a dose of 16 mg of drug is to
be infused in a patient with leukemia. The percent concentration (w/v) of
doxorubicin in the infusion is:

A. 0.0019%
B. 0.0035%
C. 0.0062%
D. 0.0074%
23. A vial with a stock solution contains 20 million units of penicillin. The
label directions state that when 32.4 mL of sterile water for injection are
added, an injection containing 500,000 units of penicillin per milliliter
results. A prescription order for a patient with syphilis reads: dispense 1
million units of penicillin per milliliter. How many milliliters of sterile water
for injection should be used to prepare the product?

A. 5.3 ml
B. 9.7 ml
C. 12.4 ml
D. 14.6 ml

24. A patient is taking potassium supplements as part of his regimen for the
management of arrhythmias. He takes one 10mEq tablet of potassium. He has
decided to substitute bananas, a good source of potassium, for his
supplement.
Assuming one medium banana contains approximately 415 mg of potassium,
how many bananas will he have to take daily to get the equivalent of one
tablet of potassium supplement?

A. 1 Banana
B. 2 Bananas
C. 3 Bananas
D. 4 Bananas

25. A newly-diagnosed diabetic 65 kg patient is receiving an insulin dose of


0.5unit/kg/day. His insulin regiment consists of a combination of regular
insulin and Lente insulin in a ratio of 2:1 of Lente to regular insulin; 2/3 of
the total dose administered in the morning, and 1/3of the total dose
administered in the evening. What is the morning dosing regimen for this
patient?

A. 7 units Lente and 4 units regular insulin given subcutaneously qAM


B. 4 units Lente and 7 units regular insulin given subcutaneously qAM
C. 7 units Lente and 15 units of regular insulin given subcutaneously
qAM
D. 15 units Lente and 7 units regular insulin given subcutaneously qAM

26. A 3-year-old is receiving diphenydramine for the relief of allergies. The


prescribed dosage for this pediatric patient is 1.25mg/kg po q4h NTE
100mg/day. The elixir comes in 12.5mg/5ml. The child weighs about 35
pounds. Which of the following is the recommended amount for a single dose
of diphenidramine for this child? NTE: Not to exceed.

A. 1 tsp
B. 1.25 tsp
C. 1.5 tsp
D. 1.75 tsp

27. A pharmacy intern has just compounded an oral suspension of the


antibiotic cefuroxime axetil. The pharmacist on duty recommended storing
the suspension in the refrigerator for maximum stability. If stored at room
temperature, the half-life of the drug is only 10 days. If the original
concentration of the constituted suspension was 250 mg/5 mL, how much
cefuroxime axetil will remain per 5 mL after storage at room temperature for
15 days?

A. 25.0 mg
B. 44.2 mg
C. 74.6 mg
D. 93.7 mg
28. A patient presents with a prescription for testosterone transdermal patch.
Each patch contains 24.5 mg of testosterone and releases the drug at a rate of
5mg/24 hours. The patch is intended to be worn for 24 hours, then removed
and replaced by a new patch. Which of the following is the percentage of
total testosterone that has been released on removal of the patch?

A. 5.0%
B. 12.2%
C. 20.4%
D. 24.5%

29. A cancer patient has been prescribed sustained-release morphine 30mg po


TID, and morphine 10mg po q2h prn for breakthrough pain. During a follow-
up visit, the patient explains that he is currently using his prn morphine at a
rate of 2 doses before taking his sustained-release dose. His increased
reliance on prn morphine shows a lack of effective pain control. Which of the
following regimens would help better control his pain and reduce the need for
prn morphine?

A. Sustain- release morphine 35 mg po TID


B. Sustain- release morphine 55 mg po TID
C. Sustain- release morphine 75 mg po TID
D. Sustain- release morphine 95 mg po TID

30. An asthmatic patient receives a prescription for albuterol metered-dose


inhaler (MDI), which delivers 90 ug of drug per actuation. Approximately
11% of the dose is effectively delivered to lungs. The amount delivered to
lungs can be increased by an additional 9% by using a spacer. What is the
amount of drug delivered to lungs if the patient receives 2 actuations using a
spacer?

A. 16 ug
B. 18 ug
C. 27 ug
D. 36 ug

31. A patient is receiving 2.5 mg of albuterol for nebulization. Approximately


95% of the drug solution will be administered and 5% will remain in the
nebulizer. A patient usually inhales about only 10% of the total solution
nebulized while 90% is lost in the environment. About 9% of the amount
inhaled is actually absorbed in lungs. What is the amount absorbed following
the administration of 2.5 mg of albuterol by nebulization?

A. 21 ug
B. 25 ug
C. 30 ug
D. 33 ug

32. You are an intern in a community pharmacy. The pharmacist on duty


gives you a formula for an ophthalmic preparation base. The formula calls for
1L of a 0.02% w/v solution of benzalkonium chloride. How many milliliters
of a 1:750 w/v solution should be used to obtain the amount of benzalkonium
chloride needed in preparing the ophthalmic solution?

A. 75 ml
B. 100 ml
C. 125 ml
D. 150 ml

33. A medication order calls for triamcinolone suspension to be diluted with


normal saline solution to provide 2.5 mg/ml of triamcinolone for injection.
Assuming that each 5 ml of the suspension contains 135 mg of triamcinolone,
how many milliliters should be used to prepare 5 ml of the prescribed dilution
for injection?

A. 0.46 ml
B. 1.2 ml
C. 2.4 ml
D. 3.5 ml

34. A hospitalized patient is receiving an antibiotic, imipenim, for the


treatment of a severe infection. The patient profile shows: 73-year-old
female, 5'3", 167lbs with a CrCl (creatine clearance) of 26 ml/min. The
normal dose of imipinem is 500mg IV q6h. Assuming the following
guidelines in patient with kidney impairment what would be the adjusted q6h
dose in this patient?

CrCl (ml/min) Recommended Dose


31-70 500mg IV q8h
21-30 500mg IV q12h
6-20 250mg IV q12h

A. 125 mg IV q6h
B. 167 mg IV q6h
C. 250 mg IV q6h
D. 500 mg IV q6h

35. A 13-year-old 72 lbs boy presents with signs of sinusitis. He receives a


prescription for nine-day course of clarithromycin. Dosing recommendations
are7.5mg/kg po BID, with a maximum of 500mg/dose. Which of the
following clarithromycin strengths and sizes would you dispense?

A. 125mg/5ml in 100 ml bottle


B. 125 mg/ 5ml in 200ml bottle
C. 250mg/5ml in 100ml bottle
D. 250mg/5ml in 200ml bottle
36. A parenteral nutrition solution contains 600 mL of 5% w/v protein
hydroxylase,
400 mL of 50% w/v dextrose solution, 35 mL of a 20% w/v sterile potassium
chloride solution, 100 mL of sodium chloride solution, and 10 mL of a 10%
calcium gluconate solution. The solution is to be administered over 6 hours.
The dropper is calibrated to deliver 20 drops/ml. What is the rate of infusion?

A. 35 drops/min
B. 41 drops/min
C. 53 drops/min
D. 64 drops/min

37. A pharmacist needs to set up a patient-controlled analgesia epidural


infusion of 0.5 mg/ml morphine that allows the patient to self-administer a
dose every 30 minutes. The epidural dose of morphine should not exceed 10
mg/24 hours. What would be the maximum volume administered per dose
that would not exceed 10 mg/24 hours?

A. 0.25 ml
B. 0.42 ml
C. 1.10 ml
D. 1.55 ml

38. A 65-year-old male patient with chronic kidney disease and history of
congestive heart failure receives an order for dopamine drip. Dopamine
enhances kidney perfusion. The order reads: start dopamine at
2.5mcg/kg/minute. The patient weighs 78 kgs. A standard dopamine drip is
400mg in 500ml D5W. Which of the following is the calculated infusion rate
in ml/hr for this patient?

A. 2.5 ml/hr
B. 7.8 ml/hr
C. 14.6 ml/hr
D. 20.3 ml/hr

39. Erythropoietin is an endogenous hormone that stimulates the production


of erythrocytes in the treatment of anemia. The normal dose is 150 units/kg
divided into three doses per week. A 55-year-old weighing 185 lbs has been
scheduled to start treatment with erythropoietin. You have 10,000 units/ml of
erythropoietin. Which of the following is the calculated a single dose in ml of
erythropoietin for this patient?

A. 0.25ml
B. 0.42 ml
C. 1.0 ml
D. 1.44 ml

40. A patient is being switched from morphine drip to morphine oral tablets.
The patient had been receiving 3 mg/hr of the morphine as analgesic. 1mg of
IV morphine is equivalent to approximately 6mg of oral morphine. Oral
morphine sustained release tablets are available in 15mg, 30mg, 60mg, and
100mg tablets. The tablet dosing frequency is either every 8 hours or every
12 hours. Which of the following would be a recommended daily regimen for
this patient using as few tablets/dose as possible?

A. 1 tablets of the 100 mg strength q12


B. 2 tablets of the 100 mg strength q12
C. 4 tablets of the 60 mg strength q12
D. 6 tablets of the 30 mg strength q12

41. A cancer patient is receiving multiple cycles of chemotherapy for colon


cancer. The following drugs are administered over a 4-week period, with the
cycle repeating every 6 weeks.
Fluorouracil 500 mg/m2 IV, D-1, 8,15, 22
Leucovorin 20 mg/m2 IV, D-1, 8, 15, 22

Fluorouracil is available in an injection containing 50 mg/ml. How many


milliliters of each of fluorouracil injection would be administered during a
treatment cycle for a patient with a body surface area (BSA) of 1.7 m2?

A. 45 mL
B. 68 mL
C. 51 ml
D. 72 ml

42. A patient on chemotherapy is scheduled to receive granulocyte


macrophage colony stimulating factor (GM-CSF) to stimulate the production
of white blood cells. The prescribed dosage regimen is 240 mg/m2/day
intravenously over a period of 5 hours. BSA = 1.68 m2. The hospital
pharmacy has in stock lyophilized powder of GM-CSF for injection in 250
mg and 500 mg vials. What would be the final concentration of GM-CSF if
the daily dose is diluted to 50 ml with 0.9% sodium chloride solution?

A. 2 mg/ml
B. 5 mg/ml
C. 8 mg/ml
D. 10 mg/ml

43. A patient presents with a prescription for azelastine nasal spray for
seasonal allergic rhinitis. Two sprays per nostril BID. The nasal spray
contains 0.1% w/v of azelastine hydrochloride. Each standard container
delivers 200 metered sprays of 0.137 mL each. How much azelastine
hydrochloride would be contained in each spray? Two sprays per nostril BID.
Which of the following is the amount of azelastine administered in each
nostril?
A. 69 mcg
B. 137 mcg
C. 206 mcg
D. 274 mcg

44. A live attenuated vaccine contains 1.25 international units per 0.5 ml. The
postexposure dose is 2.5 international units administered on the day of the
exposure and an additional 2.5 international units on days 3, 8, 16, and 25
after exposure. How many milliliters of vaccine are needed for the full course
of treatment?

A. 3.5 mL
B. 5.0 mL
C. 7.5 mL
D. 10.0 mL

45. You are the pharmacist on duty. The following is a formula for a
clotrimazole and gentamicin otic preparation for an outpatient.

Clotrimazole 1.2 g
Gentamicin Sulfate 350 mg
Polyethylene Glycol ad 120 mL

The product is administered by drop. What is the amount of clotrimazole in


each drop of the otic preparation? The dropper delivers 20 drops/ml.

A. 0.5 mg
B. 1.0 mg
C. 1.5 mg
D. 2.0 mg

46. A prescribed insulin preparation contains 100 international units of


insulin glargine, equivalent to 3.6 mg per milliliter of injection. How many
international units of insulin glargine would be present in each 0.9 ml of
injection?

A. 85 International Units
B. 90 international Units
C. 95 International Units
D. 100 International Units

47. A patient presents to the pharmacy with a prescription for eye drops. The
prescription reads: 0.004% w/v latanoprost containing 0.015% w/v of the
preservative benzalkonium chloride.
Which of the following pairs represents the required amount in mg of
latanoprost and benzalkonium chloride for 3.0 ml of the ophthalmic
preparation?

A. 0.12 mg latanoprost; 0.45 mg benzalkonium chloride


B. 0.25 mg latanoprost; 0.66 mg benzalkonium chloride
C. 0.41 mg latanoprost; 0.15 mg benzalkonium chloride
D. 0.80 mg latanoprost; 0.30 mg benzalkonium chloride

48. You are an intern in a community pharmacy in the process of


compounding a gel containing famciclovir and chlorhexidine. The formula is
shown below.

Famciclovir 1.5g
Chlorhexidine Digluconate 300 mg
Propylene Glycol 1 mL
Hydroxypropyl Methylcellulose 300 mg
Preserved Water ad 10 g
How many milliliters of 20% w/v aqueous solution of chlorhexidine would
be required to compound this preparation?

A. 0.5 ml
B. 1.0 ml
C. 1.5 ml
D. 2.0 ml

49. You are an intern in a community pharmacy in the process of


compounding an interferon alpha ophthalmic solution. The formula is shown
below.

Interferon alpha 100 million units


Benzyl Alcohol 100 mg
Ammonium Acetate 7.7 mg
Human Albumin 10 mg
Sterile Water for Injection ad 10 mL

The pharmacy has in stock 18 million units or 67 mcg in 3 mL vials of


interferon alpha solution. Which of the following pairs represents the number
of micrograms and units of interferon alpha in each 0.1 mL of the final
preparation?

A. 0.9 mcg; 500,000 units


B. 1.8 mcg; 500,000 units
C. 3.7 mcg; 1,000,000 units
D. 4.5 mcg; 1,000,000 units

50. A patient is receiving oxtriphylline for the management of asthma. The


maintenance dose of oxtriphylline is 13.2 mg/kg/day or 800 mg, whichever is
less in 4 divided doses. How many 100 mg tablets of oxtriphylline should a
patient weighing 90 kg take as single dose?

A. 1 tablet
B. 2 tablets
C. 3 tablets
D. 4 tablets
51. A medication order calls for 5 mcg/kg of body weight of filgrastim to be
administered subcutaneously to a patient weighing 162 lb. The source of the
drug is a vial containing 0.5 mg in each 2.5 ml of the solution. How many
milliliters of the solution should be injected?

A. 1.25 ml
B. 1.47 ml
C. 1.66 ml
D. 1.84 ml

52. Two patients, #1 and #2, each weighing 127 lbs. are receiving ceftriaxone
according to the following regimens respectively 7.5 mg/kg every 12 hours
and 5 mg/kg every 8 hours. Which of the following statements best describes
the difference in the total quantity of ceftriaxone administered over a period
of 24 hours?

A. Patient #1 will receive 25% less drug than Patient #2


B. Patient #1 will receive 35% less drug than Patient #2
C. Patient #1 will receive 50% less drug than Patient #2
D. Patient #1 and Patient #2 will receive equal amounts of drug

53. A patient presents with a prescription for amoxicillin. The prescription


calls for “Amoxicillin 500 mg po BID for 7 days”. What is the volume of
amoxicillin 135 mg/5ml required to dispense a 7-day supply?

A. 260 ml
B. 340 ml
C. 550 ml
D. 670 ml

54. A cancer patient has been scheduled to receive ondansetron for the
management of chemotherapy-induced emesis. The intravenous dose of
ondansetron is three 0.15 mg/kg doses infused over15 minutes. The patient
weighs 70 kg. The daily dose is diluted to 50 ml with 5% dextrose. What
flow rate, in ml/hr, would be needed to administer the dose over 15 minutes?

A. 100 ml/hr
B. 200 ml/hr
C. 300 ml/hr
D. 400 ml/hr

55. An antiviral drug is administered intravenously to a hospitalized patient at


a loading dose of 50 mcg/kg over 10 minutes, followed by continuous
intravenous infusion of 0.375 mcg/kg/minute. The patient weighs 72.5 kg.
How many micrograms of the drug would be administered to the patient in
the first hour of therapy?

A. 1.5 mg
B. 3.0 mg
C. 5.0 mg
D. 7.5 mg

56.
Penicillin G Add 9.6mL of sterile water to a final
For I.V. injection concentration of 1,000,000 units/
1,000,000units 20mL
Stable for 48 hours at room
temperature or 7 days in the
refrigerator

The physician’s order reads: Penicillin G 350,000 units IV q8h


How many ml are required for one dose?
A. 2.5 ml
B. 5.0 ml
C. 7.0 ml
D. 8.5 ml
57.
Azithromycin Add 9.8mL of sterile water to a final
For I.V. injection concentration of 100mg/ mL
1g Stable for 7 days in the refrigerator.

The physician’s order reads: Zithromax 250 mg IV b.i.d.


How many ml are required for one dose?

A. 2.0 ml
B. 2.5 ml
C. 3.0 ml
D. 3.5 ml

58. The following is a formula for a miconazole and tolnaftate topical


preparation.

Miconazole 2% w/v
Tolnaftate 1 g
Polyethylene Glycol 300 qs ad 100 mL

How many grams of tolnaftate would be needed to prepare 250 ml of the


formulation?

A. 2.5 g
B. 5.0 g
C. 7.5 g
D. 10.0 g

59. A patient receiving chemotherapy presents with a prescription for


antiemetic suppositories. Below is the formula for 30 antiemetic
suppositories.

Metoclopramide Hydrochloride 1.2 g


Haloperidol, powder 30 mg
Lorazepam 30 mg
Benztropine 30 mg
Cocoa butter 56 g

While compounding the formula you used 30 1mg lorazepam tablets, each
weighing 200 mg and 15 benztropine 2 mg tablets, each weighing 180 mg as
sources of the two components. What would be the weight of each
suppository?

A. 0.2 g
B. 1.2 g
C. 2.2 g
D. 3.2 g

60.
Ceftriaxone Add 3.2mL of sterile water to a final
For I.V. injection concentration of 300mg/ mL
1g Stable for 7 days in the refrigerator.
The physician’s order reads: Cefoxitin 750 mg IV t.i.d. How many ml are
required daily?

A. 2.5 ml
B. 5.0 ml
C. 7.5 ml
D. 10.0 ml
61. You are the pharmacist on duty in a hospital pharmacy. A patient receives
an order for potassium iodide as expectorant. The physician ordered 0.5 ml of
potassium iodide expectorant. The label reads 355 mg/tsp. How many
milligrams are contained in this dose?
A. 17.5 ml
B. 35.5 ml
C. 51.0 ml
D. 64.5 ml

62. How many liters of 95% v/v alcohol should be used in preparing 1 gallon
of a 1:1000 solution of benzethonium chloride in 70% v/v alcohol?

A. 1.6 liters
B. 2.8 liters
C. 3.5 liters
D. 4.0 liters

63. You are an intern in a community pharmacy. You are instructed to


reconstitute a dry powder of gentamycin. According to the guidelines, the
powder is diluted with water to 80 ml to prepare a prescription containing
125 mg of amoxicillin per 5 ml. What is the percentage strength of
amoxicillin in the prepared prescription?

A. 1.8 w/v
B. 2.0 w/v
C. 2.5 w/v
D. 3.2 w/v

64. You are the pharmacist on duty in a hospital pharmacy. A physician


orders 600 mg of ampicillin IM q8h. The directions for dilution on the 2 g
vial of ampicillin reads: Reconstitute with sterile water to a total volume of
5.0 ml. How many ml would you instructs the nurse to administer to the
patient per dose?
A. 0.5 ml
B. 1.0 ml
C. 1.25 ml
D. 1.5 ml

65. You are the pharmacist on duty in a hospital pharmacy. A physician


ordered 180 mg of phenytoin po q8h. The patient weighs 67 kg. The label of
the drug reads 250 mg per 5 ml. How many milliliters should be administered
to this patient daily?
A. 3.6 ml
B. 5.5 ml
C. 7.2 ml
D. 10.8 ml

66. A prescription calls for 50mcg/hr patch of fentanyl; the patch has a
duration of action of 72 hours. If a prescriber wants to switch a patient from
fentanyl patch to oral oxycodone, how many 5mg tablets of oxycodone are
required to fill a 7-day supply? Approximately 5mg of oxycodone q12h can
be substituted for each 25mcg/hr of transdermal fentanyl.

A. 7 tablets
B. 14 tablets
C. 21 tablets
D. 28 tablets

67. A 68-year-old patient is taking 5mg of oral meperidine six times per day
for the relief of pain associated with stomach cancer. If his physician wants to
switch him to hydromorphone suspension, how many mls of hydromorphone
0.75 mg/ml would be required to fill a week supply? Assume hydromorphone
and meperidine are equipotent.

A. 140 ml
B. 280 ml
C. 350 ml
D. 475 ml
68. You are the pharmacist on duty in a hospital pharmacy. You have just
received an order for 8% propranolol. In order to dispense 8% of 250 ml
solution of propranolol, what is the required volume in ml of stock solution?
The stock solution is 100mg/ml.

A. 150 ml
B. 200 ml
C. 250 ml
D. 300 ml

69. You are the manager in a community pharmacy. If 1000 tablets of


linezolid cost $7580, and the mark up on the prescription is 20%, what would
be the retail price of 50 tablets?

A. 115
B. 279
C. 455
D. 758
Answers

1. D

2. D
1 mEq = 1 Eq/1000 = 58.44 g/1000 = 0.05844 g = 58.44 mg.
Amount of sodium chloride in 3 mEq per mL = 58.44 mg per mEq × 3 mEq
per mL = 175.32 mg per mL.

3. C
Specific gravity = Number of grams of the substance / Number of milliliters
of the substance

4. D

5. A

6. B
Elixirs are unsaturated solution of sugar; they are not convenient for children
due to high alcohol content.

7. D
Automated drug dispensing systems are also convenient for frequently used
medications.
8. D
9. C
Many drugs (active ingredient) act by binding to receptor; they are receptor
ligands such agonists, antagonist or partial agonists.

10. C
Some parenteral preparations contain oil such as castor oil. Parenteral
preparations are also free of precipitate.

11. C

12. B
Glyceraldehyde will leave an oily residue.

13. B

14. D
Impaired fertility, DNA damage, cancer and impaired fertility are chronic
reactions.

15. D
Final check prior to product release must be performed by a pharmacist.

16. B

17. C

18. C

19. D
Alternatively, the pharmacist can rely on a valid lot number and beyond-use-
date (if available). If expiry is not available, a date of receipt should be
recorded on the raw material OR a Certificate of Analysis (C of A) for raw
materials that is maintained in the records.

20. D
1.8 mg/min x 60min x 24hrs x 1 day/4 doses x 100% IV/85% PO = 762 mg
PO QID
21. D
250 inhalations/30 days = 8.3 inhalations per day ~ 2 inhalations QID

22. C

23. C 12.4 ml sterile water for injection

24. A
MW of K = 40 mg/mmol Eq wt = 40 mg/mEq, since valence = 1
10 mEq KCl = 10mEq K+ and 10mEq of Cl
10 mEq K x 40 mg K x 1 banana/415 mg = 0.96 bananas about 1 banana

25. D
0.5 unit/kg x 65 kg = 33 units per day
33 units x 2/3 = 22 units for AM administration
33 untis x 1/3 = 11 units for PM administration

Breakdown of 22 units for AM administration


22 x 2/3 = 14.67 units ~ 15 units Lente
22 x 1/3 = 7.3 units ~ 7 units Regular insulin

26. B
35 lb = 15.6 kg
Daily dose is 15.6 kg x 1.25 mg = 19.5 mg x 6 (24hrs/4 = 6) = 117 mg
117 mg has exceeded the maximum daily amount therefore the daily dose is
100mg
Single dose is 100mg/6 = 16.7 mg
16.7 mg/12.5 = 1.3 tsp because each tsp or 5ml has 12.5 mg of
diphenhydramine
1.3 tsp ~ 1.25 tsp

27. D
15 days= 1.5 half-lives

28. C
29. B
Total current daily dose required for pain relief: Sustained-released morphine
as prescribed = 30 mg x 3 + prn morphine (2 doses before each sustained
morphine dose) 10 mg x 6 = 150 mg
Regimen B will provide sufficient morphine to provide the required daily
amount.

30. D
180 ug for 2 actuations
Percentage of albuterol delivered to lungs using a spacer = 11% + 9% = 20%
Amount delivered to lungs = 180 ug x 20% = 36 ug

31. A
Amount nebulized: 2.5mg x 95% = 2.375 mg
Amount inhaled: 2.375 x 10% = 0.2375 mg = 237.5 ug
Amount absorbed: 237.5 ug x 9% = 21.4 ug ~ 21 ug

32. D

33. A
Amount of triamcinolone required for 5mlinjection: 2.5 ml x 5 = 12.5 ml
Volume of suspension needed: 5 x 12.5/135 = 0.46 ml

34. C
Based on CrCl value and guidelines the recommended dose is 500 mg IV q
12 h. 50% reduction of normal q6h dose = 500 mg/2 = 250 mg q6h

35. C
72 lb ~ 33 kg
Single dose is 33 kg x 7.5 mg/kg = 247.5 mg
Therefore 250/5ml strength would be preferred
Total volume required: 10 ml daily (BID) x 9 days = 90 ml required. 100ml
bottle would be preferred.

36. D
37. B
48 doses daily are required for administration every 30 min
Maximum amount of morphine per dose: 10 mg/48 = 0.21 mg
Volume of epidural infusion required to provide 0.21 mg:0.21/0.50 = 0.42 ml

38. C
Dose required per hour for 78 kg patient: 78 x 2.5 mcg x 60 = 11700 mcg/hr
= 11.7 mg/hr
Volume of dopamine drip equivalent to 11.7 mg: 11.7 x 500/400 = 14.6 ml
Infusion rate is 14.6 ml/hr

39. B
185 lb = 84 kg
Total weekly dose is 84 kg x 150 units = 12600 units per week
Single dose is 12600/3 = 4200 units
Single does in ml is 4200 units/10000 units/ml = 0.42 ml

40. B
Oral morphine daily dose: (3 mg IV/hr)(24 hrs/day)(6mg oral/1mg IV) = 432
mg oral/day. 2 tablets of 100 mg strength q12

41. B

42. C

43. D
Volume of 200 sprays = 200 x 0.137 = 27.4 ml
0.1% means 0.1 g per 100ml therefore 27.4 ml = 27.4 mg = 27400 mcg
Amount of azelastine in each spray = 27400 mcg/200 = 137 mcg
For 2 sprays per nostril: 137 mcg x2 = 274 mcg azelastine hydrochloride per
dose (two sprays per nostril)

44. B
1 ml of vaccine is needed to provide 2.5 international units. 1 ml x 5 days = 5
ml
45. A
Amount of clotrimazole in 1ml = 1.2g/120ml = 0.01g = 10mg
In 1 drop: 10 mg/20 drops = 0.5 mg

46. B
In 0.9 ml injection, you have 0.9 x 3.6 mg = 3.24 mg
3.24 mg is equivalent to 3.24/3.6 x 100 units = 90 units

47. A
Latanoprost: (3 x 0.004 /100) x 1000 = 0.12 mg
Benzalkonium chloride: (3 x 0.015/100) x 1000 = 0.45 mg

48. C

49. C

50. B

51. D
162 lb = 73.5 kg
Total dose: 5 mcg/kg x 73.5 kg = 367.5 mcg = 0.3675 mg
Volume required: 2.5 mlx 0.3675 mg/0.5 mg = 1.84 ml

52. D

53. A
7 g needed daily therefore 7 g or 7000 mg for 7 days
Volume of amoxicillin solution required: 7000 mg x 5 ml/135 mg = 259.3 ml
~ 260 ml

54. B

55. C

56. C
350,000 units / 1,000,000 units x 20 ml = 7ml
57. B
250 mg/100 mg x1 ml = 2.5 ml

58. A

59. C
60. C
Volume per dose: 750 mg/300 x 1 ml = 2.5 ml
Daily volume required = 2.5 ml x 3 = 7.5 ml
61. B
1tsp = 5 ml
355 mg x 0.5 ml / 5 = 35.5 ml
62. B

63. C
64. D
2 g = 2000mg per 5 ml
Volume per dose = 5 x 600 / 2000 = 1.5 ml
65. D
Volume per dose: 180mg x 5 / 250mg = 3.6 ml
Daily dose = 3.6 ml x 3 = 10.8 ml
66. D
Two 5mg tablets of oxycodone q12h are required for 50 mcg/hr fentanyl
patch prescribed initially. Four tablets daily. 4 x7-day supply = 28 tablets

67. B
Daily amount of meperidine = 5 mg x 6 = 30 mg
Weekly = 30 mg x 7 = 210 mg
Volume of hydromorphone suspension required to dispense 210 mg of
hydormorphone assuming meperidine and hydromorphone are equipotent:
210/0.75 = 280 ml
68. B
Amount of propranolol in 8% 250 ml: 8 x 250/100 = 20g
Volume of stock solution required: 100 mg/ml = 0.1g/ml
20/0.1 = 200 ml
69. C
Cost for 50 tablets: 7580 x 50 / 1000 = $379
Mark up is $379 x 20/100 = $75.8 ~ $76
Market value of 50 tablets = $379 + $76 = $45
Competency #7: Practice Setting

Questions

1. Which of the following statements are examples of conflict between a


pharmacy manager ethics obligations and management policies?

A. Acting on behalf of a pharmaceutical company to promote their newly


approved drug
B. Accepting a vacation package for your spouse from a pharmaceutical
firm
C. Investing in a company whose drug compliance aids are recommended
by your pharmacy
D. All of the above

2. All of the following are good inventory management strategies, EXCEPT:

A. Having in place a reordering schedule to avoid being out of stock


B. Establish a stock rotation routine to decrease the amount of expired
drugs
C. Maintaining an inventory that reflects provincial demand
D. Handle returns promptly to receive credit from the manufacturer

3. All are required on a trimmed drug label, EXCEPT:


A. Name of the drug
B. Refills schedule
C. The date the prescription is dispensed
D. The prescription number

4. A pharmacy has recently experienced a noticeable decrease in sales. What


will be the best way to reduce the pharmacy expenses?

A. Reduce the number of pharmacy technicians


C. Reduce the number of staff
D. Reduce pharmacists overlap
E. Reduce the duration of counseling sessions

5. A pharmacy has recently experienced an incidence of drug dispensing


error. Which of the following strategies would NOT be a primary concern in
handling the issue?

A. Discuss the situation with the staff involved


B. Assess the situation to recommend remedies
C. Submit a report to Health Canada
D. Notify the patient and apologize

6. The decision to replace a preexisting drug with a newly approved drug in a


hospital drug formulary could be based on all of the following factors,
EXCEPT:

A. Enhanced effectiveness
B. Low cost
C. Once a day dosing
D. More invasive administration
7. The style of management in which the pharmacy manager implements
owns opinions without consultations is called:

A. Consultative
B. Autocratic
C. Bureaucratic
D. Benevolent

8. A 29 years old female with a terminal condition is refusing an


experimental drug which may prolong her life. The patient is fully competent
and aware of the consequences of her decision. Her husband insists that the
hospital pharmacist gives her the experimental drug. Which of the following
ethical principle gives the pharmacist the right NOT to provide the
experimental drug?

A. Confidentiality
B. Veracity
C. Autonomy
D. Justice

9. In the process of drug dispensing which of the following statements


describes a part-fill?

A. Verbal approval from a practitioner authorizing a pharmacist to


dispense additional quantity of drug pursuant to a prescription
B. Written approval from a practitioner authorizing a pharmacist to
dispense additional quantity of drug pursuant to a prescription
C. Authorization by a full pharmacist to dispense additional quantity of
drug pursuant to a previously dispensed prescription
D. Verbal or written prescription prescribed to be dispensed in divided
amounts

10. All of the following statements are correct concerning a verbal


prescription, EXCEPT:

A. A verbal prescription may be received from a physician recorded voice


message
B. The pharmacist must make a written record of a verbal prescription
and include her/his signature or initial
C. A verbal prescription is not permitted for Fiorinal C1/4
D. A verbal prescription is permitted for narcotic preparations

11. Which of the following is the formulation of the analgesic fiorinal C1/2?

A. Butalbital 50 mg, caffeine 40 mg, ASA 330 mg, codeine 15 mg


B. Butalbital 50 mg, caffeine 40 mg, ASA 330 mg, codeine 30 mg
C. Butalbital 50 mg, caffeine 40 mg, ASA 330 mg, codeine 35 mg
D. Butalbital 50 mg, caffeine 40 mg, ASA 330 mg, codeine 40 mg

12. All of the following statements are correct regarding refill authorizations,
EXCEPT:

A. The pharmacist must always create a new prescription number to


ensure effective record keeping
B. The pharmacist must cancel any unused refill authorizations remaining
on any previous prescription if a patient presents a new prescription for
a previously dispensed drug
C. The pharmacist must advise the other pharmacy of the new
prescription if unused refills are at another pharmacy
D. All of the above

13. A full pharmacist may authorize a prescription renewal by dispensing


additional quantity of drug pursuant to a previously dispensed
prescription. All of the following guideline must be followed by a
pharmacist for a prescription renewal, EXCEPT:

A. Create a written record


B. Assign his/her identification number
C. Assign the original prescription number
D. Ensure the drug dispensed is consistent with the previous
prescription

14. In a hospital setting which of the following statement describes best


the term “individual patient prescription system”?

A. A form of drug distribution in which drugs are dispensed in patient-


specific labeled drug containers
B. A form of drug distribution in which orders for each patient are dispensed
individually and packaged in unit-of-use packages containing one dose
C. A form of drug distribution in which drugs are dispensed in uniformly
designed and labeled drugs containers for all patients
D. A form of drug distribution where drugs are stocked in a patient care area
and are not labeled for a particular patient

15. In a hospital pharmacy inpatient prescription labels must include:

A. The generic name, strength and dosage form


B. Hospital approved abbreviations and symbols
C. A unique patient identifier
D. All of the above
16. In a hospital, unused dispensed drugs must be returned to the hospital
pharmacy and must not be re-dispensed with few exceptions. Which of the
following statements is NOT an exception to this guideline?

A. A dispensed drug returned to the hospital pharmacy in a sealed dosage


unit or container as originally dispensed
B. The label of the dispensed drug is intact and includes a legible drug lot
number and expiry date
C. Another care unit requires the same drug in the same strength and form
immediately
D. None of the above

17. In a hospital setting a targeted substance may be sold or provided without


a prescription, upon receipt of an order specifying the name, quantity and, if
applicable, the strength per unit of the targeted substance to:

A. A licensed dealer who sold or provided the targeted substance or who is


licensed to destroy targeted substances other than those that he produced,
made, assembled, sold or provided
B. A practitioner or a pharmacist if the order states that the targeted substance
is required because of a delay or shortfall in an order placed for the substance
C. Another hospital, if the order is placed by a pharmacist or practitioner
practicing in the other hospital and authorized by the hospital to order
targeted substances on behalf of that hospital and if the order states that the
targeted substance is required because of a delay or shortfall in an order
placed for the targeted substance
D.All of the above

18. During the process of the closure of a pharmacy where targeted


substances are stored, a pharmacist must inform the Minister of Health the
date of the closure, the location to which the targeted substance was moved
and the quantity of the targeted substance that has moved within:

A. 7 days after the closure


B. 10 days after the closure
C. 7 days before the closure
D. 10 days before the closure

19. A pharmacy dispensary area should be accessible only to authorized


personnel and contain no products inappropriate to the practice of pharmacy.
The pharmacy manager should ensure all of the following, EXCEPT:

A. Prescription drugs for external use are stored separately from internal
and injectable drugs
B. Non-prescription drugs for external use are stored separately from
internal and injectable drugs
C. Appropriate storage areas are designated for the storage of flammable
and hazardous products
D. Expired and unusable drugs are marked clearly and stored within the
current inventory until they are properly disposed

20. Which of the following statements is NOT an element of successful drug


disposal programs?

A. Public awareness through advertising, promotion of the service and


community education
B. Improved convenience for customers
C. Funding for waste disposal through staff contribution
D. Organized collection centre

21. A medication sample is defined as a trial package of medication


distributed to the pharmacist without cost. It would not include free or bonus
goods provided as part of a larger purchase. A pharmacist may dispense
sample medication according to which of the following guidelines?
A. The pharmacist may charge a professional fee and inform the patient
when medication samples are being dispensed
B. The sample must be appropriately labeled according to regulations
C. If the medication sample contains a Schedule I drug, a prescription is
required for distribution to the patient
D. All of the above

22. Emergency contraception regimens are two doses of oral contraceptive


tablets. The first dose is administered within 72 hours of unprotected
intercourse; the second dose is taken 12 hours later. The number of tablets
taken depends on the product used. In general, which of the following
combinations of ethinyl estradiol and levonorgestrel is taken with each dose?

A. 0.020 or 0.025 mg ethinyl estradiol and 0.100 or 0.125 mg


levonorgestrel
B. 0.030 or 0.050 mg ethinyl estradiol and 0.125 or 0.150 mg
levonorgestrel
C. 0.050 or 0.060 mg ethinyl estradiol and 0.250 or 0.300 mg
levonorgestrel
D. 0.100 or 0.120 mg ethinyl estradiol and 0.500 or 0.600 mg
levonorgestrel

23. The Triplicate Prescription Program (TPP) was established to monitor the
use of certain drugs prone to misuse and abuse for non-medical purposes. To
prescribe any of the medications listed under the TPP, which of the following
prescribers MUST use the Triplicate Prescription Program form?

A. Physicians - Dentists
B. Dentists - Veterinarians
C. Veterinarians - Physicians
D. All of the above

25. A triplicate prescription must be presented to the pharmacist by the


patient within:
A. Three days of the prescribing date
B. Seven days of the prescribing date
C. Fourteen days of the prescribing date
D. Twenty-one days of the prescribing date

26. Financial literacy is essential for any business owner including pharmacy
owners or managers. Which of the following business decision are expected
to be impacted by the owner’s financial literacy?

A. Performance evaluation
B. Resource allocation
C. Sustainability
D. All of the above

27. In Canada the number of prescriptions dispensed continues to increase


each year. Which of the following factors have positive effect on this trend?

A. Earlier diagnosis and treatment of chronic conditions


B. Aging of population
C. Direct-to-consumer advertising
D. Effective disease prevention strategies

28. The inventory turnover measures the number of turnovers of the


inventory over the course of a given period. Therefore, the inventory turnover
is an assessment of business efficiency. The Days Sales in Inventory is
another important value for the evaluation of business efficiency. This
number represents how long on average it takes to turn over the inventory.
Assuming that Wellness Pharmacy has a yearly inventory turnover of 7.5
what will be its Days Sales in Inventory?
A. 28
B. 49
C. 90
D. 157

29. A pharmacy profit margin measures how much out of every dollar of
sales the pharmacy maintains as earnings. This value is essential in the
process of resources allocation. Assuming that in 2013 Hope Pharmacy has a
net income of $650,000 and sales of $1,330,000 what will be its profit
margin?

A. 2
B. 5
C. 8
D. 11

30. Inventory typically represents the largest current asset for community
pharmacies. The goal of effective inventory management is to minimize the
carrying costs of excessive inventory and the shortage costs associated with
too little inventory. All on the following are examples of carrying costs,
EXCEPT:

A. Losses due to theft


B. Filling drug balances owed
C. Staff salary for handling expired drugs
D. Insurance and taxes

31. A pharmacy managerial decision making in the operation of a community


pharmacy requires a thorough understanding and consideration of many costs
including:

A. Sunk costs
B. Direct and indirect costs
C. Variable and fixed costs
D. All of the above

32. The cash break-even analysis is used in a pharmacy to determine the sales
volume needed to pay for fixed expenses by taking into consideration the
variable costs associated with those sales.
Cash break-even occurs when the:

A. Operating cash flow (OCF) = 0


B. Operating cash flow (OCF) > 0
C. Operating cash flow (OCF) < 0
D. Operating cash flow (OCF) = 1

33. The primary consideration for a pharmacy owner or manager when


allocating resources must be profitability. Profitability leads to business
sustainability. Which of the following it the most commonly used business
performance assessment?

A. Return on investment (ROI)


B. Return on sales (ROS)
C. Residual income (RI)
D. Net working capital (NWC)

34. The operating cycle of a pharmacy is the average time between the
acquisition of inventory and when cash is collected from receivables. Sunny
Pharmacy has purchased recently $150,000 in seasonal allergy medications.
The invoice was paid 30 days following receipt of the inventory. The
inventory was sold 25 days later with an account receivable period of 25
days. What is the pharmacy operating cycle for this particular inventory?
A. 30 days
B. 50 days
C. 55 days
D. 80 days

Answers

1. E

2. C
Your inventory should reflect local demand. Use of state-of-art computer
software to manage inventory is another good inventory management
practice.
3. B
A trimmed label is use when the container is too small to accommodate a
standard label. Name of the patient is also required on a trimmed label.

4. D
Reduction of drug inventory is another option.

5. C
Scheduling a staff workshop on the reduction of drug dispending errors
would be also useful.

6. D
Invasive administration routes such as IV are not convenient to the patient.

7. B

8. C

9. D
A and B describe a refill; a refill may be verbal or written.
C describes a prescription renewal.

10. C
A verbal prescription is permitted for narcotic preparations including fiorinal
C1/4 and fiorinal C1/2

11. B
A is fiorinal C1/4
Fiorinal C is a narcotic preparation used as analgesic for the relief of tension
headaches. Fiorinal C comes in two different formulation C1/4 and C1/2. It is
contraindicated in patients with bleeding disorders, peptic ulcer, respiratory
diseases (asthma, pneumonia) and porphyria. The formulation fiorinal does
not contain codeine: Butalbital 50 mg, caffeine 40 mg and ASA 330 mg

12. A
The pharmacist must create a new prescription number if a refill
authorization involves a different drug identification number, practitioner or
directions for use.

13.C
A new prescription number must be assigned.

14. A
B describes: unit dose distribution
D describes: ward stock

15. D

16. C
The other applicable requirement is the drug integrity can be verified.

17. D

18. B
The pharmacist has 10 days after the closure to provide to the Minister with
the required information.

19. D
The pharmacy manager should ensure that expired and unusable drugs are
removed from current inventory and stored in designated areas awaiting
proper disposal.

20. C

21. D
The pharmacist may charge a professional fee but not charge for the cost of
the medication. Distribution to patients of sample medication containing
Schedule II, III or unscheduled drugs does not require a prescription.
Without further packaging or labelling, there should be no charge for
distributing the manufacturer’s packaged sample of non-prescription
medication.
22. D
Examples of emergency contraception regimens include:

2 tablets of Ovral®: each tablet contains 250ug d-norgestrel/50 ug ethinyl


estradiol
4 tablets of Triphasil®: each yellow tablet contains125ug levonorgestrel/30ug
ethinyl estradiol
5 tablets of Alesse®: each pink tablet contains 100ug levonorgestrel/20ug
ethinyl estradiol
2 tablets of Preven®: each tablet contains 250 mcg levonorgestrel/50 mcg
ethinyl estradiol
1 tablet of Plan B®: each tablet contains 750 mcg levonorgestrel

Progestin-only mini pills may also be used, although this regimen is less
convenient because of the number of tablets that must be taken. The timing of
the first dose of medication is critical. The regimen becomes completely
ineffective by day 6 or 7 when implantation usually occurs. The sooner after
unprotected intercourse the tablets are taken, the more effective they will be.

23. A
Veterinarians are voluntarily involved in the program.

25. A
If the patient fails to present the prescription within three days (72 hours), the
prescription becomes void and the patient should be asked to get a new
triplicate prescription. However, the pharmacist may use their discretion to
call the prescriber and obtain approval to use an “expired” triplicate
prescription. The authorization should be documented.

26. D

27. D
Disease prevention strategies reduce drug consumption. Discovery and
approval of new drugs is another factor leading to increased number of
prescriptions.

28. B
Days Sales in Inventory = 365 days (one year) ÷ Inventory Turnover

29. A
Profit Margin = Net income ÷ Sales

30. B
The cost of filling drug balances owed is a shortage cost. Drug storage
expense is another example of carrying costs.

31. D
Opportunity costs such as the cost of leasing a larger pharmacy location and
the profit margin should be also considered. Sunk costs are those that have
already been incurred.

32. A

33. A

34. D
Operating Cycle = Inventory Period + Accounts Receivable Period
Inventory Period is the time it takes to acquire and sell inventory
Accounts Receivable Period is the time between the sale of inventory and the
collection of the receivable.
Cases Scenarios

Case 1

Mr. Lee is a diabetic patient with nephropathy who has just received a
prescription for an antihypertensive. Following clinical test his SCr value is
160 umol/L.

1. Nephropathy is determined based on albumin to creatinine ratio (ACR).


Which of the following ACR value range defines nephropathy?

A. Higher than 2.0 mg/mmol


B. Lower than 2.0 mg/mmol
C. Higher than 2.8 mg/mmol
D. Lower than 2.8 mg/mmol

2. Which of the following hypertensive drugs would be recommended to Mr.


Lee as initial treatment?

A. Hydrochlorothiazide
B. Bisoprolol
C. Valsartan
D. Oxprenolol

3. What will be the initial dosage for the drug?

A. 50 mg/day
B. 60 mg/day
C. 70 mg/day
D. 80 mg/day

4. Which of the following should be monitored during the course of therapy?

A. Serum creatinine - Sodium


B. Potassium - Serum creatinine
C. Sodium - Potassium
D. All of the above
5. Which of the following diuretics may be added to Mr. Lee’s therapeutic
regimen as second line therapy?

A. Loop
B. Osmotic
C. Thiazide
D. Carbonic Anhydrase inhibitor
Case 2

GT is a 28 years old patient being treated for a second episode of depression.


He is already taking fluoxetine.

1. What is the recommended minimum duration of treatment for GT?

A. 6 months
B. 12 months
C. 18 months
D. 24 months

2. GT is suspected to have treatment resistant depression therefore switching


class should be considered. His physician is considering switching from
fluoxetine to a non-selective irreversible MAOI. Which of the following
MAOIs would not be considered?

A. Phenelzine
B. Tranylcypromine
C. Moclobemide
D. Isocarboxazid

3. What is the recommended washout time for switching from fluoxetine to


phenelzine?

A. 1 week
B. 2 weeks
C. 5 weeks
D. 7 weeks

4. Which of the following drug classes is the first- choice for the treatment of
depression?

A. MAOIs
B. TCAs
C. SNRIs
D. SSRIs

Case 3

RS is a 41 years old patient diagnosed with community-acquired pneumonia


caused by methicillin-resistant Staphylococcus aureus (MRSA). His
pneumonia-specific severity of illness score (PSI) is 83.

1. Which of the following laboratory test findings has the highest


contribution in PSI?

A. Arterial pH < 7.35


B. Blood urea nitrogen > 11 mmol/L
C. Hematocrit < 30%
D. Partial pressure of arterial oxygen < 60 mm Hg

2. Which of the following drugs would be recommended to RS?


A. Cloxacillin
B. Cefazolin
C. Vancomycin
D. All of the above

3. What is the minimum duration of treatment for RS?

A. 3 days
B. 5 days
C. 7 days
D. 10 days

Case 4

You are the pharmacist in charge of training a pharmacy intern. One of your
regular customers IB comes in with a prescription for citalopram 20 mg daily.
IB has been diagnosed with schizophrenia 6 years ago. When you approach
him you notice surprisingly that he doesn’t really say much and hands you
the prescription and heads to the waiting area. His profile shows: male 32
years old; current medications: olanzapine 20 mg HS for the last 4 years
1. You decided to provide some background information to help the
pharmacy intern gain better understanding of the condition. Which of the
following statements is appropriate to use?

A. The positive and negative symptoms of schizophrenia are due to


dopamine excess
B. The 5HT2 serotonin receptor has been implicated in the
pathophysiology of schizophrenia
C. Affinity to D1 dopamine receptor primarily has been linked to
improvements in the positive symptoms
D. Blocking dopamine in the mesocortical dopamine pathway is
responsible for extrapyramidal adverse effects

2. The pharmacy intern is now asking about the risk factors for
schizophrenia. Which of the following statements is true concerning risk
factors?

A. If one identical twin has schizophrenia, the risk of schizophrenia


in the second is close to 100%.
B. The peak age of onset is 15-25 for men and women
C. Schizophrenia is strictly a neurotransmitter abnormality and if
corrected the patient would be cured.
D. Environmental risk factors for this condition include lower
socioeconomic status and living in a city.

3. During a previous interaction with IB before his diagnosis, you recall that
the patient has complained about hearing voices and becoming suspicious of
everything and everyone. For this reason, he is more comfortable spending
more time in his parents’ basement. Which of the following is a positive
symptom of schizophrenia?

A. Decreased speech
B. Antisocial behavior
C. Hearing whispers
D. Flat affect
4. The pharmacy intern mentioned that he learned in class that olanzapine has
been associated with significant weight gain. How often should IB have his
fasting blood glucose checked ideally?

A. Every 8 weeks
B. Every 12 weeks
C. Every 18 weeks
D. Every 20 weeks

5. Which of the following antipsychotics has the lowest incidence of weight


gain?

A. Ziprasidone
B. Clozapine
C. Risperidone
D. Quetiapine

6. Which of the following is an appropriate course of action regarding IB’s


prescription for citalopram?

A. Fill the prescription as usual as there is no concern


B. Call his physician to change the dosage
C. Fill the prescription but warn the patient that it can increase the
bioavailability of olanzapine and lead to more adverse effects
D. Call his physician to change the prescription to another class of
antidepressants

7. IB currently smokes about a pack of cigarettes per day and is thinking of


quitting. Which of the following would be appropriate to recommend to this
patient?

A. He should talk to his doctor first, as he may need to increase the dose
of olanzapine
B. He should talk to his doctor first, as he may need to switch to another
antipsychotic
C. He should talk to his doctor first, as he may need to decrease his
olanzapine dose
D. He should talk to his doctor first, as he may need to increase the
frequency of administration of olanzapine

Case 5

A 26 years old male patient, PR, enters the pharmacy and hands you a
prescription for clarithromycin 250 mg bid for 10 days. He explains that
clarithromycin has been prescribed by a physician form a walk-in clinic for a
chest infection. His profile shows that he is currently on risperidone 3 mg bid
(30-day supply dispensed) last filled 38 days ago.

1. What is the appropriate course of action for the clarithromycin


prescription?

A. Fill it as prescribed
B. Change to another medication to avoid P450 related interactions
C. No concern. Unlike erythromycin, clarithromycin is not
metabolized through the P450 enzymes pathway
D. Recommend azithromycin to prevent higher than normal
bioavailability of risperidone

2. During your interaction with PR, you should:

A. Look for signs of positive or negative symptoms considering that


usually schizophrenia requires long-term management
B. Inform the patient about the risk of combining risperidone and
clarithromycin
C. Ask the patient if his schizophrenia is being monitored by his
physician
D. All of the above

Case 6

CT is one of your regular patients. He came to ask you a question regarding


his schizophrenia medication. He has been taking quetiapine 300 mg bid to
help control his symptoms, but he has found some of his symptoms have
been coming back. He was on risperidone therapy 2 years ago. His
psychiatrist has recommended that he change his regimen to clozapine. He
has heard that it has quite a few side effects and because he knows and trusts
you, he wanted to discuss them with you. He wants to know as much as you
can tell him about this drug and its side effects.

1. Clozapine has a:

A. Low risk of sedation


B. Low risk of urinary retention
C. High risk of sexual side effects
D. High risk of an increase in total cholesterol

2. When discussing the possible change to clozapine for CT, which of the
following statements is the most appropriate?

A. This change is appropriate, as CT is having an increase in his


symptoms and clozapine may help for patients who fail on other
antipsychotics.
B. This change is appropriate, as most patients with schizophrenia
will eventually require clozapine therapy.
C. This change is not appropriate, as a change to any SGA is likely
to produce the same effect as clozapine.
D. This change is not appropriate, as the cost of monitoring and risk
of adverse effects is too high for most patients with
schizophrenia.

3. CT heard that there is a large amount of monitoring with clozapine for a


possible blood adverse reaction. When discussing monitoring with CT, which
of the following is appropriate to include?

A. Severe blood disorder (agranulocytosis) occurs in about 5% of


patients on clozapine.
B. There is also an increased risk of seizures at doses above 300 mg
daily
C. All patients on clozapine must be registered in a national
database, to prevent people who have had the blood problem
(agranulocytosis) from taking clozapine again
D. The risk of this problem is the highest in the first year of
treatment

4. CT decides to take clozapine therapy. He wants to know if there are any


other options if clozapine doesn’t work. Which of the following could be
considered to add to his clozapine if it was not effective?
A. Lithium
B. Risperidone
C. Fluoxetine
D. Lamotrigine
Case 7

Mr. John is a 69-year-old man who has been coming to your pharmacy for a
number of years to pick up his prescriptions for hypertension and
dyslipidemia. He is currently taking hydrochlorothiazide 25 mg once daily
and atorvastatin 20 mg once daily. He mentioned to you that he is
experiencing frequent urination and that the problem has been getting
gradually more noticeable over the past 5 years, but he chose to ignore it. He
is afraid that he might have prostate cancer. You recommended that he should
visit his physician for evaluation.

1. Which of the following statements about BPH (benign prostate


hyperplasia) and prostate cancer are correct?

A. PSA (prostate specific antigen) is a biomarker of prostate cancer


and diagnostic tool of BPH
B. With symptoms of frequency and reduced volume, the chances
of being diagnosed with BPH or prostate cancer are about even.
C. An enlarged prostate indicates that prostate cancer is a more
likely diagnosis than BPH.
D. All of the above

2. After you try to convince Mr. Smith to see his doctor, he agrees to
complete an IPSS questionnaire. His score is 15. How would you interpret
this?

A. The score indicates little, if any, prostate symptoms


B. The score indicates mild prostate symptoms
C. The score indicates moderate prostate symptoms
D. The score indicates severe prostate symptoms
3. Mr Smith is back 3 weeks later and thanks you for convincing him to see
his doctor. He has had assessments conducted and found out his PSA
(prostate specific antigen) is 1.8 ng/mL. He is also very happy that the doctor
has ruled out prostate cancer. How should the PSA result be interpreted?

A. He should be treated with therapy that addresses disease


progression ± symptoms
B. He should be treated with therapy to address disease progression
only
C. He does not need to be treated with pharmacotherapy, but
watchful waiting is indicated
D. He should be treated for symptoms only

4. Which of the following drugs would be the least appropriate to recommend


for replacement of Mr Smith’s hydrochlorothiazide prescription?

A. Irbesartan
B. Metoprolol
C. Aliskiren
D. Verapamil
Case 8

JC is a 57-year-old man who comes into the pharmacy with a prescription for
finasteride and tamsulosin. He tells you that these are the first chronic
prescriptions he has taken his entire life. He has the occasional back problem
and takes ibuprofen 400 mg q4h prn for the pain.

1. Which of the following statements about these two drugs is correct?

A. Finasteride will help control symptoms within 1–2 weeks


B. Tamsulosin needs to be titrated in order to prevent hypotension-
related side effects
C. Finasteride improves prostate volume but not urine retention
issues
D. Tamsulosin therapy does not improve prostate volume

2. After 7 months of treatment JC is doing well, but his doctor wants him to
stop taking the tamsulosin to see if he continues to do well. What is the
rationale behind this?

A. Tamsulosin toxicity may occur after 6 months due to storage of


a drug metabolite in the liver.
B. Tamsulosin may not be required after 6 months because
finasteride also controls symptoms.
C. Tamsulosin often cures symptoms of BPH after being taken for
6 months.
D. Tamsulosin requires that the patient take a one-month “drug
holiday” due to tolerance issues.

3. JC tells you that he has a friend who takes dutasteride instead of


finasteride. He would like to know what the differences are. Which of the
following statements is true?

A. Dutasteride is more effective than finasteride due to inhibition of


type I and type II 5-alpha reductase receptor types.
B. Finasteride has a higher drug interaction risk than dutasteride.
C. Both drugs decrease PSA screening values by approximately
50%.
D. Dutasteride should be taken with food, while finasteride can be
taken without regard to food.

4. JC said he tried taking saw palmetto on his own before going to the doctor.
Which of the following statements about saw palmetto is true?

A. Randomized controlled studies have demonstrated benefit for


treatment of symptoms but not disease progression.
B. Randomized controlled studies have demonstrated benefit for
treatment of disease progression but not symptoms.
C. Randomized controlled studies have demonstrated benefit for
both treatment of symptoms and disease progression.
D. Randomized controlled studies have demonstrated benefit for
neither treatment of symptoms nor disease progression.

5. JC tells you that another friend of his is taking finasteride and alfuzosin
instead of finasteride and tamsulosin. Which of the following is the least
correct statement concerning alfuzosin and tamsulosin?

A. Alfuzosin has as shorter duration of action than tamsulosin


B. Tamsulosin has less drug related interactions
C. Alfuzosin requires less time to reach clinical effectiveness than
tamsulosin
D. Both tamsulosin and alfuzosin are both selective alpha 1 adrenergic
receptor antagonists

6. After 3 years, JC has been recommended for surgery. Which of the


following statements about surgery is true?

A. TUIP is a less expensive procedure than TURP.


B. Transurethral microwave therapy is most appropriate for
patients with a large prostate.
C. TURP is associated with a low incidence of retrograde
ejaculation (< 10%)
D. HOLEP is preferred in patients with small prostate size
Case 9

You are a newly hired pharmacist. Ms Ryan is a 69-year-old female with


chronic obstructive pulmonary disease. She is one of the pharmacy regular
patients. On review of the patient’s profile you realize she has never received
a pneumococcal vaccine.

1. State the drug-related problem as you would best communicate it to the


physician:

A. Please consider providing this patient with a prescription for


pneumococcal vaccine.
B. The patient has chronic obstructive pulmonary disease and
requires a pneumococcal vaccine.
C. The patient requires a pneumococcal vaccination as soon as
possible.
D. The patient is at risk of contracting pneumonia and therefore
requires a pneumococcal vaccine.

2. What might be the shared goal of therapy for this patient in this case?

A. Cure the disease


B. Normalize laboratory values
C. Prevent a disease
D. Reduce symptoms
Case 10

HR is a 24-year-old student. She is visiting your pharmacy to discuss her


headaches. She explains says that her headaches are usually worse around
exam time. Ibuprofen used to work, but now she finds that even her usual
dose of one 400 mg tablet does not provide much relief. Mary says that her
pain throbs on the right side of her head.

1. Which statement about migraine headaches is TRUE?

A. About three times as many women as men are diagnosed with


migraine headaches.
B. Confirmed diagnosis of migraine headache can be made by the
pharmacist.
C. Migraine symptoms are generally consistent from one individual
to another.
D. Migraine headaches are commonly characterized by bilateral
throbbing pain aggravated by physical activity
2. Which quality regarding HR’s headache pain would be least indicative of
migraine headache symptoms?

A. Pain on one side of the head


B. Pain worsening with exposure to light
C. Pain lasting about 30 to 60 minutes without treatment
D. Pain that is accompanied by nausea and vomiting

3. In what percentage of migraine sufferers do aura symptoms occur?

A. 10%
B. 30%
C. 50%
D. 70%

4. What does “scotomata” refer to?

A. Bright flashes of light within the visual field


B. Seeing many colours in the visual field
C. Seeing floating objects in the visual field
D. A perception of a blind or dark spot in the visual field.

5. If HR scores 4 on the MIDAS questionnaire, at what level of disability is


she as a result of her migraine attacks?
A. Little or no disability
B. Mild
C. Moderate
D. Severe

6. If HR is otherwise healthy, what single dose of acetaminophen would be


the minimum recommended for treatment of acute migraine?

A. 350 mg
B. 450 mg
C. 650 mg
D. 750mg

7. If HR wanted to try magnesium to prevent migraine headache, what dose


would you recommend?

A. 300 mg daily
B. 600 mg daily
C. 900 mm daily
D. 1,200 mg daily

Case 11
Ms John is a 72-year-old woman who smokes 12 cigarettes daily and has had
a recent compression fracture in one of her vertebrae. Ms John does not
suffer from vasomotor symptoms. She hands you a prescription and you
notice that the doctor has written T-score = -2.6. She is currently taking
hydrochlorothiazide 25 mg daily for blood pressure, diltiazem 240 mg CD for
blood pressure and atrial fibrillation control, and warfarin for clot prevention
secondary to atrial fibrillation.

1. Based on her T score and history, which of the following statements best
defines Bessie’s present condition?

A. Mild osteoporosis
B. Postmenopausal osteoporosis
C. Osteoporosis
D. Severe osteoporosis

2. Which of the following diagnostic tools is the gold standard for


osteoporosis risk assessment?

A. Dual energy X-ray absorptiometry


B. Bone formation biochemical markers
C. Bone resorption biochemical markers
D. Quantitative ultrasound

3. How much total elemental calcium and vitamin D supplementation from


all sources would you recommend for this patient?

A. 1,500 mg calcium and 800 IU vitamin D


B. 1,500 mg calcium and 400 IU vitamin D
C. 1,000 mg calcium and 800 IU vitamin D
D. 1,000 mg calcium and 400 IU vitamin D
4. Which of the following mechanisms describes best the biological effect of
smoking on Bone Mass Density (BMD)?

A. Toxic chemicals of smoke break down bone


B. Nicotine activates osteoclasts.
C. Nicotine reduces calcium absorption.
D. Smoking may cause increased metabolism of estrogen.

5. The drug teriparatide is indicated in:

A. Postmenopausal women with severe osteoporosis.


B. Postmenopausal women severe osteoporosis who have failed other
therapies.
C. Men with primary or hypogonadal severe osteoporosis severe
osteoporosis who are intolerant to other therapies.
D. All of the above
Case 12

Mrs. Yong is a 54 years old female patient. Her profile shows a history of
Type 2 diabetes for 4 years. She has no known allergies. Her current
medication is glyburide 10mg daily.

1. Mrs. Yong comes into the pharmacy to pick up some over the counter
medications for her medicine cabinet. Which of the following medications
should NOT be recommended because they may adversely affect her blood
glucose levels?

A. Pseudoephedrine
B. Acetaminophen
C. Diphenhydramine
D. Dextromethorphan

2. What is the most common side effect of glyburide?


A. Lactic acidosis
B. Epigastric fullness
C. Cough
D. Hypoglycemia

3. Mrs. Yong tells you she has a postprandial blood glucose reading of 10
mmol/L. The most appropriate advice is:

A. Within normal range - his diabetes appears well controlled


B. High - refer patient to his physician
C. Low - recommend carrying glucose tablets to prevent hypoglycemia
D. High - suggest switching to insulin

4. She is also concerned about alcohol intake. Your counseling may include
all of the following, EXCEPT:

A. Alcohol may interact with certain oral hypoglycemic agents


B. Alcohol may impair the body's ability to recover from hypoglycemia
C. Alcohol may cause sexual dysfunction
D. Alcohol may aggravate peripheral neuropathy

Case 13

Mr. John is a 55 years old patient admitted in the emergency room with
suspected acute coronary syndrome. His profile shows that he has been using
beclomethasone for 18 months for the relief of asthma.

1. Wrong statement concerning the TIMI score.

A. Comprehensive risk assessment tool in patient with STEMI


B. Comprehensive risk assessment tool in patient with unstable angina
C. Predicts risk of death and recurrence of ischemic events
D. TIMI stands for Thrombolytic In Myocardial Infraction

2. Which of the following factors does not contribute in the calculation of


TIMI score?

A. Older than 65
B. ASA administration within 1 week
C. 2 cardiac risk factors
D. Elevation of CK-MB

3. In case NSTEMI is established, which of the following drugs may be not


be an appropriate treatment option for Mr. John?

A. Heparin
B. ASA
C. Nadolol
D. Eptifibatide

Case 14
PG is a 74 years old female patient with Alzheimer’s disease. PG can no
longer drive; she also requires assistance to take her daily medication and use
the phone.

1. What is the likely FAST rating for this patient?

A. 3
B. 4
C. 5
D. 6

2. Which of the following test is used in the assessment of Alzheimer’s


disease?

A. MMSE
B. CT scan
C. Homocysteine
D. All of the above

3. The toxicity of some Alzheimer’s drugs is increased by CYP2D6 or


CYP3A4 inhibitors such as paroxetine, erythromycin, prednisone and
grapefruit juice and their effectiveness is reduced by CYP2D6 or CYP3A4
inducers such as carbamazepine, phenytoin, smoking and rifampin. Which of
the following Alzheimer’s drugs is affected by such interactions?

A. Galantamine
B. Memantine
C. Rivastigmine
D. All of the above
Case 15

JR is a 60 years old male patient with known sulfonamides allergy. His


medical history includes Deep Vein Thrombosis (DVT) and Urinary Tract
Infection (UTI) – just diagnosed. He is currently on warfarin 1mg daily, INR
2.4. A recent lab test shows a creatinine clearance (CrCl) of 20mL/min. A
culture test shows that the microorganism involved in his UTI is resistance to
ampicillin but sensitive to sulfa, ciprofloxacin, cephalosporins.

1. JR presents a new prescription for ciprofloxacin 500mg bid x 7 days for


recurrent urinary tract infections. You would:

A. Dispense the ciprofloxacin as ordered with a note to monitor INRs


B. Call the physician and recommend decreasing ciprofloxacin dose to
250 mg bid x 7 days with a note to monitor INR
C. Call the physician and recommend sulfamethoxazole/trimethoprim i
bid x 7 days with a note to monitor INR
D. Call the doctor and recommend ampicillin 250mg qid x 7 days with a
note to monitor INR

2. Two weeks later JR complains of heart burn especially at night. JB has


already tried antacids and wonders if there is anything else that would help.
Which of the following drugs would NOT be an appropriate therapeutic
option for this patient?

A. Alginic acid
B. Cimetidine
C. Famotidine
D. Ranitidine

Case 16

Ms Ali is a 40-year-old female patient. She has no known allergies and has
been a smoker for 15 years. Ms Ali’s profile shows that is currently taking
zopiclone 7.5 mg qhs prn. She presents the following a new prescription for
Clarithromycin 500 mg bid x 7 days, Metronidazole 500 mg bid x 7 days and
Bismuth subsalicylate ii tabs qid x 7 days.

1. What is the most likely diagnosis according to this new prescription drug
regimen?

A. Escherichia coli intestinal infection


B. Helicobacter pylori infection induced peptic ulcer
C. Salmonella intestinal infection
D. Intestinal amoebiasis
2. Which of the following recommendations would NOT be part of Ms Ali’s
counseling?

A. Take metronidazole with food or milk to reduce GI effect


B. Bismuth subsalicylate must be taken 2 hours after clarithromycin and
metronidazole
C. Bismuth subsalicylate may make the tongue and stool black
D. Store clarithromycin suspension at 15C to 30C

Case 17

A 38 years old male asthmatic patient who has been on topical glaucoma
therapy for 2 months is still complaining about pain due to elevated ocular
pressure. He has also noticed a change in his eye pigmentation.

1. Which of the following drugs has been prescribed?


A. Carbonic anhydrase inhibitor
B. Beta blocker
C. Prostaglandin analogue
D. Cholinergic agonist

2. Which of the following side effects is NOT related to the drug prescribed?

A. Foreign body sensation


B. Burning
C. Lengthening of eye lashes
D. Decreased iris pigmentation

3. Which of the following is NOT an appropriate next step to enhance this


patient’s therapeutic outcome?

A. Add topical non-selective beta blocker


B. Add topical carbonic anhydrase inhibitor
C. Add topical alpha 2 agonist
D. Add topical cholinergic agent

Case 18
Mr. Singh is a 69 years old male patient. His medical history shows that he
has been diagnosed with Parkinson’s disease 2 years ago. He is also suffering
benign prostatic hypertrophy (BPH) and Type 2 diabetes controlled with diet.
His currently taking levodopa/carbidopa.

1. The patient is complaining about episodes of freezing. What would you


recommend?

A. Switch his levodopa/carbidopa to long-acting formulation


B. Add selegiline to levodopa/carbidopa
C. Add bromocriptine to levodopa/carbidopa
D. Initiate a drug holiday

2. Mr. Singh wants to purchase diphenhydramine as he is having trouble


sleeping at night. You advise him that he should not use diphenhydramine
because of his:

A. Caloric restricted diet


B. Benign prostatic hypertrophy
C. Levodopa therapy
D. Rigidity

3. Mr. Singh is also wondering why he needs carbidopa. Your explanation


would be:

A. Prevents neuropathy
B. Controls symptoms of hypertension
C. Reduces levodopa dose requirement
D. Induces the release of dopamine
4. What is the best advice you can give Mr. Singh regarding how he should
take his levodopa/carbidopa?

A. Crush the tablets and dissolve in soft food or fruit juice


B. Supplement with vitamin B6 100mg po daily
C. Take with a light snack
D. Take with a high protein snack

Case 19

Ms. John is a 75 years old male patient with no known allergies. His medical
history shows: Alzheimer’s disease, Raynaud’s disease,
Hypercholesterolemia. He is currently taking donepezil 5 mg daily,
nifedipine 30 mg daily, ASA 325 mg daily, lorazepam 1 mg qhs prn,
pravastatin 20mg qhs.

1. Identify the mechanism of donepezil

A. Inhibit the breakdown of serotonin in the brain


B. Increase acetylcholine levels in the brain
C. Inhibits the breakdown of norepinephrine
D. Act as a gamma aminobutyric acid agonist

2. Ms John’s nutritionist reports that grapefruit juice or orange juice is her


regular beverage. Grapefruit juice should be avoided because it interacts
with:

A. Donepezil
B. Lorazepam
C. Nifedipine
D. Pravastatin

3. Which recommendation would you suggest to best manage the grapefruit


juice?
A. Space apart the grapefruit juice and the interacting medication by at
least 2 hours
B. Discontinue the interacting medication since risks outweigh the
benefits
C. Monitor for increased levels and response of the interacting medication
D. Discontinue the grapefruit juice

4. One month later Ms. John receives the following prescription for
community acquired pneumonia. Which of the following would be the best
therapeutic regimen for this patient?

A. clarithromycin 500 mg bid x 10 days


B. erythromycin 333 mg tid x 10 days
C. doxycycline 100 mg tid x 10 days
D. sulfamethoxazole/trimethoprim DS i bid x 10 days

Case 20

Ms. Paul is a 51 years old female patient. Her medical history includes
gastroesophageal reflux disease (GERD), atrial fibrillation (AF). She has no
known drug allergies. She is currently taking rabeprazole 10 mg daily,
ramipril 10 mg daily, amiodarone 200 mg daily.

1. Ms Paul is now complaining about allergic rhinitis. Which of the following


symptoms is more characteristic of seasonal allergic rhinitis?

A. Nasal congestion
B. Sore throat
C. Itchy eyes
D. Tearing
2.Ms Paul returns to the pharmacy two weeks later with a prescription for an
intranasal corticosteroid. Which of the following statements would you
include when counseling this patient?

A. Therapy should only be used for three months


B. Beneficial effects require regular usage
C. Discontinue a few days after symptoms resolve
D. Therapy should be initiated at the maximum recommended dose

Case 21

JT is a 50 years old female patient. Her medical history includes: borderline


hypertension controlled well with diet for 3 years and seasonal allergy. She is
currently taking calcium 500 mg bid and vitamin D 1000 IU daily.

1. JT calls you for some information about menopause and hormone


replacement therapy (HRT). Which of the following are benefits of estrogen
therapy?

A. Increases calcium resorption from bone


B. Decreases vasomotor flushing within 4 weeks
C. Decreases frequency of migraine headache
D. Decreases risk of breast cancer

2. JT does not wish to resume her menses. Which regimen is most suitable?

A. Conjugated estrogen 0.625 mg daily & medroxyprogesterone 2.5 mg


daily
B. Estradiol 1 mg daily and medroxyprogesterone 5 mg days on days14-
25 of each month
C. Conjugated estrogen 0.625 mg daily & micronized progesterone 300
mg daily
D. Estradiol 1 mg daily and micronized progesterone 300 mg daily
Case 22

MJ is a 9 years old boy who weighs 33 kg. He has seasonal allergies. MJ has
just been diagnosed with asthma. His mother brings in a prescription for:
ipratropium/salbutamol inhaler 2 puffs before exercising and 2 puffs 3-4
times daily prn shortness of breath.

1. What would be your counselling?

A. Recommend adding theophylline syrup to the ipratropium/salbutamol


therapy
B. Recommend adding a corticosteroid to the ipratropium/salbutamol
therapy
C. Recommend changing the prescription to salbutamol inhaler
D. Dispense the prescription as written and counsel the mother on its
proper use

2. Two weeks later, MJ and his mother return to the pharmacy because he is
frequently waking up at night with shortness of breath. You would advise
MJ’s mother to see the doctor to get a prescription for:

A. Cromoglycate inhaler
B. Salbutamol inhaler
C. Fluticasone inhaler
D. Theophylline syrup
Case 23

DC brings in a prescription for triamcinolone 1 spray in each nostril TID


PRN.

1. You would:

A. call the doctor to change the directions to 1 spray in each nostril three
times daily
B. call the doctor to change the directions to 2 sprays in each nostril once
daily
C. dispense as written and explain that the patient may not see results for
at least 3 weeks
D. dispense as written but explain that the medication should be used
regularly, not prn, to receive the maximum benefit

2. When counseling on the triamcinolone you would include all of the


following
EXCEPT:

A. Blow your nose if necessary, prior to use and avoid blowing your nose
for 15 minutes after use
B. Contact your doctor if you experience severe nose bleeds
C. Contact your doctor if you are experiencing yellow or green discharge
D. Do not take OTC antihistamines while on triamcinolone

Case 24

Ms. Pete is 32 years old female patient with history of hyperthyroidism


(Graves' disease) and migraine headaches. Ms Pete has sulfonamides allergy.
She currently taking propylthiouracil 100 mg tid, propranolol 40 mg bid
acetaminophen/codeine/caffeine (Tylenol No. 2), 1-2 tid prn

1. She comes to your pharmacy after being discharged from the hospital with
the diagnosis of Graves' disease. Which of the following is NOT a symptom
of Graves Disease?

A. Sweating, palpitations
B. Insomnia, palpitations
C. Weight gain, palpitations
D. Constipation, palpitations

2. Which of the following statements concerning propylthiouracil is correct?

A. It decreases the thyroid's ability to synthesize thyroid hormone


B. It is contraindicated in pregnancy
C. It is used to control symptoms until spontaneous remission occurs
D. The most frequent adverse effect is osteoporosis

3. With regard to the use of a beta blocker in the management of


hyperthyroidism, which one of the following is TRUE?

A. Aggravates migraine
B. Controls tremor
C. May cause neutropenia
D. Reduce thyroid sensitivity to TSH

Case 25

Mrs. Gill is a 61 years old female patient who weighs 100 kg. Her medical
history shows: osteoarthritis primarily in legs for 8 years, mild edema in
ankles. She has no known allergies. Mrs. Gill is currently taking
hydrochlorothiazide 25 mg daily for ankle swelling,
acetaminophen/codeine/caffeine (Tylenol No. 2) 1 q4h prn.

1. Mrs. Gill was admitted to hospital with complaint of increased swelling in


ankles and severe pain in her left leg. She was diagnosed with deep vein
thrombosis (DVT). What factor most likely aggravated her condition?

A. Obesity
B. Laxative abuse
C. Gender
D. Caffeine

2. Which of the following drugs would most often be used as initial therapy
for acute treatment of deep vein thrombosis in the hospital?

A. Heparin
B. ASA
C. Streptokinase
D. Warfarin

3. Which of the following drugs would be the least efficient as prophylactic


agent for deep vein thrombosis?

A. Enoxaparin
B. ASA
C. Warfarin
D. Heparin
Case 26

While dispensing HIV/AIDS medications for Mr. Ted, who is married to


your friend, you begin to wonder if your friend is aware of her husband’s
diagnosis.

1. Which of the following would be appropriate steps in handling this


situation?

A. Investigate during your counseling if Mr. Ted has disclosed his


diagnosis to his wife
B. Encourage him to tell his wife because she may be infected then
explain how early diagnosis could affect therapeutic outcome. You will
refrain from telling her.
C. Counsel Mr. Ted on strategies to prevent transmission
D. All of the above

2. Which of the following statements BEST guides you in resolving this


dilemma?

A. A pharmacist is obliged to protect the patient's right of confidentiality,


if the patient is capable of making rational decisions
B. A pharmacist may breach confidentiality when the failure to disclose
information will place other persons in serious danger.
C. The burden is on Mr. Ted. He will probably do the responsible thing
and tell your friend
D. The doctor should ensure the patient understands the importance of
preventing the spread of the disease.

Case 27

P.R. is a 56-year-old woman with type 2 diabetes, hypertension, and


dyslipidemia. Her current medications include metformin, 1 g twice daily;
glyburide, 10 mg twice daily; enteric-coated aspirin, 325 mg daily; lisinopril
20 mg daily; HCTZ, 12.5 mg daily; and lovastatin, 40 mg daily. She
developed an upper respiratory infection (URI) for which she was prescribed
erythromycin, 333 mg three times daily for 10 days.

1. What are possible drug-drug interactions?

A. Lovastatin-Lisinopril
B. Erythromycin-Lisinopril
C. Lovastatin-metformin
D. Erythromycin- Lovastatin
2. What option is best to protect this patient from a clinically significant drug
interaction?

A. Counsel her regarding the drug interaction. Instruct her to stop taking
lovastatin and call her health care provider if she develops any unexplained
muscle aches or weakness, especially if accompanied by fever, fatigue, or
tea-colored urine.
B. Have her hold off on the lovastatin until the course of erythromycin is
finished.
C. Advise changing her prescription to another antibiotic such as amoxicillin
or azithromycin
D. Advise changing lovastatin prescription to either pravastatin or fluvastatin

Case 28

A 33 years old male patient has been diagnosed with mycoplasma


pneumoniae infection.

1. Which of the following classes of antibiotics is NOT commonly used in


the treatment of mycoplasma pneumoniae?
A. Penicillins
B. Macrolides
C. Tetracyclines
D. Fluoroquinolones

2. He receives a prescription for a macrolide. Which of the following


macrolides has the lowest incidence of adverse effects?

A. Azithromycin
B. Erythromycin
C. Clarithromycin
D. Telithromycin

3. All are side effects of macrolides, EXCEPT:

A. Dose-related tinnitus
B. Photosensitivity
C. Cholestatic hepatitis
D. Torsade de pointes

4. What is the standard dosage of clarithromycin XL for the treatment of


mycoplasma pneumoniae infection?

A. 500 mg po once daily


B. 1000 mg po once daily
C. 500 mg po BID
D. 1000 mg po BID

Case 29
JA is presenting with an acute attack of gout. Gout is a condition
associated with either an increased production of uric acid or a decreased
excretion of uric acid. Excess uric acid in the body is converted to uric
acid crystals that are deposited in tissues and joints most commonly in the
big toe.

1. Goals of management of gout in JA are:

A. To provide pain relief


B. To identify secondary causes
C. To prevent recurrence
D. All of the above

2. Which of the following drugs could precipitate gout?

A. Nicotinic acid
B. Levodopa
C. Atenolol
D. Pyrazinamide

3. During your counselling, you would advise JA to:

A. Reduce meat intake


B. Reduce alcohol intake
C. Reduce fluid intake
D. All of the above
Case 30

SP is a 34-year-old female who is planning to travel to Europe for a one-


week holiday. She is currently taking valproic acid 400 mg twice daily as
an antiseizures agent.

1. Before the flight SP should be advised of all the following, EXCEPT:

A. To keep taking her medication at same time


B. To keep well hydrated
C. That the medication may precipitate deep vein thrombosis
D. Sleepiness and arrhythmias are signs of overdose

2. Valproic acid is also used for:

A. Manic episodes associated with bipolar disorder


B. Insomnia
C. Parkinsonism
D. Anxiety
Case 31

PS is a 43-year-old female who is seeking your assistance regarding


medications that could be recommended for the treatment of insomnia.

1. What questions should you as pharmacist ask PS:

A. Whether the problem is related to difficulty in falling asleep or


staying asleep
B. Duration and frequency of occurrence
C. Outcome of previous interventions if any
D. All of the above

1. During your interview, you learned that PS is in the process of


moving to Spain and has been experiencing insomnia for the past
five days. She explains that she has difficulty falling asleep. Which
of the following antihistamines may be dispensed to PS for short-
term relief:

A. Promethazine
B. Cetirizine
C. Loratadine
D. Fexofenadine

Case 32

MZ is a 46-year-old female who is visiting Toronto from Dallas, USA for


5 days. MZ is visiting your pharmacy complaining about vomiting and
diarrhea. Her current medication is gliclazide 150mg bid.

1. Questions should be asked to MZ to obtain information on:


A. Food intake during the last 72 hours
B. Severity of symptoms
C. Incidence of dizziness, blurred vision, sweating and numbness or
tingling of the mouth
D. All of the above

2.MZ should be advised of the following recommendations, EXCEPT:

A. Monitor blood glucose


B. Maintain high fluid intake
C. Increase gliclazide dose to three times daily
D. Maintain the initial dosing of gliclazide

3.Which of the following products could potentiate hypoglycemia in this


patient?

A. Ibuprofen
B. Probiotics
C. Bismuth subsalicylate
D. All of the above

Case 33

PT is an 8-year-old boy who is visiting your pharmacy with his mother.


His mother explains that the young boy has headache, nausea, vomiting
and sunburn.

1. Which of the following would be your primary concern?


A. Urine analysis
B. Otoscopic examination
C. Measurement of body temperature
D. Blood analysis

2. In case the previous investigation turns out negative, you would


conclude that PT is most likely suffering from:

A. Dehydration
B. Fatigue
C. Infection
D. Anemia

3. Based on your conclusion regarding the health status of PT, which of


the following tests would be appropriate to further evaluate this young
patient’s condition?
A. Serum sodium
B. Pulse
C. Behaviour status
D. All of the above

4. Which of the following products would be recommended for PT?


A. Calamine lotion
B. Oral rehydration salts
C. Diclofenac gel
D. All of the above
5.You would also counsel PT’s mother on the use of sunscreen for the
prevention of sunburns. All of the following sunscreens can be used in
children of all ages, EXCEPT:

A. Titanium dioxide
B. Zinc oxide
C. Oxybenzone
D. Magnesium silicate
Case 34

AD is a 46-year-old male who has undergone a nephrectomy. He has been


prescribed morphine intramuscularly on a prn basis post-operatively.
However post-operative pain relief was inadequate. Patient presented with
nausea, vomiting and respiratory depression.

1. Which of the following statements regarding post-operative pain


treatment are correct?

A.It is recognized that pain control is better achieved when the


therapeutic regimen is adjusted for each patient and according to each
situation
B.The desired degree of analgesia should outweigh the occurrence of
side-effects
C.The intravenous route of administration for morphine is a less
satisfactory route
D.All of the above

2. Which of the following opioids analgesic is NOT synthetic?

A. Diacetylmorphine
B. Oxycodone
C. Fentanyl
D. Methadone
Case 35

You are counseling a 23 years old female patient who is about to start
using Accutane for the treatment of severe acne.

1. All of the following statements regarding Accutane therapy are


correct, EXCEPT:

A. All patients must sign a consent form prior to the initiation of


therapy
B. It is recommended that all female patients of child bearing age,
even those with history of infertility, use two reliable methods of
birth control
C. All female patients of child bearing age must have one negative
pregnancy test before using Accutane
D. All female patients of child bearing age must have a monthly
pregnancy test while on Accutane and one month after stopping
Accutane

2.Which of the following information would be appropriate to include


to your counseling?

A. Your acne may seem to get worse during the first few weeks of
treatment
B. In general, the first signs of healing occur after 2 to 3 weeks of
treatment
C. It may take 1 to 2 months before noticing the beneficial effect of
Accutane
D. All of the above

3.You will also counsel this patient to avoid skin smoothing procedures
such as waxing, dermabrasion or laser procedures while on Accutane
and for at least:

A. 2 months after discontinuation


B. 3 months after discontinuation
C. 5 months after discontinuation
D. 6 months after discontinuation

4. Which of the following must be avoided by this patient for at least


one month following the discontinuation of Accutane?

A. Becoming pregnant
B. Breastfeeding
C. Giving blood
D. All of the above

5. Which of the following drugs should not be taken during Accutane


treatment due to high risk of increased cranial pressure?

A. Tetracyclines
B. Statins
C. ACE inhibitors
D. Tricyclic antidepressants
Case 36
PJ is a 52 years old male patient with no known allergies. His medical
history shows hypercholesterolemia and gastric ulcer. He is physically
active, at least three times weekly, and a non-smoker. He is currently
on atorvastatin po 20mg qhs - initiated 3 years ago and cimetidine po
300 mg qid – initiated 15 days ago. He is visiting your pharmacy with a
new prescription for sildenafil po 50 mg for erectile dysfunction.

1.Which of the following guidelines is PJ probably following regarding


the administration of cimetidine?

A. Take with meals


B. Take one hour before or two hours after an antacid
C. Take at bedtime
D. All of the above

2.Which of the following drugs are associated with erectile


dysfunction?

A. Alpha blockers
B. Thiazides
C. Phenothiazines
D. All of the above

3. Which of the following is an appropriate step regarding PJ’s


prescription for sildenafil?

A. Dispense as written and advise the patient to take the drug 30 -60
minutes before sexual activity
B. Dispense as written and advise the patient to avoid taking the
drug after a heavy or high fat meal
C. Dispense as written and advise the patient not to take the drug
within two hours of taking cimetidine
D. Call PJ’s doctor to recommend prescribing a lower dose, 25 mg
instead of 50 mg
4.Which of the following drugs are used in the treatment of premature
ejaculation?

A. Sildenafil
B. Fluoxetine
C. Clomipramine
D. All of the above

5.Testosterone replacement is another treatment for erectile


dysfunction. The duration of action of testosterone patch Androderm
is:

A. 24 hours
B. 36 hours
C. 48 hours
D. 60 hours

6.Priapism which is sustained, and painful erection is one of the side


effects of sildenafil. The risk factors for priapism include:

A. Sickle cell anemia


B. Polycythemia
C. Myelofibrosis
D. All of the above
Case 37

Ms. Peter is a 29 years old female patient with no known allergies. She
has been on combined oral contraceptive Alesse EE 20
ug/levonorgestrel 0.1mg for the past 5 years. She has been recently
diagnosed with active tuberculosis. A sputum test showed that the
strain is non-resistant to isoniazid. She is visiting your pharmacy with a
prescription for isoniazid 300 mg daily, rifampin 500 mg daily and
ethambutol 1 g daily for 2 months then isoniazid 300 mg daily and
rifampin 500 mg daily for 7 months.

1.Which of the following statements concerning rifampin is NOT


accurate?

A. Drug of choice for the treatment of latent and active tuberculosis


B. Active against Mycobacterium tuberculosis, M. leprae and
Neisseria meningitides
C. Semisynthetic derivative of rifamycin B and inhibits RNA
synthesis
D. All of the above

2. Tuberculin Skin Test (TST) used in the diagnosis of tuberculosis is


also called:

A. Coomb’s test
B. LH test
C. Mantoux test
D. Clinitest

3.The frequency and duration of treatment of latent tuberculosis using


isoniazid is:

A. Daily for 4 months


B. Daily for 6 months
C. Daily for 7 months
D. Daily for 9 months

4. To prevent isoniazid induced peripheral neuropathy, you would


counsel Ms. Peter to take:

A. Ascorbic acid 25 – 50 mg daily


B. Folic acid 25 – 50 mg daily
C. Pyridoxine 25- 50 mg daily
D. Thiamine 25 – 50 mg daily

5.Rifampin decreases the effectiveness of oral contraceptives. Which of


the following is NOT an appropriate strategy to manage this
interaction?

A. Recommend an alternate method of contraception


B. Recommend an oral contraceptive with a concentration of ethinyl
estriol > 35 ug
C. Recommend a combination therapy with isoniazid and
ethambutol for 12 months
D. None of the above

6.Which of the following progestins has the lowest estrogenic,


pregestational and androgenic effects?

A. Drospirenone
B. Norgestrel
C. Norgestimate
D. Desogestrel
Case 38

You are a hospital pharmacist. Dr. Neil is seeking your assistance to


prescribe phenytoin to his patient. Dr. Neil is a newly recruited
physician who is not familiar with the drug.

1.You would inform Dr. Neil that phenytoin is contraindicated in:

A. Patients receiving diuretics


B. Patients with dehydration
C. Patients on low-salt diet
D. All of the above

2. You would also inform Dr. Neil that phenytoin is involved in several
drug related interactions. Which of the following drug increases the
renal clearance of phenytoin resulting in the reduction of effectiveness?

A. Theophylline
B. Methyldopa
C. Probenecid
D. Tetracycline

3.Dr. Neil is also concerned about his patient’s recent diabetes


diagnosis. The endocrine adverse effects of phenytoin include:

A. Hyperglycemia
B. Hypothyroidism
C. Hyponatremia
D. All of the above
Case Scenarios Answers

Case 1

1. A
C is the value for women

2. C
As initial treatment for diabetic patients with nephropathy use ACEI or
ARB.

3. D

4. B

5. A
Loop diuretics are recommended for diabetic patients with nephropathy
and SCr > 150 umol/L

Case 2
1. D
Treatment for at least 2 years is required after 2 or more episodes of
depression.

2. C
Moclobemide is reversible and selective MAOA inhibitor. Selegiline
and rasagiline are irreversible selective MAOB inhibitors.

3. C
Five-day washout for switching from moclobemide to any
antidepressant; two-week washout from irreversible MAOIs to any
antidepressant. In other cases, a washout period is not required.

4. D
SSRIs have greater tolerability and ease of dosing.

Case 3

1. A
Their respective contribution follows this order A>B>C=D
Sodium <130 mmol/L is another factor to consider.

2. C
Other treatment options include linezolid and
sulfamethoxazole/trimethoprim. Cloxacillin and cefazolin are used to
treat CAP caused by methicillin-susceptible Staphylococcus aureus.

3. B
With PSI score smaller or equal to 90 treat as outpatient for a minimum
of 5 days. It the PSI score is higher or equal to 91 treat in hospital for
usually a minimum of 10 days.

Case 4

1. B
D2 receptor is primarily linked to schizophrenia therapy.

2. D
Another risk factor is maternal infection during the first and second
trimester. The peak age of onset is 15-25 for men and 25-35 for
women.

3. C
Other positive symptoms include hostility, suspiciousness, excitement
and grandiosity. Decreased speech and antisocial behavior are negative
symptoms. ”Flat Affect” means reduced expression of emotions via
facial expression or voice tone; inability of experience pleasure is also a
negative symptom of schizophrenia.

4. B

5. A
The order from highesto lowest weight increase is: clozapine =
olanzapine > risperidone = quetiapine > ziprasidone

6. A
However, citalopram interacts with many antidepressants such as
TCAs, paroxetine, fluvoxamine and fluoxetine. Health Canada has
recently recommended that citalopram should no longer be used at
doses greater than 40 mg per day due to study results indicating a dose-
dependent potential for QT prolongation.

7. C
Cigarette smoke induces the enzyme 1A2 resulting in decreased levels
of clozapine and olanzapine. IB may need a dose reduction of
olanzapine.

Case 5

1. A
Clarithromycin inhibits the enzyme CYP3A4 which will increase the
bioavailability of risperidone leading to increased adverse reactions.
However, according to the patient’s profile, his prescription for
risperidone was last filled 38 days ago. Correct, azithromycin is not
prone to P450 drug-drug interactions but considering that risperidone
has been last filled 38 days ago there is not concern.

2. D

Case 6

1. D

2. A

3. C
Clozapine causes agranulocytosis in about 0.5–1% of individuals. The
risk is highest in the first 6 months and requires weekly blood tests for
WBC and neutrophils monitoring. After 6 months, blood
tests should be done every 2 weeks. The risk of seizures is dose-
dependent and increased at doses above 600 mg daily.

4. B
Combination therapy is more effective.
Adjunctive medication in schizophrenia refers to the addition of a non-
antipsychotic drug to an antipsychotic. Combination strategies refer to
the combination of two antipsychotics.

Case 7

1. A
PSA can be used to determine prostate volume.

2. C
IPSS stands for International Prostate Symptoms Score. Scoring key:
0–7 = mild; 8–19 = moderate; 20 or more = severe.

3. A
Current consensus recommends that a PSA threshold of ≥ 1.5 ng/mL be
used to identify those symptomatic men who should be assessed for
medical therapy aimed at minimizing disease progression and
symptoms. Men with a PSA less than 1.5 ng/mL with symptoms should
be treated with a goal of minimizing symptoms only.

4. D
Smooth muscle relaxants such as calcium channel blockers may reduce
bladder emptying.

Case 8
1. D

2. B
There are two classes of drugs approved for treatment of BPH in
Canada. Long-term 5-alpha reductase inhibitors, alpha-blockers, and
combinations of the two classes are effective for symptom control,
while only 5-alpha reductase inhibitors as monotherapy or in
combination therapy are able to prevent the progression of BPH.

3. C

4. D

5. C
Both require about one week to reach effectiveness and do not require
titration unlike terazosin and doxazosin.

6. A
TURP stands for transurethral resection of the prostate; TUIP stands for
Transurethral incision of the prostate. Holmium laser enucleation
(HOLEP) is used for larger prostate glands and for patients using
anticoagulation. Greenlight laser or photoselective vaporization
prostatectomy is suitable for most men considering surgical
alternatives, especially those using anticoagulation. Transurethral
microwave therapy (TUMT) involves microwave energy delivered via
a microwave antenna placed in a urethral catheter. It causes deep, rapid
tissue heating, while a cooling system circulates water to protect
adjacent tissue. TUMT is a reasonable treatment choice for the patient
who has moderate symptoms, small-to-moderate gland size, and a
desire to avoid more invasive therapy for potentially less effective
results.

Case 9
1. D

2. C

Case 10

1. A
Migraines are diagnosed in about 7.9% of all Canadians over the age of
12

2. C
Migraine headaches pain could persist between 2 to 72 hours.

3. B
Aura symptoms occur in about 30% of migraine sufferers and most
often precede the headache pain by 10 to 60 minutes.

4. D

5. A
MIDAS questionnaire grading
I Score 0-5 Little or no disability; II Score 6-10 Mild disability; III
Score 11-20 Moderate disability; IV Score 21+ Severe disability

6. C
The maximum single dose is 1,300mg
7. B
Magnesium deficiency is found in 50% of patients affected by migraine
headaches.

Case 11

1. D
Normal BMD T-score between 2.5 and -1.0
Osteopenia (low BMD) T-score between -1.0 and -2.5
Osteoporosis T-score less than -2.5
Severe Osteoporosis T-score lower than -2.5 and evidence of fragility
fracture

2. A

3. A
D is the required intake for women and men 19 to 50 years old.

4. D
Women smokers are likely to reach menopause early.

5. D
Parathyroid hormone derivative (teriparatide) is available in prefilled
pen device. Pen delivers 20 injection mcg teriparatide per dose and
must be discarded after 28 days.

Case 12

1. A
Pseudoephedrine may lead to increased secretion of insulin due to
agonistic effect on the adrenergic system. Acetaminophen interferes
with enzymatic glucose test resulting in false decrease in blood glucose
level.

2. D

3. A

4. C

Case 13

1. A
TIMI score is used in NSTEMI.

2. C
Other contributing factors are ST segment deviation on ECG by 0.5
mm or higher; elevation of troponin, known coronary disease. One
point is assigned to each to calculate the TIMI score. 2 episodes of
angina within 24 hrs is also a factor.

3. C
Mr. John has asthma which is a contraindication for the use beta
blocker in this case.

4. D

Case 14
1. B
PG has mild stage of the disease. FAST stands for Functional
Assessment Staging Tool

2. D
CBC, TSH, electrolytes, kidney function, calcium and blood glucose
could be assessed as well. MMSE stands for Mini Mental State
Examination.

3. A
However, urine alkalizers such as carbonic anhydrase inhibitors may
decrease the clearance of memantine. Donepezil is also affected by
such interaction.

Case 15

1. B

2. B
Cimetidine interacts with warfarin.

Case 16

1. B

2. B

Case 17

1. C
2. D

3. A
Beta blockers are effective in the treatment of glaucoma. However,
they should be avoided in patients with asthma due to the induction of
bronchospasm mediated by the blockage of specifically beta 2 receptor.

Case 18

1. D
Rigidity is a sign of chronic use of levodopa
Tip: High protein diet results in decreased levodopa absorption

2. B

3.C
Carbidopa increases the bioavailability of levodopa.

4. C
A light snack reduced the risk of nausea

Case 19

1. B

2. C
Pravastatin is not metabolized by P450 enzymes therefore it has low
potential of drug-drug interactions
3. D

4. D
Clarithromycin and erythromycin increase the level of nifedipine. The
dosage of doxycycline is 100 mg BID PO 1st day then 100 mg daily

Case 20

1. C

2. B

Case 21

1. B

2. A
Continuous therapy is required, not cyclic.

Case 22

1. C
In children, ipratropium is reserved for severe and acute asthma.
2. C

Case 23

1. B
2 sprays daily to a maximum of 8 sprays daily

2.D
The patient should also avoid the use of decongestant nasal sprays
while on triamcinolone.

Case 24

1. D
Other symptoms include irritability, diarrhea, fatigue, heat intolerance,
hand tremors, weight loss and goiter

2. A
Side effects include difficulty tasting food, hair loss, swelling of the
neck, tingling of hands and feet

3. B

Case 25

1. A

2. A
3. B

Case 26

1. D

2. A

Case 27

1. D
Erythromycin increases blood level of statins

2. C

Case 28

1. A
Penicillins are used for the treatment of Streptococcus pneumoniae

2. A

3. B
Adverse effects are more pronounced with erythromycin

4. B
Use 500 mg po BID for clarithromycin regular release
Case 29
1.D
An acute attack of gout is characterized by a rapid onset of pain,
swelling and inflammation usually affecting the big toe. Pain relief and
prevention of attacks are the primary interventions. Any secondary
causes such as alcohol intake, dietary excess, and drug therapy should
be identified. JA should be advised on lifestyle measures which can be
adopted to reduce incidence of recurrence.

2. C
Other drugs that may cause hyperuricemia are cyclosporine, diuretics,
ethambutol, pyrazinamide, theophylline, and salicylates at low dose.

3. D
JA should be advised on nonpharmacologic strategies to reduce the
incidence of acute attack of gout. He should be advised to keep well
hydrated, reduce alcohol intake and maintain an optimal body weight.
He should be advised to exercise regularly and to follow a healthy diet
low in purines.

Case 30

1. C
SP is taking valproic acid which can be used in all forms of seizures.
Considering that SP is travelling across different time zones she will be
advised to keep taking the medication at the same time of country of
origin to ensure that drug plasma concentrations are not significantly
altered. Since SP will be on holiday for one week she should be advised
to keep taking the drug at the same time of the country of origin even
during the holiday. Good hydration status during the flight will reduce
the incidence deep vein thrombosis. Valproic acid may increase
bleeding time as a side-effect and hence its administration does not
increase risk of deep vein thrombosis.

2. A
Valproic acid is effective in all forms of epilepsy. It is particularly
useful in the treatment of primary generalized seizures, absence and
myoclonic seizures and partial seizures. In addition, it may be used in
the treatment of acute manic episodes of bipolar disorder. Valproic acid
is also used in migraines prophylaxis.

Case 31

1. D
Insomnia may arise as a result of a number of factors such as
environmental (sound, temperature), use of drugs (e.g. stimulants),
psychological (e.g. stress) and disease states (e.g. restless legs
syndrome, sleep apnea). It is also important to determine whether the
patient is suffering from initial insomnia (difficulty falling asleep) or
maintenance insomnia (difficulty staying asleep). The duration of the
condition is another important factor.

2. A
Promethazine is a first-generation antihistamine and a member of the
phenothiazine group. It has antihistamine activity (H1 antagonist) and
prominent sedative effects. Promethazine may be recommended in the
management of insomnia for short-term use. Cetirizine, loratadine and
fexofenadine are second-generation antihistamines which lack highly
sedative properties; cetirizine is more sedating than loratadine which is
considered non-sedating. Antihistamines are intended to be used for
only two to three nights at a time such as when stress, travel or other
disruptions interfere with sleep. Tolerance to the sedative effects of
antihistamines could develop quickly with chronic use. Promethazine
has 2 to 8 hours duration of action.
Case 32

1. D
The symptoms of nausea and vomiting in MZ may indicate
gastroenteritis or a change in diet due to the travel. Considering that
Toronto is not a high-risk area for gastroenteritis, a change in diet is
more likely. Since MZ is diabetic she is should be also asked about
the occurrence of symptoms that may indicate hypoglycemia such as
dizziness, blurred vision, sweating and numbness or tingling of the
mouth.

2. C
A complication that may occur is dehydration and electrolyte
imbalance. Therefore, high fluid intake should be maintained. In
addition, MZ should be advised to monitor her blood glucose levels to
detect early the onset of hypoglycemia. The maximum dose of
gliclazide is 320 mg daily. If her symptoms persist, MZ should refer
to her physician; she is staying for only 5 days.

3. A
Salicylates potentiate hypoglycemia.

Case 33
1. C

2. A
The indicators which point towards this possible clinical diagnosis are
the occurrence of sunburn, headache, and nausea.

3. D

4. D
Diclofenac gel as analgesic. Oral rehydration salts are useful to
counteract dehydration. Calamine lotion has soothing effect.

5. C
Other physical sunscreens that provide both UVA/UVB protection and
used in people of all ages are kaolin, magnesium silicate and ferric
chloride. Chemical sunscreens including oxybenzone (UVA/UVB),
avobenzone (UVA) and padimate (UVB) are not recommended in
children younger than 6 months. Unlike chemical sunscreens, physical
sunscreens provide immediate protection.

Case 34

1. A
The treatment of pain should be started as early as possible since
unrelieved pain may have psychological effect resulting in the
reduction of pain management effectiveness. Opioid analgesics such as
morphine are the first-line of therapy in post-operative analgesia. They
can be used to control most post-operative pain but a balance has to be
achieved between the degree of analgesia required and the occurrence
of side-effects. Nausea, vomiting and respiratory depression are side-
effects associated with the use of opioids including morphine. In the
immediate post-operative period, the intravenous route of
administration is recommended. Oral administration may be convenient
at a later stage.

2. A
Diacetylmorphine is heroin.
Case 35

1. C
All female patients of child bearing age must have two negative
pregnancy tests, one must be done in the laboratory, before using
Accutane. In addition, it is recommended that all patients avoid
excessive exposure to sunlight. If necessary, use at least SPF15
sunscreen

2. D

3. D
Accutane induces dryness of skin, lips, mouth and lining to the nose.
Recommend the use of moisturizer and lip balm.

4.D

5. A

Case 36

1. D
One of the daily doses should be taken at bedtime.

2. D
Other drugs associated with erectile dysfunction are MAOIs, SSRIs and
TCAs
3. D
Cimetidine increases the levels of sildenafil; the interaction can be
managed by providing a lower dose. A heavy or high fat meal reduces
the absorption of tadalafil (Cialis) not sildenafil (Viagra).

4.D
SSRIs and TCAs are also used to manage premature ejaculation.

5. A
Androderm patch comes in 2 strengths: 12.2 mg and 24.3 mg

6. D
Leukemia is another risk factor.

Case 37

1. A
Isoniazid is the drug of choice for the treatment of latent and active
tuberculosis.
Rifampin is the drug of choice for the treatment of active tuberculosis
in polytherapy

2. C
Coomb’s is used to detect antibodies; LH test stands for Luteinizing
Hormone test or fertility test; Clinitest is a copper reduction glucose
test; Quellung is used to detect bacteria capsule using India ink.

3. D
Another optional frequency and duration of treatment of latent
tuberculosis using isoniazid is twice weekly for 9 months. B is the
frequency and duration of treatment of latent tuberculosis using
rifampin when the strain is isoniazid resistant or in patients unable to
tolerate isoniazid.

4. C
Pyridoxine is also called vitamin B6

5. C
As example, Ovral contains Ethinyl Estriol (EE)50ug/ norgestrel 0.25
mg

6. A
Drospirenone is a derivative of aldosterone antagonist.

Case 38

1. D
Other contraindications include severe cardiovascular and kidney
disease, history of ACEIs induced angioedema, severe debilitation,
brain damage, pregnancy, lactation and children younger than 12 years

2. A
The remaining drugs decrease the renal clearance of phenytoin leading
to adverse reactions.

3. D
Bonus Questions

1. An 8-year old child is brought to the hospital emergency room. She


complains of cough, wheezing and shortness of breath. Her mother
explains that her symptoms occur once weekly. Her nighttime
symptoms are less than twice a month. A lung function test shows
FEV1 (forced expiratory volume 1) equals to 83% of normal value.
The physician on duty is seeking the guidance of the hospital
pharmacist to select the best treatment for the patient. Which of the
following drugs would be preferred in this patient?

A. Theophylline
B. Salbutamol
C. Formoterol
D. Montelukast

2. Several clinical trial centres are participating in a study to evaluate


the efficacy of a newly developed cholesterol lowering drug. 1,000
patients have been enrolled in the study; half received the drug and half
received a placebo. Neither the physician in charge of the study nor the
patients are allowed to know the treatment allocation of the two groups.
Which of the following statements best describes the study?

A. Phase I and single blind


B. Phase II and double blind
C. Phase III and double blind
D. Phase IV and single blind

3. A 9-year-old female presents to a community pharmacy shortly after


her birthday with erythema over her earlobes in a symmetric pattern.
Her mother explains that she recently received a pair of earrings from
her older sister as birthday present. Which of the following
hypersensitivity reactions is consistent with the patient’s symptoms?
A. Hypersensitivity Type I
B. Hypersensitivity Type II
C. Hypersensitivity Type III
D. Hypersensitivity Type IV
Case 1: Questions 4 and 5

A 22-year old woman presents with shortness of breath. She explained


that in high school, she occasionally had shortness of breath and would
wheeze after running. She now experiences the same symptoms when
she visits her friend who has a cat. Her symptoms have progressively
worsened over the past year and occur, in average, five times per
week. Lung function test by spirometer shows EFV1 (forced
expiratory volume 1) equals to 83 percent of normal value. She is also
experiencing nighttime coughing and occasional wheezing about four
times a month.

4. Following further assessment, a diagnosis of asthma is made. Which


of the following statements best describes the condition of this patient?

A. Mild intermittent asthma


B. Mild persistent asthma
C. Moderate persistent asthma
D. Severe persistent asthma

5. Which of the following drugs would be the preferred therapeutic


option in this patient?

A. Inhaled fluticasone
B. Nedocromil
C. Zafirlukast
D. Terbutaline

6. You are a hospital pharmacist. A 30-year old female is scheduled for


surgical removal of gallbladder. Her pre-operative vital signs are
normal. After the administration of midazolam, rocuronium and
propofol, she was intubated and volatile anesthesia started. She
suddenly became flushed and tachycardic. Her blood pressure
decreased. Anaphylactic reaction is suspected, and epinephrine was
administered immediately. She was later referred for allergy testing.
Which of the following statements best describes the symptoms
experienced by this patient?

A. Hypersensitivity Type I
B. Hypersensitivity Type II
C. Hypersensitivity Type III
D. Hypersensitivity Type IV

7. A newly hired laboratory technician presents to the emergency room


following accidental exposure to parathion. He is experiencing severe
spastic muscular paralysis, hypotension, bradycardia, vomiting, tearing
and salivation. Atropine is quickly administered by injection. He soon
recovers but continues to experience mild muscular spasms. As a
hospital pharmacist, which of the following substances would you
recommend in conjunction with atropine to help regenerate
acetylcholinesterase (AChE) following parathion poisoning?
A. Donepezil
B. Pralidoxime
C. Neostigmine
D. Oxygen

8. A 38-year-old woman presents to your pharmacy with a new


prescription for thiabendazole. Few days ago, she visits her physician
with classical symptoms of chronic strongyloidiasis: episodic urticaria
affecting buttocks and perianal area, abdominal pain and diarrhea. Her
complete blood count showed eosinophilia and stool microscopy
reveals the presence of strongyloides stercoralis. The patient has:
A. Bacterial infection
B. Viral infection
C. Roundworm infection
D. Fungal infection

9. A 28-year-old HIV-infected man returns to the clinic for a follow-up


appointment. His laboratory results obtained 1, 4 and 7 months prior
showed CD4 counts of 524, 608 and 692 cells/mm3. He has no HIV-
related symptoms and has not had any AIDS-defining illnesses. A
blood test for HLA-B5701 gene is positive. The patient is very
interested in taking antiretroviral therapy and there are no concerns
with therapy compliance. His physician is seeking the guidance of the
hospital pharmacist to select an appropriate medication regimen for the
patient. Which of the following drugs should be avoided in this patient?

A. Enfuvirtide
B. Nevirapine
C. Abacavir
D. Raltegravir

10. A 29-year-old woman is diagnosed with asymptomatic HIV in


pregnancy. Her initial laboratory results show a CD4 count of 482
cells/mm3 and an HIV RNA level of 53,638 copies/ml. She is 11 weeks
pregnant feeling well with minimal nausea. Which of the following
drugs has been shown effective in the reduction of vertical transmission
when administered by IV at time of delivery?

A. Zidovudine
B. Efavirenz
C. Indinavir
D. Didanosine

Case 2: Questions 11 and 12


A 17-year-old high school student visits her physician with chief
complaint of recurrent acne. She explains that she has been bothered by
facial acne for the past 4 years. She describes facial breakouts that
began gradually, varied in severity, worsened during menses, and never
completely cleared. She tried over-the-counter (OTC) benzoyl peroxide
gel and wash, as well as numerous facial cleansing products without
noticeable improvement. Her menstrual cycle is regular.

11. Her physician prescribes a topical antibiotic combined with benzoyl


peroxide. Which of the following antibiotic has been prescribed?

A. Spironolactone
B. Adapalene
C. Doxycycline
D. Clindamycin

12. During your interaction with the patient, she asks why the antibiotic
is combined with benzoyl peroxide. Which of the following would be
an appropriate explanation?

A. To minimize skin irritation


B. To prevent sun sensitivity
C. To reduce the risk of antibiotic resistance
D. To prevent acne recurrence

13. A 6-year-old boy is brought to the pediatrician with complaints of


itching around the anus; he has become restless and is having difficulty
sleeping. The physician ordered a blood exam and pinworm paddle test.
No parasites were reported from blood films. However, the paddle test
findings were consistent with pinworm eggs: 50 to 60 micrometers in
length by 20 to 30 micrometers wide. A diagnosis of enterobiasis
caused by pinworm is made. Which of the following drugs are
recommended for the treatment of enterobiasis?

A. Albendazole, Mebendazole
B. Praziquantel, Albendazole
C. Pyrantel pamoate, Niclosamide
D. Mebendazole, Pyrantel pamoate

14. A 44-year-old male presents to the emergency room with 40ºC


fever and general malaise of one-week duration, and watery stools
since the last three days. He denies alcohol or smoking habits, illicit
drug use or sexual risk contacts. He had returned from a trip to
Guatemala two weeks ago. A blood test reveals positive thick smear for
Plasmodium falciparum with 35% of parasitemia. Which of the
following therapeutic agents could effectively treat gametocytic and
exoerythrocytic forms of the parasite?

A. Artemisinin
B. Mefloquine
C. Primaquine
D. Pyrimethamine

15. A 39-year-old African-Canadian HIV-infected woman presents for


a routine follow-up visit. She started antiretroviral therapy three years
ago with tenofovir-emtricitabine-darunavir-ritonavir. On routine
laboratory testing, her HIV RNA is undetectable and her CD4 count
has improved by 50%. Her physical exam reveals marked
hyperpigmentation of palms and soles. Which of the drugs in her
antiretroviral regimen is most likely responsible for the
hyperpigmentation?

A. Tenofovir
B. Emtricitabine
C. Darunavir
D. Ritonavir

16. Nitrofurantoin is an antibiotic used to treat urinary tract infections


(UTIs). The reduction of nitrofurantoin by bacterial nitrofuran
reductase leads to multiple reactive intermediates that damage bacterial
ribosomal protein, DNA and other biomolecules. Nitrofurantoin is
classified as:

A. Pregnancy category A
B. Pregnancy category B
C. Pregnancy category C
D. Pregnancy category D

17. A 28-year-old nurse sustains a needle stick injury to her gloved


palm while drawing blood from a male patient hospitalized for newly
diagnosed AIDS and Pneumocystis pneumonia (PCP). The needle stick
involved had visible blood on it, and it clearly penetrated the nurse’s
right palm. She has no known medical history and does not take any
medications. She receives immediately the first dose of HIV post
exposure (PEP) prophylaxis. All of the following are drugs of choice
for PEP, EXCEPT:

A. Didanosine
B. Lamivudine
C. Lopinavir
D. Ritonavir

18. A 27-year-old male patient presents to the pharmacy with a new


prescription for 10 million units of interferon alpha three times per
week. He visits his physician a week ago with complains of fever,
fatigue, nausea, abdominal pain, dark urine, joint pain and yellowing of
skin. He is an International Aid Worker who travels extensively to
Africa, Central and Southeast Asia, and Eastern Europe. A diagnosis of
chronic hepatitis B is made following a blood test. Which of the
following adverse reactions would be most likely experienced by the
patient during the course of interferon alpha therapy?

A. Lipodystrophy
B. Agranulocytosis
C. Mazzotti reaction
D. Flu-like syndrome

19. A 25-year-old HIV-positive male patient is brought to the


emergency department. He is incoherent and aggressive. His mother
explains that he has been having increasing episodes of disorientation
over the past few weeks. She also informed the attending physician
that her son has been unable to afford the prescribed antiretroviral
drugs due to recent job loss. Routine blood testing reveals CD4 count
of 150/mm3 and brain CT scan reveals ring-enhancing lesions. He was
diagnosed with toxoplasma encephalitis. The physician on duty is
seeking the advice of the hospital pharmacist to select an appropriate
treatment. Which of the following drugs is a recommended treatment
for toxoplasma encephalitis?

A. Proguanil + sulfadiazine
B. Suramin + sulfadiazine
C. Metronidazole + sulfadiazine
D. Pyrimethamine + sulfadiazine

20. A 16-year-old boy presents to the pharmacy with a rash on his back,
both forearms, and shoulders about 24 hours following a hot air balloon
ride over the Grand Canyon without wearing protective clothing or
sunscreen. You refer him to the clinic for proper management and
recommend the application of sunscreen prior to sun exposure to
prevent sunburns. Which of the following sunscreen filters is well
tolerated, blocks UV rays and provides immediate protection against
UVA and UVB rays following application?

A. Oxybenzone
B. Azobenzone
C. Zinc oxide
D. Homosalate

Case 3: Questions 21 and 22


A type 2 diabetic female patient on insulin notices a marked early
morning increase in her blood glucose. Her physician recommends
night time blood glucose monitoring between 3 am – 5am for 3 nights.
The results do not reveal nocturnal hypoglycemia.
21. Which of the following statements is accurate concerning the
condition this patient is experiencing?
A. Somogyi effect also called post-hypoglycemia hyperglycemia
B. Reduced release of growth hormone, epinephrine and glucagon
C. Dawn Phenomenon due to wearing-off of administered insulin
D. Up-regulation of insulin receptors due to chronic hyperglycemia

22. All of the following strategies could help manage her condition,
EXCEPT:

A. Avoid carbohydrates at bedtime


B. Reduce the dose of insulin
C. Delay the administration time of insulin
D. Switch to a different type of insulin

23. You are a hospital pharmacist. An overweight patient presents with


a constellation of symptoms including moon face, buffalo hump and
purple stretch marks on the abdomen, thighs, breasts and arms.
Following further assessment, a diagnosis of Cushing’s disease caused
by pituitary adenoma is made. He is scheduled for X-radiation of the
pituitary gland. It usually takes several months for adequate
symptomatic and metabolic improvement following radiation
treatment. Which of the following therapeutics would you recommend
adjunctively to induce fast suppression of cortisol synthesis?

A. Vasopressin
B. Fludrocortisone
C. Ketoconazole
D. Spironolactone

Case 4: Questions 24 and 25

A 38-year-old female patient visits her physician with chief complaint


of nervousness. She further explains that she also suffers from heat
intolerance, palpitations, increased appetite and insomnia. In addition,
she has lost at least 7 lbs in the past five weeks. A diagnosis of
hyperthyroidism is made following blood test. The physician
recommends an initial trial of an agent known to interfere with the
biosynthesis of thyroid hormones by preventing the iodination and
proteolytic release of thyroid hormones.
24. Which of the following products has been recommended?

A. Methimazole
B. Thiopropyluracil
C. SSKI (saturated solution of potassium iodine)
D. Propranolol

25. Unfortunately, the initial therapy has not been effective in this
patient. The physician decides to switch the patient to methimazole.
The most concerning adverse reaction during methimazole therapy is:

A. Dizziness
B. Edema
C. Stomach upset
D. Agranulocytosis

26. A 57-year-old male visit your pharmacy with a new prescription for
the management of Diabetes Insipidus (DI). He was recently referred to
a specialist for the evaluation of polyuria which he had been
experiencing for at least one year. As a result, he is also experiencing
excessive thirst. His medical history is unremarkable except for mild
hyperlipidemia which is managed with diet and exercise. He was
diagnosed with Diabetes Insipidus following blood electrolyte test and
urinalysis. Which of the following drugs has been prescribed?

A. Pergolide
B. Desmopressin
C. Cosyntropin
D. Follitropin

27-year-old Hispanic-Canadian woman, 5’4’’ 192 lbs BMI 33, presents


to the clinic for her annual checkup. She had gestational diabetes
during her last pregnancy five years ago. No other significant history.
Her blood test results show: FPG 12.5 mmol/L (normal 4.0 to 7.0
mmol/L); glycated hemoglobin (HbA1c) 8.5% (normal 4-6%);
microalbuminuria is negative. Her physician recommends initiating
drug therapy combined with lifestyle changes. She receives a
prescription for an antihyperglycemic in the class of sodium-glucose
co-transporter 2 (SGLT2) inhibitors. Which of the following
antihyperglycemics has been prescribed?

A. Empagliflozin
B. Exenatide
C. Sitagliptin
D. Rosiglitazone

28. Which of the following statements would you include in your


counselling to reduce a leading adverse reaction associated with
empagliflozin therapy?

A. Avoid sun exposure


B. Maintain adequate hydration
C. Avoid spicy foods
D. Reduce sodium intake

Case 5: Questions 29 and 30

A 50-year-old diabetic patient on drug therapy visits the clinic for a


routine check-up. His laboratory results show: blood pH 7.1 (normal:
7.35 – 7.45); pO2 82 mmHg (normal: 75 - 100 mmHg); pCO2 = 29
mmHg (normal: 33 – 44 mmHg); NaHCO3 14 mEq/L (normal: 22- 28
mEq/L).

29. Based on the laboratory results, which of the following drugs is the
patient taking?
A. Pioglitazone
B. Insulin
C. Acarbose
D. Metformin

30. Which of the following statements is incorrect regarding the drug in


question 29?

A. Adverse effects include weight loss


B. Chronic use increases the risk of vitamin B12 deficiency
C. Inhibits gluconeogenesis
D. Hypoglycemia is a major adverse effect

31. A 37 years old patient visits your pharmacy with a new prescription
for a combined oral contraceptive (COC). During your interaction she
mentioned that she has been on sumatriptan nasal spray prn for almost
a year for the treatment of migraines. Which of the following
contraceptive would you recommend to this patient?

A. Combined estrogen/progestin monophasic


B. Combined estrogen/progestin biphasic
C. Combined estrogen/progestin triphasic
D. Progestin only

32. A 55-year-old patient with Type 2 diabetes mellitus is non-


compliant with medication and diet recommendations. The simplest
and informative way to assess for past non-compliance and long-term
glycemic control would be to perform which of the following tests?
A. Fasting blood glucose test
B. Glucose tolerance test
C. Glycated hemoglobin (HbA1C) test
D. Urine ketone test

33. A 56-year-old obese female patient complains of fatigue, thirst and


excessive frequency of urination. She has a history of hypertension,
hyperlipidemia and sulfa allergy. Her current therapeutic regimen
includes losartan and atorvastatin. Type 2 diabetes is suspected, and a
blood test is ordered. The average of two fasting plasma glucose (FPG)
measurements was 12.3mmol/L. She was diagnosed with metabolic
syndrome. Which of the following sets of results defines metabolic
syndrome in a female patient?

A. Waist circumference 99 cm; HDL 0.88 mmol/L; BP 134/90


mmHg
B. Waist circumference 100 cm; HDL 1.2 mmol/L; BP 129/82
mmHg
C. Waist circumference 80 cm; HDL 0.9 mmol/L; BP 130/80 mmHg
D. Waist circumference 78 cm; HDL 1.0 mmol/L; BP 127/83 mmHg

34. A 39-year-old patient is undergoing assisted reproductive


technology (ART) to improve her chances of conceiving. Her
therapeutic regimen includes a combination of luteinizing hormone
(LH) and follicle stimulating hormone (FSH) extracted from the urine
of menopausal women. Which of the following therapeutic agent was
administered?

A. Oxandrolone
B. Menotropin
C. Liotrix
D. Estrone

35. A 29-year-old woman, presents with a 2-year history of debilitating


migraine headaches with a pain scale rating of 10 out of 10 (10 being
the highest level of pain). The pain is unilateral and occurs, in average,
17 days each month and last for 5 hours. During the attacks, she also
experiences photophobia, hypersensitivity to sounds, distorted vision
with visible auras, dizziness and occasional nausea and vomiting. Her
symptoms are not effectively relieved with abortive drugs. Her
physician initiates a trial of a prophylactic agent that interferes with the
release of acetylcholine. Which of the following drugs has been
prescribed?

A. Onabotulinum toxin A
B. Verapamil
C. Propranolol
D. Topiramate

36. A 52-year-old male teacher is being evaluated for recurrent


thromboembolism. An episode of pulmonary embolus three months
earlier has been followed by deep vein thrombosis a week later. Despite
apparent adequate anticoagulant therapy with warfarin 10 mg daily his
INR remains below normal range. His current therapeutic regimen
includes lisinopril for high blood pressure, cholestyramine for
hyperlipidemia, timolol eye drops to reduce intraocular pressure in the
right eye, acetaminophen prn 500mg for back pain. Which of the drugs
in the patient’s therapeutic regimen is likely to interact with warfarin
resulting in reduced anticoagulation effect?

A. Lisinopril
B. Cholestyramine
C. Timolol
D. Acetaminophen
37. A 68-year-old retired lawyer initially complained of frequency of
micturition, urinary urgency, and hesitancy associated with a weak
stream. Over the past months, he has reported a few episodes of
hematuria and incontinence. In addition to his urologic symptoms, the
patient complained of low-grade, constant back pain and bouts of
constipation. A digital rectal examination revealed enlarged prostate
gland with several palpably discreet nodules. His prostate-specific
antigen (PSA) level is 25ug/L (range: 0.0–4.0 ug/L) up from 12ug/L six
months ago. Histologic evaluation of the biopsy specimens revealed
adenocarcinoma representing 55% of biopsied material confirming a
diagnosis of prostate cancer. An antagonist of androgen receptor was
prescribed. Which of the following drugs has been prescribed?

A. Anastrozole
B. Megestrol
C. Leuprolide
D. Flutamide

Case 6: Questions 38 and 39

A patient is being treated with combination chemotherapy including


carmustine, etoposide, bortezomib and prednisone for multiple
myeloma. After three weeks of treatment, the patient presents to the
emergency department with a stripe of red blisters that wraps around
the left of his torso. He explains that the rash was preceded by pain,
burning and sensitivity to touch. His physician is seeking the guidance
the hospital pharmacist to identify the offending drug.

38. Which of the drugs in the patient's drug regimen is the most likely
offending drug?

A. Carmustine
B. Etoposide
C. Bortezomib
D. Prednisone

39. Which of the following agents can be used prophylactically to


reduce the incidence of the adverse reaction experienced by the patient?

A. Ketoconazole
B. Prednisone
C. Acyclovir
D. Naproxen

40. A 53-year-old man presents with changes in bowel frequency and


pencil-thin stools with occasional bright red blood. Following further
assessment, a diagnosis of colon cancer was made. Treatment is
initiated with leucovorin, 5-fluorouracil and oxaliplatin. Which of the
following statements describes best the rational use of leucovorin in the
therapeutic regimen?

A. Prevents myelosuppression associated with oxaliplatin therapy


B. Acts as 5-fluorouracil antagonist to protect normal cells
C. Enhances the potency of 5-fluorouracil
D. Prevents oxaliplatin-induced emesis

41. A 56-year-old female patient with recurring gastric ulcer and


chronic atrophic gastritis for 10 years presents with chief complaint of
fatigue. Physical exam reveals pale skin, glossitis and dyspnea. Her
laboratory results show Hemoglobin 100 g/L (N: 120 -160 g/L), MCV
119 femtoliter (N: 76 -100 femtoliter) and RBC 3.4 (N: 4.2 – 5.4 x
1012/L). Which of the following treatment options would be appropriate
for this patient?

A. Ferumoxytol
B. Filgrastim
C. Iron dextran
D. Cyanocobalamin

42. A 66-year-old man with a past medical history of atrial fibrillation


(AF), hypertension, and type 2 diabetes mellitus has been on
anticoagulation therapy with warfarin for 6 years. His recent
international normalized ratio (INR) is 2.5 within normal range. He has
had no bleeding or thrombotic complications. His current medications
include warfarin, metformin, lisinopril, and atorvastatin. The patient
has recently learned about dabigatran. He would like to know if he
should be switched from warfarin to dabigatran. Which of the
following statements best describes the benefit of switching the patient
from warfarin to dabigatran?

A. Overdose of dabigatran is effectively managed by the


administration of vitamin K
B. Oral administration of dabigatran is more patient friendly
compared to warfarin
C. Monitoring is not required during dabigatran therapy
D. Dabigatran is useful in the management of thrombocytopenia-
induced by warfarin

43. An otherwise healthy 61-year-old man receives an induction


anesthetic for an endoscopy for the diagnosis of upper GI bleeding. The
administration of the anesthetic was followed by fast loss of
consciousness and suppression of cough and gag reflexes. Which of the
following induction anesthetics was administered to the patient?

A. Thiopental
B. Etomidate
C. Propofol
D. Ketamine
44. An 8-year-old boy who fell off his bike has a laceration on his right
knee that requires sutures. Lidocaine hydrochloride as a 2% solution
with epinephrine 1:100,000 is used for local infiltration anesthesia.
What is the purpose of the inclusion of epinephrine?

A. To produce vasoconstriction and increase the duration of action


of the local anesthetic
B. To produce vasodilation and increase the absorption of the local
anesthetic into systemic circulation
C. To decrease, selectively, the sensitivity of C fibers to the local
anesthetic
D. To decrease the sensitivity of sympathetic nerve endings to the
local anesthetic

45. Ramucirumab belongs to the class of targeted therapies in the


treatment of cancer. Ramucirumab is typically combined with
paclitaxel. All of the following statements concerning ramucirumab are
accurate, EXCEPT:

A. Is a promoter of angiogenesis
B. Acts as vascular endothelial growth factor receptor (VEGFR)
antagonist
C. Approved for the treatment of gastric cancer
D. Is a monoclonal antibody

46. A 7-year-old boy is scheduled for a procedure to address a


mandibular fracture sustained after a fall. He has no history of
anesthetic complications. An inhaled anesthetic was administered.
Halfway through the procedure symptoms of hyperthermia including
muscle rigidity, tachypnea and tachycardia were noted. Which of the
following agents could be administered to help manage the symptoms
of the patient?

A. Thiopental
B. Lidocaine
C. Ketamine
D. Dantrolene

47. A 62-year-old lady presents to the clinic with swollen, red and itchy
upper lip. About 24 hours ago, she had seen her dentist for root canal
treatment. On the day of her dental procedure, a local anesthetic gel
was applied to her upper gum to reduce injection pain prior to receiving
an injectable local anesthetic. An extirpation was performed, and the
root canal was medicated with sodium hypochlorite and sealed with a
temporary dressing. According to the patient, her lip began to swell
steadily during the next 8 hours to the point that she decided to seek
medical help. She has previously received the same injectable local
anesthetic without any reaction. Therefore, an allergic reaction to the
local anesthetic gel was suspected. Which of the following local
anesthetics was most likely applied to her gum prior to receiving an
injectable anesthetic?

A. Mepivacaine
B. Benzocaine
C. Lidocaine
D. Bupivacaine

48. 43-year-old female patient diagnosed with nephrolithiasis is


referred for endoscopic ureterolithotripsy. Midazolam, propofol and
sevoflurane were administered to achieve balanced general anesthesia.
Which of the following is the most reliable sign that stage III surgical
anesthesia has been reached?

A. Non-rapid eye movement


B. Dilation of pupils
C. Severe hypotension
D. Loss of consciousness

49. Midwives offer primary care to healthy pregnant women and


normal newborn babies from early pregnancy through birth and
postpartum period. According to new regulations, which of the
following three drug categories midwives may autonomously prescribe,
compound, dispense or administer for specific indications?

A. Anti-coagulants for prophylaxis or treatment of thrombosis


B. Benzodiazepines for therapeutic relief in prodromal labour or
short-term management of excessive anxiety in postpartum
period
C. Narcotics for pain relief in labour or during postpartum period
D. All of the above

50. Cold chain refers to the process used to maintain optimal


conditions, particularly temperature, during the transport, storage and
handling of vaccines. Which of the following statements concerning the
cold chain is inaccurate?

A. Cold chain begins at the manufacturer and ends with proper


storage at the pharmacy
B. Monitoring of vaccines' cold chain is required to ensure potency
is maintained
C. Exposure of a vaccine to environmental conditions not
recommended for the product is called a cold chain break
D. Trained personnel is a key element of an effective cold chain

51. A pharmacist is about to administer a vaccine. He is preparing a


checklist to assess and determine the preferred route of administration.
The preferred route of administration must be:

A. Appropriate for the vaccine being administered


B. Appropriate for the client
C. Appropriate for the pharmacy setting
D. All of the above

52. Which of the following post-vaccination reactions monitoring


requires on-site observation?

A. Syncope
B. Pain
C. Arthus reaction
D. Seizures
Answers
1. B
Since the symptoms occur at low frequency, a quick-relief drug would
be sufficient. Quick relief medications for asthma are short-acting
bronchodilators. Examples of short-acting bronchodilators: Salbutamol,
fenoterol and terbutaline. Note: Health Canada has approved
Symbicort, a combination corticosteroid (budesonide) and long-acting
bronchodilator (formoterol) as rescue agent. Symbicort has been used
previously as long-term control agent for prevention. Symbicort is now
used for both prevention and rescue.

2. C
The size of the study, 1000 patients, indicates a phase III study. In
addition, the study is double blind since the physician in charge and the
patients are not aware of treatment allocation.

3. D
The symptoms are consistent with allergic contact dermatitis (ACD), an
allergic response caused by contact with an allergenic substance. ACD
is a type 4 hypersensitivity reaction also called delayed hypersensitivity
reaction because it takes few days to develop. Unlike other types of
hypersensitivities, it is not antibody-mediated but rather cell-mediated.

4. B

5. A
Corticosteroids are generally considered the most effective treatment
available for long-term control.

6. A
Type I hypersensitivity reaction called immediate hypersensitivity
reaction involves immunoglobulin E (IgE)–mediated release of
histamine and other mediators from mast cells and basophils. Examples
include anaphylaxis and allergic rhinoconjunctivitis.

7. B
Pralidoxime is typically used to treat organophosphate (e.g. parathion,
malathion) poisoning. Organophosphates act by binding to the hydroxy
group in the active site of the acetylcholinesterase (AChE), thereby
blocking its activity. Pralidoxime binds to the unblocked active site of
the enzyme to induce conformational change resulting in the
displacement of the poison; this action of pralidoxime helps regenerate
AChE.

8. C

9. C
Severe hypersensitivity reaction to abacavir is strongly associated with
HLA-B5701 gene. Patient who tested positive for HLA-B7501 have
significant risk for abacavir hypersensitivity syndrome.

10. A
Zidovudine is a nucleoside reverse transcriptase inhibitor (NRTI).
11. D
Clindamycin and erythromycin are antibiotics used topically combined
with benzoyl peroxide. Adapalene and tazarotene are topical retinoids.
Doxycycline and minocycline are tetracyclines used orally in the
treatment of severe acne. Spironolactone is an antiandrogen used orally
to treat severe acne.

12. C
Benzoyl peroxide has a bactericidal effect on Propionibacterium acnes
bacteria associated with acne and does not induce antibiotic resistance.

13. D
The most reliable method for diagnosing pinworm infection is the use
of pinworm paddle; they are small spatulas with adhesive material on
one side. The sticky side is applied to the perianal region of the patient
early in the morning before bathing or defecating. The spatula is
examined, adhesive side up, microscopically for eggs.

14. C
Common side effects of primaquine include nausea, vomiting, and
stomach cramps.

15. B
Emtricitabine is a nucleoside reverse transcriptase inhibitor (NRTI) for
the prevention and treatment of HIV infection in adults and children.
In the general population, the most common adverse reactions are
diarrhea, headache, nausea, and rash. Skin hyperpigmentation affecting
either the palms of the hands and/or the soles of the feet is seen almost
exclusive in patients of African origin.

16. B
17. A
PEP must be started within 72 hours after a recent possible exposure to
HIV, but the sooner the better. Every hour counts. The duration of
therapy is 28 days. PEP is effective, but not 100%. Kaletra (lopinavir
and ritonavir) and Combivir (zidovudine and lamivudine).

18. D
Symptoms of Flu-Like Syndrome include:
Fever
Chills
Myalgias/arthralgias
Headache
Poor appetite
Nausea, vomiting
Diarrhea
Nasal stuffiness
Cough
Bone pain
Fatigue

19. D
Toxoplasmia encephalitis (TE) is caused by the protozoan Toxoplasma
gondii. Clinical disease is rare among patients with CD4 T lymphocyte
(CD4) cell counts >200 cells/µL. Leucovorin can be added to reduce
the likelihood of hematologic toxicities associated with pyrimethamine
therapy.

20. C
Zinc oxide is a physical sunscreen and protects against the entire
spectrum of UVA and UVB rays.

21. C
Nocturnal hypoglycemia followed by increased blood glucose is a sign
of Somogyi effect.

22. B
Instead, the dose of insulin should be increased or switch to longer
acting insulin.

23. C
While ketoconazole blocks the synthesis of ergosterol in fungi, it is also
a potent inhibitor of several enzymes necessary for the conversion of
cholesterol to steroid hormones such as testosterone and cortisol. Based
on its antiglucocorticoid effect, ketoconazole is used as an adjunct
agent for the suppression of glucocorticoid synthesis in the treatment of
Cushing's syndrome.

24. C

25. D
Agranulocytosis is an acute condition involving severe reduction of
white blood cells

26. B

27. A
Adverse reactions of empagliflozin: genital infections, UTIs,
hypotension associated with dehydration, dose related LDL changes

28. B

29. D
The results show signs of lactic acidosis. Metformin is known to induce
lactic acidosis. Metformin is a biguanide antihyperglycemic agent used
for treating non-insulin-dependent diabetes mellitus (NIDDM). It
improves glycemic control by decreasing hepatic glucose production,
decreasing glucose absorption and increasing insulin-mediated glucose
uptake. Metformin may induce weight loss and is the drug of choice for
obese NIDDM patients.

30. D
Metformin does not cause hypoglycemia.

31. D
Contraindications to COC: cardiovascular diseases, liver disease,
gallbladder disease, smoking, breast cancer, advanced diabetes,
migraines, less than 6 weeks postpartum and breastfeeding

32. C
HbA1C test identifies the three-month average plasma glucose
concentration. The test is limited to a three-month average because the
lifespan of a red blood cell is 120 days.

33. A
Metabolic syndrome (MetS) is a health disorder that, left untreated,
greatly increases the risk of many chronic illnesses. MetS is diagnosed
when a patient has three of the following conditions:
High blood pressure (≥ 130/85 mm Hg, or receiving medication)
High blood glucose levels (≥ 5.6 mmol/L, or receiving
medication)
High triglycerides (≥ 1.7 mmol/L, or receiving medication)
Low HDL-Cholesterol (< 1.0 mmol/L in men or < 1.3 mmol/L in
women)
Large waist circumference (≥ 102 cm in men, ≥ 88 cm in women)
34. B

35. A
Onabotulinum toxin A (botox) is also used to treat hyperhidrosis.

36. B
Cholestyramine reduces INR by decreasing the absorption of warfarin.
Monitor INR more frequently when starting or stopping cholestyramine
and avoid administering cholestyramine within 2 hours of warfarin.
Acetaminophen increases INR at doses higher than 2g/day by reducing
the metabolism of warfarin.

37. D

38. C

39. C
Bortezomib is associated with a high rate of shingles, prophylactic
acyclovir can reduce the risk. Other adverse reactions of bortezomib
include peripheral neuropathy and myelosuppression leading to
neutropenia and thrombocytopenia. It acts as proteasome inhibitor.

40. C
Leucovorin is an active metabolite of folic acid and an essential
coenzyme for nucleic acid synthesis. Leucovorin can be used to
increase the efficacy of 5-fluorouracil or to selectively “rescue” cells
from the adverse effects of methotrexate.
A leucovorin metabolite (5-methyl-tetrahydrofolate [5-MTHF])
stabilizes the bond formed between 5-fluorouracil metabolite
(fluorodeoxyuridine monophosphate) and thymidylate synthetase. This
effect causes a decrease in intracellular levels of the enzyme and a
resulting decrease in the production of thymidylate. In this way,
leucovorin enhances the activity of 5-fluorouracil. Leucovorin is
usually administered just before 5-fluorouracil.
Methotrexate inhibits nucleic acid synthesis by blocking the activation
of folic acid. Leucovorin is folic acid in its active (reduced) form, so it
allows nucleic acid synthesis to proceed even in the presence of
methotrexate. Leucovorin is usually administered 24 hours after
methotrexate so that it does not interfere with the therapeutic effect of
methotrexate.

41. D
The lab results show signs of megaloblastic anemia therefore
cyanocobalamin would be the most appropriate treatment.

42. C

43. C

44. A

45. A
The process of angiogenesis supports tumor growth, therefore as
anticancer agent ramucirumab inhibits angiogenesis.

46. D
Dantrolene belongs to a group of medications known as muscle
relaxants. In addition to its usefulness in the management of malignant
hyperthermia, dantrolene is used to treat chronic muscle spasms caused
by conditions such as cerebral palsy, multiple sclerosis, stroke, or
spinal cord injury.

47. B
Benzocaine is a local anesthetic commonly used as a topical pain
reliever. It is the active ingredient in many anesthetic products for oral
ulcers, sore throat, cold sores, toothache, sore gums and otic pain.
Benzocaine is an ester type local anesthetic. Ester-type local anesthetics
may cause allergic reactions due to their conversion to para-
aminobenzoic acid (PABA). Examples: procaine, cocaine, tetracaine,
novocaine
Amide-type local anesthetics are not converted to PABA, therefore
they are better tolerated. Examples: lidocaine, bupivacaine,
mepivacaine, articaine
Note: Ester-type have 1 I in their name whereas amide-type have 2 Is

48. A
Ureterolithotripsy is the fragmentation of kidney stones into small
pieces to help their clearance.

49. D

50. A
Cold chain begins at the manufacturer and ends with the administration
of the vaccine to the vaccine recipient.

51. D

52. A
A minimum of 15 minutes observation is required to monitor the
incidence of syncope or anaphylaxis.
Overview of Arthus reaction: An Arthus reaction is a large, localized
reaction characterized by pain, swelling, induration and edema. It
usually begins within 48 hours following immunization and develops
gradually over a period of hours. The reaction is due to circulating
antigen-antibody complexes formed when there is a large amount of
circulating antibodies prior to injection of the antigen. This results in
massive swelling at the injection site that may involve the entire limb.
Arthus reactions may be seen with too frequent boosters of tetanus-
containing vaccines, and they have been observed following repeat
doses of pneumococcal polysaccharide vaccine after short internals.
NEW Questions – 2019

1.The Canadian Federal Government has set the minimum age for
recreational cannabis use at 18 years. However, all provinces have
raised the minimum age to 19 years except for:

A. British Columbia and Ontario


B. Manitoba and New Brunswick
C. Alberta and Quebec
D. Saskatchewan and Nova Scotia

2.Which of the following statements is the recommended strategy in the


treatment of neuropathic pain using cannabinoids?

A. After failure of ≥1 other drug, a trial of prescription cannabinoids


rather than cannabis may be reasonable
B. After failure of ≥1 other drug, a trial of prescription cannabinoids
or cannabis may be reasonable
C. After failure of ≥3 other drugs, a trial of prescription
cannabinoids rather than cannabis may be reasonable
D. After failure of ≥3 other drugs, a trial of prescription
cannabinoids or cannabis may be reasonable
3.You are a hospital pharmacist. A pediatric physician, Dr. Kim is
wondering if his 6 years old patient suffering from seizures associated
with Lennox-Gastaut syndrome would be a good candidate for
cannabinoid treatment. You inform Dr. Kim that the use of
cannabinoids is recommended in patients older than:

A. 2 years
B. 5 years
C. 8 years
D. 12 years

4.Inhaled cannabis is not recommended due to:

A. Difficulty dosing, risk of respiratory damage, and multi-


component composition
B. Cost, fast onset of action and multi-component composition
C. Risk of respiratory damage, high risk of addiction, and multi-
component composition
D. Fast onset of action, short duration of action and multi-
component composition

5.With the legalization of cannabis in Canada, pharmacists are advised


to incorporate screening for cannabis use disorder into daily practice.
Which of the following symptoms is NOT considered a sign of
cannabis use disorder?

A. Poor motivation
B. Respiratory problems
C. Euphoric state
D. Reduced work performance

6. A 28 years old male naïve cannabis user is considering trying


cannabis edibles. Which of the following is an appropriate
statement?

A. Cannabis edibles are legal from a licensed producer


B. Cannabis edibles require a prescription
C. Cannabis edibles are not legal for purchase
D. Cannabis edibles are legal from an authorized retail store

7. In 2018, the proportion of adults Canadian who reported using


cannabis daily is:
A. 6%
B. 14%
C. 26%
D. 33%

8. You are a hospital pharmacist. A newly admitted patient medical


record shows:
36.5 ºC body temperature on admission; Normal range = 35.8ºC to
37.3ºC
The normal range shown is based on:
A. Oral temperature
B. Tympanic temperature
C. Rectal temperature
D. Axillary temperature

9. A healthy patient will have an oxygen saturation (SpO2) ≥ 97%.


Oxygen saturation is measured using a pulse oximeter sensor attached
to the patient’s:
A. Toe or Finger
B. Finger or Earlobe
C. Earlobe or Toe
D. Toe, Finger or Earlobe

10. In general, the symptoms of cannabis withdrawal onset in 1-2 days


and peak in 2-6 days. Which of the following is NOT a sign of cannabis
withdrawal?

A. Irritability
B. Muscle twitching
C. Sleep disturbance
D. Weight loss

11. The current "two-stream" cannabis system in Canada includes


medically authorized cannabis and cannabis through retail sale. Which
of the following statements is NOT accurate regarding Medical
Cannabis?

A. Accessed exclusively by mail or courier


B. Dosing control is similar to prescription cannabinoids
C. Covered occasionally by private insurance
D. Veteran’s affairs coverage, max 3 g daily dried cannabis

12. Which of the following statements best describes the term “medical
cannabis”?

A. Products derived from the whole cannabis plant or its extracts


containing a variety of active cannabinoids and terpenes, which
patients take for medical reasons, after interacting with and
obtaining authorization from their health care practitioner.
B. Products derived from the whole cannabis plant containing a
variety of active cannabinoids which patients take for medical
reasons, after interacting with and obtaining a written prescription
from their health care practitioner.
C. Products containing pure synthetic analogues of cannabis active
ingredients which patients take for medical reasons, after
interacting with and obtaining authorization from their health
care practitioner.
D. Products containing cannabis plant extracts and terpenes which
patients take for medical reasons, after interacting with and
obtaining a written prescription from their health care
practitioner.

13. Mr. Smith has recently obtained medical authorization from his
physician to access cannabis for the management of neuropathic pain.
He is visiting the pharmacy to seek your advice on how to obtain
cannabis for medical purpose. Which of the following options will
NOT be recommended to Mr. Smith?

A. Licensed producer listed on Health Canada’s website


B. Register with Health Canada to grow a limited amount for his
own use
C. Designate someone to produce cannabis for him
D. From his local pharmacy

14. Which of the following statements is an accurate description of


cannabis?

A. Therapeutic product containing cannabinoids


B. CBD oil
C. Cannabinoid containing product with DIN
D. Nabilone

15. Lennox-Gastaut syndrome (LGS) is a severe form of childhood


epilepsy that is defined by generalized multiple type seizures, slowness
of intellectual growth, and a specific electroencephalogram disturbance.
Which of the following cannabinoids is indicated for the treatment of
seizures associated with Lennox-Gastaut syndrome?

A. Nabiximols
B. Nabilone
C. CBD
D. Cannabis

16. Mr. Lenn is a naïve cannabis user who is seeking your counselling.
Your counselling will NOT include which of the following statements?

A. Oral cannabis has slower onset compared to inhaled cannabis


B. Vaped cannabis is as potent as smoked cannabis
C. Inhaled cannabis has shorter duration of action than oral cannabis
D. Smoked cannabis has higher risk of respiratory adverse effect
than vaped cannabis

17. Mrs. Peter had two episodes of deep vein thrombosis. She has been
using warfarin for the past 9 months for prevention. DVT is the most
common type of venous thrombosis. Mrs. Peter is visiting the
pharmacy to discuss her interest in recreational cannabis. Which of the
following cannabinoids may induce CYP 1A2 resulting in decreased
levels of warfarin?

A. Vaped cannabis
B. Oral THC/CBD
C. Oral CBD
D. Smoked cannabis

18. Which of the following statements best describes passive


immunity?

A. Immune response is mediated by antibodies produced by the


body’s B-cells as effector molecules. Both B- and T-cells may be
involved in this response.
B. Immune response is mediated the body's immune system to
remember an encounter with a specific antigen and to react more
swiftly to the antigen in a later encounter.
C. Immune response is mediated by antibodies produced by another
human being or animal. Protection is generally limited and
diminishes over time.
D. Immune response is mediated by IgM antibodies and stimulated
by an encounter with an antigen for the first time. Immunologic
memory is produced.

19. Which of the following statements best describes booster


immunization?

A. Attempt to achieve maximum immunization coverage of the


population
B. A second immunization with a specific vaccine to insure
maximum protection
C. Immunization program aimed at a specific group or population
D. Immunizations that are required by law in some provinces or
territories
20. The surveillance of vaccine safety helps maintain public confidence
in vaccines and immunization programs. Syndromic surveillance is best
described as:

A. The surveillance that is carried out after the vaccine has been
approved for sale to the general public. This type of surveillance
allows the identification of rare adverse events.
B. A system of surveillance where the responsible agency relies on
mandatory reporting by front-line healthcare providers or
agencies, and the responsible agency does not stimulate
reporting.
C. An active case-finding surveillance based on a regular review of
hospital admission records. Canada’s pediatric active surveillance
system is an example of this type of vaccine monitoring.
D. The surveillance of health data to detect epidemics, monitor their
impact on public health, characterize affected populations, and
monitor the effectiveness of response to the epidemic.

21. Which of the following statements best describes an immunization


record?

A. It consolidates immunization from multiple sources, including


any reports of adverse events, into one confidential record.
B. It is a record of all immunization a person has received. This
record is produced in two forms, professional chart and take-
home.
C. It outlines the optimum timing of immunization when primary or
secondary immunization is received after the recommended age.
D. It conveys whether a patient is eligible, due or overdue for a
specified vaccine.

22. An adjuvant is a substance such as aluminum salt added to a


vaccine. An adjuvant acts to:

A. Increase the release of antigens, stimulate local inflammation to


attracts immune cells to the site, and reduce the physical size of
the antigen for more efficient uptake by antigen processing cells.
B. Slow the release of antigens, reduce local inflammation to attracts
immune cells to the site, and reduce the physical size of the
antigen for more efficient uptake by antigen processing cells.
C. Increase the release of antigens, reduce local inflammation to
attracts immune cells to the site, and enlarge the physical size of
the antigen for more efficient uptake by antigen processing cells.
D. Slow the release of antigens, stimulate local inflammation to
attracts immune cells to the site, and enlarge the physical size of
the antigen for more efficient uptake by antigen processing cells.

23. Cannabis math: What is the estimated THC dose if 1 joint


containing 0.5 grams of 12% THC dried cannabis is smoked? Assume
50% loss by combustion.

A. 5 mg
B. 15 mg
C. 30 mg
D. 60 mg
Answers

1.C

2. C
Prescription cannabinoids are preferred due to more efficient dosing.

3. A

4. A

5. C
Euphoria is not a sign of disorder instead it is a common reaction to
cannabis. Other signs of cannabis use disorder include problematic
relationships and poor academic performance.

6. C
Current 2018 legal status in Canada: prescription cannabinoids are
Schedule II (controlled substances). Dried cannabis and oils are legal
from a licensed producer with physician authorization, or from a
cannabis retail store. Cannabis edibles are not legal for purchase.

7. A
Prevalence of cannabis use in 2018: 14% of Canadian adults used
cannabis in the last 3 months, 6% used daily and 1% were registered
for medical use.

8. A
Axillary temperature: Usually 1ºC lower than oral temperature.
Tympanic membrane (ear) temperature: Usually 0.3°C to 0.6°C higher
than oral temperature.
Rectal temperature: Usually 1ºC higher than oral temperature.

9. D
Finger and earlobe are most commonly used. If a pediatric oximeter is
not available, an adult probe can be used on child’s thumb or big toe. If
the patient has nail polish or henna dye on fingers, a toe can be used.

10. B
Muscle twitching is a common adverse effect of cannabis; other
adverse effects are feeling “high”, sedation, speech disorders, dizziness
and euphoria. Other symptoms of cannabis withdrawal are anger,
physical discomfort, cannabis craving, restlessness, appetite change,
anxiety and aggression.

11. B
Less dosing control compared to prescription cannabinoids.

12. A
13. D
Currently, pharmacists do not dispense medical cannabis.

14. B
Cannabinoids are all therapeutic products containing cannabinoids;
Cannabis is dried cannabis plant material or cannabis extracts without a
DIN such as CBD oil; Prescription cannabinoids are cannabinoid-
containing medications with a DIN such as nabilone or nabiximols.

15. C
THC and CBD are the two primary substances found in cannabis.
Nabiximols (Sativex) is extracted THC/CBD used to treat advanced
cancer pain, Multiple Sclerosis neuropathic pain or spasticity; Nabilone
(Cesamet) is THC analogue used to treat severe nausea and vomiting
due to cancer chemotherapy, AIDS-related anorexia, palliative pain and
neuropathic pain.

16. B
Vaped cannabis is two times more potent than smoked cannabis;
smoking destroys the active ingredients by combustion.

17. D
All cannabinoids have additive CNS effects with alcohol,
benzodiazepines, opioids etc.

18. C
A refers to humoral immunity; B refers to memory or secondary
immune response; D refers to primary immune response.

19. B
A refers to universal immunization; C refers to targeted immunization;
D refers to mandatory immunization.

20. D
A refers to post-marketing surveillance; B refers to passive
surveillance; C refers to active surveillance.
21. B
A refers to immunization registry; C refers to delayed immunization
schedule; D refers to immunization status.

22. D
An adjuvant increases the body’s immune response to a vaccine.

23. C
Amount of THC in 1 joint. 500 mg cannabis x 12% THC = 60 mg
If 50% loss to combustion, then the amount of THC smoke is: 60 mg
THC x 50% = 30 mg
Appendix A: Common substances withdrawal syndromes

Substance Withdrawal syndromes


Benzodiazepines Insomnia
Hand tremor
Increased pulse
Sweating
Hallucinations (tactile, auditory or
visual)
Nausea, vomiting
Psychomotor agitation
Anxiety
Grand mal seizures
Alcohol Insomnia
Hand tremor
Depression
Increased pulse
Sweating
Hallucinations (tactile, auditory or
visual)
Nausea, vomiting
Psychomotor agitation
Grand mal seizures
Cocaine/Amphetamines Sleep disturbances (hypersomnia
or insomnia)
Unpleasant dreams
Agitation
Increased appetite
Depression
Fatigue
Psychosis with amphetamines
Opioids Mood alteration
Rhinorrhea
Fever
Pupillary dilation
Piloerection
Lacrimation
Sweating
Fever
Muscles cramps
Diarrhea
Yawning

Appendix B: Drugs known to induce dysglycemia

Drug Net effect on blood glucose


Atypical antipsychotics Hyperglycemia
HIV protease inhibitors Hyperglycemia
Beta blockers Hyperglycemia
Glucocorticoids Hyperglycemia
Salicylates Hypoglycemia
Thiazide diuretics Hyperglycemia
Estrogens Hyperglycemia
Interferon alpha Hyperglycemia
Nicotinic acid Hyperglycemia
Pentamidine Hypoglycemia
Phenytoin Hyperglycemia
Valproic acid Hyperglycemia
Alcohol Hypoglycemia

Appendix C: Risk factors for Diabetes mellitus


- History of impaired fasting glucose
- History of impaired glucose tolerance
- Family history with diabetes
- Overweight
- Hypertension
- Vascular disease
- Schizophrenia
- Polycystic ovarian syndrome
Appendix D: Drugs known to induce hyperlipidemia

- Beta blockers
- Oral contraceptives
- Thiazide diuretics
- Antiretroviral agents
- Corticosteroids
- Hormone replacement therapy
Appendix E: Conditions known to induce hyperlipidemia

- Pregnancy
- Overweight
- Hypothyroidism
- Excessive alcohol intake
- Liver disease
- Nephrotic syndrome
- Diabetes
- Metabolic syndrome
- Renal failure
Appendix F: Drugs known to enhance statins myotoxicity

- Cyclosporine
- Azole antifungals
- Macrolide antibiotics
- Fibrates (mostly gemfibrozil)
- HIV protease inhibitors
- Amiodarone
- Non-dihydropyridine calcium channel blockers
Appendix G: Drugs known to increase blood pressure
- NSAIDs
- Acetaminophen
- Oral contraceptives
- Sex hormones
- Decongestants
- Corticosteroids
- Calcineurin inhibitors (e.g. tacrolimus, cyclosporine)
- Erythropoietin
- Midodrine
- Monoamine oxidase inhibitors
- Stimulants (e.g. cocaine, caffeine, amphetamines, methylphenidate)
- Antidepressants (e.g. venlafaxine, bupropion, desipramine)
- Some herbal products :

Arnica
Bitter orange
Ephedra
Ginkgo
Ginseng
Guarana
Licorice
Senna
St. John's wort
Appendix H: Drugs known to induce weight gain

- Corticosteroids
- Insulin
- Sulfonylureas
- Thiazolidinediones
- Antihistamines
- Antiepileptics (valproic acid, phenytoin)
- Antidepressants
- Antipsychotics
- Oral contraceptives
Appendix I: Common causes of diarrhea

- Bacterial infections
- Viral infections
- Parasitic infections
- Drugs: antibiotics, colchicine, laxatives, magnesium containing
antacids
- Food intolerance: lactose
- Intestinal conditions: Crohn’s disease, ulcerative colitis, celiac disease
Appendix J: Drugs known to induce constipation

- Anticholinergic agents
- Anticonvulsants
- Diuretic
- Antipsychotics
- Parkinson’s drugs
- Calcium channel blockers
- Iron supplements
- Opioids
- Resins (cholestyramine, colestipol)
- Sucralfate
- Aluminum and calcium containing antacids
- Bismuth preparations

Appendix K: Drugs known to induce erectile dysfunction


- Monoamine oxidase inhibitors
- Alpha blockers
- Barbiturates
- Phenothiazines
- Selective serotonin receptor inhibitors
- Tricyclic antidepressants
- Thiazides
- Acetazolamide
- Atenolol
- Carbamazepine
- Cimetidine
- Clofibrate
- Clonidine
- Digoxin
- Finasteride
- Ketoconazole
- Labetalol
- Lithium
- Methadone
- Metoclopramide
- Phenytoin
- Propranolol
- Spironolactone
Appendix L: Drugs known to interact with fiber (roughage
food)

- Tricyclic antidepressants
- Antihyperglycemics (specifically metformin and glyburide)
- Statins
- Carbamazepine
- Digoxin
- Lithium
- Penicilllin
Appendix M: Highly emetogenic drugs

- Carmustine
- Cisplatin
- Cyclophosphamide
- Cytarabine
- Dacarbazine
- Dactinomycin
- Lomustine
- Mechlorethamine
- Streptozocin
Appendix N: Calcium reduces the absorption of the following
drugs

- Beta blockers
- Calcium channel blockers
- Bisphosphonates
- Digoxin
- Quinolones
- Gentamicin
- Tetracyclines
- Phenytoin
- Carbamazepine
- Iron

Appendix O: Drugs associated with osteoporosis


- Anticoagulants (heparin)
- Anticonvulsants
- Cyclosporine
- Tacrolimus
- Cancer chemotherapy drugs
- Glucocorticoids
- Gonadotropin-releasing hormone agonists
- Lithium
- Methotrexate
- Parenteral nutrition
- Thyroxine
Appendix P: Drugs known to induce photosensitivity

- Amiodarone
- Amitriptyline
- Alprazolam
- Barbiturates
- Carbamazepine
- Captopril
- Corticosteroids
- Chlordiazepoxide
- Chloroquine
- Dacarbazine
- Fluoroquinolones (varying degree)
- Fluorouracil
- Furosemide
- Griseofulvin
- Haloperidol
- Hydrochlorothiazide
- Isotretinoin
- Methotrexate
- NSAIDs (varying degree)
- Phenothiazines
- Promethazine
- Quinidine
- Quinine
- Spironolactone
- Sulfonamides
- Sulfonylureas
- Tetracyclines
- Tricyclic antidepressants
- Vinblastine

Appendix Q: Drugs and other substances associated with the


incidence of depression

- Alcohol
- Amphetamine withdrawal
- Amphotericin B
- Antipsychotic drugs
- Beta-blockers (some)
- Cimetidine
- Contraceptives (oral)
- Corticosteroids
- Cycloserine
- Hormone (estrogen) therapy
- Interferon
- Mercury
- Methyldopa
- Metoclopramide
- Reserpine
- Thallium
- Vinblastine
- Vincristine
Appendix R: Drugs associated with the incidence of syncope

Mechanism Drugs
Bradyarrhythmia Amiodarone
β-blockers
Ca channel blockers (not dihydropyridines)
Digoxin

Tachyarrhythmia Antiarrhythmic drugs


Quinidine
Orthostatic Most antihypertensives (rarely β-blockers)
hypotension Antipsychotics (primarily phenothiazines)
Doxorubicin
Levodopa
Loop diuretics
Nitrates (mostly in combination with
phosphodiesterase inhibitors for erectile
dysfunction)
Quinidine
Tricyclic antidepressants
Vincristine

Appendix S: Types of hypersensitivity reactions

Characteristics type-I type-II type-III type-IV


(anaphylactic) (cytotoxic) (immune (delayed
complex) type)
antibody IgE IgG, IgM IgG, IgM None
antigen exogenous cell surface soluble tissues &
organs
response time Immediate - minutes-hours 3-8 hours 48-72
few minutes hours
examples allergic Drug allergies farmer's tuberculin
asthma, hay lung test,
fever erythroblastosis disease poison
Goodpasture's ivy,
nephritis contact
dermatitis

Appendix T: Drugs associated with immunodeficiency

Type Examples
Anticonvulsants Carbamazepine

Phenytoin

Valproate

Immunosuppressants Azathioprine
Cyclosporine

Tacrolimus

Corticosteroids Methylprednisolone

Prednisone
Chemotherapy drugs Busulfan

Cyclophosphamide

Melphalan

Monoclonal antibodies Muromonab

Appendix V: Drugs associated with folic acid deficiency


Anticonvulsants such as phenytoin, carbamazepine, valproic acid
Metformin
Methotrexate
Sulfasalazine
Triamterene
Oral Contraceptives
Appendix W: Generic Naming of Cancer Targeted Therapies

Adapted from Cancer Genome


Appendix X: Types of Targeted Cancer Therapies

Monoclonal antibodies
Trastuzumab Used to treat HER2-positive cancers including breast,
stomach and esophageal cancers.

Bevacizumab Used to treat cancers that overexpress VEGF receptor


proteins including cervical, colorectal and ovarian cancers.

Rituximab Used to treat types of non-Hodgkin lymphoma that have


high numbers of abnormal B lymphocytes.

Cetuximab Used to treat cancers that overexpress EGFR protein


including colorectal cancer and some types of head and neck
cancers.

Tyrosine kinase inhibitors


Imatinib Used to treat some types of leukemia that carry the Philadelphia
chromosome.

Sunitinib Used to treat gastrointestinal stromal tumour (GIST), kidney


cancer and pancreatic cancer.

Gefitinib Used to treat non–small cell lung cancer and other types of
cancer that overexpress EGFR proteins.

Apoptosis-inducing drugs
Bortezomib Proteasome inhibitor. Used to treat multiple myeloma and
mantle cell lymphoma.

Oblimersen Antisense drug. Used to treat cancers that overexpress BCL2


protein including lymphomas, breast, colon and prostate
cancers.

Olaparib poly ADP-ribose polymerase (PARP) inhibitor. Use to treat


advanced BRCA-mutated ovarian cancer

Angiogenesis inhibitors
Bevacizumab Bevacizumab is also a type of monoclonal antibody. Refer
above
Sunitinib Sunitinib is also a type of tyrosine kinase inhibitor. Refer
above
Thalidomide Used to treat multiple myeloma

mTOR inhibitors
Temsirolimus and Everolimus: Used to treat kidney cancer and some
neuroendocrine tumours.

Hormonal therapies
Tamoxifen Antiestrogen for breast cancer

Flutamide Antiandrogen for prostate cancer

Goserelin LHRH analogue for breast and prostate cancers

Leuprolide GnRH analogue for breast and prostate cancers

Appendix Y: Properties of Cannabinoids

Inhaled Oral Cannabis: Pharmaceutical


Cannabis: plant oil or plant (eg. Cannabinoids
or oil smoked or Edibles) (e.g. Nabilone,
vaporized Nabiximols)
Onset of effect 5 to 10 min Up to 60 min 1 to 2 hrs
Peak effect 10 to 20 min 2 to 4 hrs 2 to 4 hrs
Duration of 2 to 4 hrs (up to Adults: 4 to 6 hrs 12 hrs (up to 24
effect 24 hrs) (up to 24 hrs) hrs)
Kids: 6 to 12 hrs
(up to 36 hrs)
Appendix Z: Physical Assessment Skills

Body Temperature
Normal (oral) = 35.8ºC to 37.3ºC
Oral temperature: Place the thermometer in the mouth under the tongue and
instruct patient to keep mouth closed. Leave the thermometer in place for as
long as is indicated by the device manufacturer.
Axillary temperature: Usually 1ºC lower than oral temperature. Place the
thermometer in patient’s armpit and leave it in place for as long as is
indicated by the device manufacturer.
Tympanic membrane (ear) temperature: Usually 0.3°C to 0.6°C higher than
an oral temperature. Do not force the thermometer into the ear and do not
occlude the ear canal.
Rectal temperature: Usually 1ºC higher than oral temperature. Use only
when other routes are not available.

Pulse
Normal resting heart rate = 60 to 100 beats per minute
Radial pulse: Use the pads of your first three fingers to gently palpate the
radial pulse at the inner lateral wrist.
Apical pulse: Taken as part of a cardiovascular assessment and when the
pulse rate is irregular. Apical pulse rate should be taken for a full minute for
accuracy and is located at the fifth intercostal space in line with the middle
of the clavicle in adults.
Carotid pulse: May be taken when radial pulse is not present or is difficult
to palpate.

Respiratory Rate (RR)


Normal resting respiratory rate = 10 to 20 breaths per minute
Count respiratory rate while you are taking the pulse rate so that the patient
is not aware that you are taking the respiration rate. Count for 30 seconds or
for a full minute if irregular.
Blood Pressure (BP)
The average BP for an adult is 120/80 mmHg, but variations are normal for
various reasons.
Systolic pressure is the maximum pressure on the arteries during left
ventricular contraction.
Diastolic pressure is the resting pressure on the arteries between each
cardiac contraction.
The patient may be sitting or lying down with the bare arm at heart level.
Palpate the brachial artery just above the antecubital fossa
medially. Wrap the BP cuff around the upper arm about 2.5 cm above the
brachial artery.
Palpate the radial or brachial artery and inflate the BP cuff until the pulse
rate is no longer felt. Then inflate 20 to 30 mmHg more.
Place the bell of the stethoscope over the brachial artery, and deflate the
cuff slowly and evenly, noting the points at which you hear the first
appearance of sound (systolic BP), and the disappearance of sound
(diastolic BP).

Oxygen Saturation (SpO ) 2

A healthy patient will have an SpO of ≥ 97%


2

A pulse oximeter sensor attached to the patient’s finger or earlobe measures


light absorption of hemoglobin and represents arterial SpO . 2

Sample of PEBC Qualifying Exam Part II (OSCE)


Cases
Canadian Pharmacy Exams™ - Pharmacist OSCE Workbook available now
at Amazon!

Interactive case #1 (Pharmacist – Standardized Physician Interaction)

Case description:

Dr Russer is waiting in the pharmacy to talk to the pharmacist regarding her


patient’s therapeutic regimen. Dr. Russer’s patient Patrick Fung has been
diagnosed with generalized seizures about 3 months ago. Patrick Fung has
been taking lamotrigine for the past 3 months; he is already on a maintenance
dose of 300 mg bid. Unfortunately, he is experiencing recurrent seizures. Dr.
Russer would like to discuss with the pharmacist if switching to another drug
would be beneficial. The candidate is expected to identify the best course of
actions(s). The candidate may interact with the physician to obtain additional
information and clarify any concern(s). A patient’s profile is provided.

Primary goals:

- Identify that, in this case, the recurrence of seizures is probably due to


sleep starvation (bar and night club musician) and lack of compliance
(the profile is showing a missed scheduled refill). Alcohol reduction
could be also beneficial.
- Recommend a re-counselling session.

Patient’s profile

Name: Patrick Fung


Gender: Male
Age: 27 years old
Medical History: Generalized seizures diagnosed 3 months ago
Allergies: None
Current Medications (Rx & nRx): Lamotrigine 300 mg bid - missed
scheduled refill
Social/lifestyle: Bar and night club musician, non-smoker, moderate alcohol
intake – 5 to 8 drinks weekly, physically active – visits the gym at least 3
times a week

The candidate is expected to:

Identify that, in this case, the recurrence of seizures is probably due to sleep
starvation (bar and night club musician) and lack of compliance (the profile is
showing a missed scheduled refill).

Recommend alcohol reduction. Alcohol interferes with the effectiveness of


antiseizure drugs

Recommend a re-counselling session.

Explain that the usual maximum maintenance dose of lamotrigine is 400mg,


therefore increasing the dose should be considered first before switching to
another drug.

Explain that if later switching to another drug becomes appropriate, add


either phenytoin, valproic acid or carbamazepine then withdraw lamotrigine
by tapering the dose. Lamotrigine, valproic acid, phenytoin and
carbamazepine are drugs of choice for monotherapy.

Advise monitoring blood levels of lamotrigine


Interactive case #2 (Pharmacist – Standardized Physician Interaction)

Case description:

Dr David is waiting in the pharmacy to talk to the pharmacist. She would like
to discuss a new prescription for her patient Karyn Bob for the prevention of
acute and delayed emesis. Karyn has been scheduled to be treated with
cisplatin which is a highly emetogenic chemotherapeutic drug. The candidate
is expected to review the prescription and identify any drug related
problem(s). The candidate may interact with the physician to obtain
additional information and clarify any concern(s). The candidate is also
expected to record recommendations and any change(s) on the prescription
review sheet. A patient’s profile is provided.

Primary goals:
- Recommend changing lorazepam to aprepitant.
- Identify aprepitant-dexamethasone interaction
- Recommend a lower dose of dexamethasone

Prescription
Prescription Review Sheet
Patient’s profile

Name: Karyn Bob


Gender: Female
Age: 47 years old
Medical History: Breast cancer – recently diagnosed
Allergies: Seasonal
Current Medications (Rx & nRx): 1 daily multivitamin, tylenol prn for pain
Social/lifestyle: Non-smoker, no alcohol intake, physically active – gentle
yoga 3 times a week

The candidate is expected to:

Recommend changing lorazepam to aprepitant (Emend).

Recommend the following dosage of aprepitant (Emend):

125 mg orally 1 hour prior to chemotherapy treatment


(Day 1) and 80 mg once daily in the morning on Days 2 and 3.

Explain that based on clinical studies neurokinin-1 (NK1) antagonists such as


aprepitant added to dexamethasone and a serotonin antagonist such as
granisetron improve the prevention of acute and delayed emesis due to
highly emetogenic chemotherapy.

Benzodiazepines such as lorazepam are primarily used for anticipatory


nausea.

Explain that since aprepitant is a CYP3A4 inhibitor, it increases the blood


levels of dexamethasone. The dose of dexamethasone should not exceed 50%
of the maximum dose of 10 mg. Change the dose of dexamethasone to 5 mg.
Interactive case #3: Osteoarthritis pain control (Pharmacist – Patient
Interaction)

Case description:

Mrs. Ahmed is your regular patient. She has been diagnosed with
osteoarthritis 3 years ago and is currently using Tylenol to manage her
symptoms. She is seeking your assistance on the selection of a
nonprescription topical analgesic for added relief. One of her friends, who is
also suffering from osteoarthritis told her that Bengay is effective and she has
been using it for almost 7 months. Mrs. Ahmed is hesitant using Bengay. She
is seeking the pharmacist’s assistance to select one of the following products:
Bengay, Zostrix or Rub A-535.

Primary goal:

- Recommend not to use Bengay or Rub A-535 due to her aspirin


allergy. Bengay and Rub A-535 contain methyl salicylate
- Recommend Zostrix (capsaicin)
Patient profile

Patient Name: Rena Ahmed


Gender: Female
Age: 66 years old
Allergies: Aspirin
Medical History: Osteoarthritis, Atopic dermatitis –controlled by the
avoidance of known irritants and application of moisturizers
Current medications: Tylenol 650 mg Q6H, 1 multivitamin daily
Social/lifestyle: Non-smoker, no alcohol use, physically active – walks daily
for 40 minutes

If asked, the standardized patient will provide the following additional


information:

Overall her knee pain is well controlled.


She would like to use a topical product for added comfort in the morning.

The candidate is expected to:

Confirm that the patient is allergic to aspirin.

Explain that due to her allergy, she should not use Bengay and Rub A-535
because both contain aspirin.

Recommend Zostrix instead.

Recommend applying Zostrix to the affected knee 3 to 4 times daily.

Warn to avoid contact with eyes.


Discuss the incidence of transient burning on application of Zostrix

Encourage the patient to continue exercising and discuss the benefit of


physical activity. If applicable, recommend body weight reduction

Summarize and assess patient’s understanding

Non-interactive case #4: Checking prescription labels


Identify any mistakes, omissions or concerns on each prescription label.
Assume each prescription is correct and omission of auxiliary labels,
pharmacy contact information or practitioner ID is not to be considered a
mistake.

Written Rx 1
Dispensed Rx 1 label
Written Rx 2
Dispensed Rx 2
label
Written Rx 3
Dispensed Rx 3 label

Answer key

Rx 1 Is this prescription label consistent


with the prescription?
No

What would you correct?

Wrong drug formulation.

Rx 2 Is this prescription label consistent


with the prescription?

No

What would you correct?

Wrong duration of treatment.


Wrong patient’s name.

Rx 3 Is this prescription label consistent


with the prescription?

No

What would you correct?

Wrong administration route


Non-interactive case #5: Checking new prescriptions

Identify any mistakes, omissions or concerns on each new prescription. Some


prescriptions have more than one problem. Omission of practitioner ID is not
to be considered a mistake.

Written Rx 1
Written Rx 2
Written Rx 3

Answer key
Rx 1 Is this prescription ready to be
processed and filled?

No

What would you correct or add?

Missing physician’s signature


Add: 1 hour or 4-6 hours after
other medications. Increase fiber
and fluid intake to prevent
constipation.

Rx 2 Is this prescription ready to be


processed and filled?

No

What would you correct or add?

Wrong dosing. 1 patch q72h. 1.5


mg patch.
Add: apply the patch behind the
ear.

Rx 3 Is this prescription ready to be


processed and filled?

No
What would you correct or add?

Wrong number of tablets

84 tablets instead of 86 tablets

You might also like